Links 5/20

[Epistemic status: I haven’t independently verified each link. On average, commenters will end up spotting evidence that around two or three of the links in each links post are wrong or misleading. I correct these as I see them, but can’t guarantee I will have caught them all by the time you read this.]

The Movement For The Restoration Of The Ten Commandments Of God had such a strong emphasis on the Commandments that they “discouraged talking, for fear of breaking the Eigth Commandment [against bearing false witness], and on some days communication was only conducted in sign language.” Pretty impressive, although I feel like they might have departed from the Decalogue a tiny bit at the part where they murdered 530 people.

The 1960s were simpler times, when ads in kids’ magazines offered to sell you a pet monkey for $19.95. “My brother is 8, I am 9 years old…and we had $19.95 because we washed cars, we mowed lawns.” What could go wrong?

Gwern on hard to notice ways that things have improved during his lifetime (ie since the late 1980s). Some things are big technology, like the Internet and electric cars. Other things are tiny improvements in everyday objects, like self-adhesive stamps, power windows in cars, wheeled luggage, TVs you don’t have to adjust the antennae on, and computer mice you don’t have to remember to clean. Radios stopped being staticky, air travel got cheaper, showers don’t run out of hot water. Still others are vast vague improvements in whole areas of life, like cleaner air and water, or Amazon-style improved logistics. Recommended.

More sentimental cartography: a map of linguistics (h/t Copular Predicate)

In the 1500s, small forces of Europeans took over large chunks of the world, most famously Cortes and Pizarro in the Americans. But what about Portuguese conquerer Afonso de Albuquerque? In 1506 – just eight years after the first European ships rounded Africa and made it to the Indian ocean at all – he and the Portugese king made a plan to conquer enough Asian coastline to control trade on the Indian Ocean. Over the next seven years, they did exactly this, taking over choice ports like Hormuz (Iran), Goa (India), and Malacca (Malaysia). Unlike the Spanish conquistadors, who had the advantage of using guns while facing Stone Age empires, Afonso generally faced enemies as advanced (and sometimes more advanced) than himself. How did he do it? Daniel Kokotajlo on Less Wrong writes about the lessons from Afonso and the other conquistadors.

The McLibel Case – activists Helen Steel and David Morris distributed anti-McDonalds pamphlets in the UK. McDonalds sued them for libel, starting a case that would cast international attention on the UK’s unusually strict and freedom-of-speech-threatening libel laws. My favorite part of this article is about a proposed settlement: “Steel and Morris secretly recorded the meeting, in which McDonald’s said the pair could criticise McDonald’s privately to friends but must cease talking to the media or distributing leaflets. Steel and Morris wrote a letter in response saying they would agree to the terms if McDonald’s ceased advertising its products and instead only recommended the restaurant privately to friends.”

538 on which states have produced the most presidential nominees. Did you know eight nominees have come from the western US, and all of them were Republicans? Why should that be?

“At this point it’s been pretty conclusively established that the ocean is weird, but one of weirder marine phenomena I’ve encountered is the sea monk or sea bishop, an animal that was sighted of the coast of Poland in 1531, washed up on Danish shores in the late 1540s and went the 16th century equivalent of viral.” Even if you don’t read this one, at least look at the pictures!

Update: new meta-analysis out of Stanford reviews 35 studies and claims that Alcoholics Anonymous is the most effective path to abstinence, and “was nearly always found to be more effective than psychotherapy in achieving abstinence”. Read this in the context of my previous article Alcoholics Anonymous: Much More Than You Wanted To Know. Overall I am glad to have some evidence to use against the Internet people who always say “science has debunked Alcoholics Anonymous”, but I’m not convinced it’s the right solution for everyone, or necessarily better than any other equally-structured programs. I may have more to say when I’ve read the study in more detail.

This Twitter thread combines a discussion of Bay Area zoning policy with one of the best puns I have ever seen (the Bay won’t let someone rezone something, and this gets described as “an unrezonable decision”).

Marvel introduces two new social-justice-themed superheroes, Snowflake and Safespace (also, they’re black and nonbinary). The writer swears he is not making fun of anyone and actually thinks this is a good idea.

The burned house horizon is the area in Europe where people from 7000 BC to 2000 BC often deliberately burned their own houses down for no apparent reason. Contains modern Romania, Bulgaria, Ukraine, etc. The “Cucuteni-Trypillian culture” built the largest cities in history up to that date, then burned them down every 75 years or so, consistently, for centuries. “Whether the houses were set on fire in a ritualistic way all together before abandoning the settlement, or each house was destroyed at the end of its life (e.g. before building a new one) it is still a matter of debate…some scholars have theorized that the buildings were burned ritually, regularly and deliberately in order to mark the end of the “life” of the house. The terms ‘domicide’ and ‘domithanasia’ have been coined to refer to this practice.” Also, “although there have been some attempts to try to replicate the results of these ancient settlement burnings, no modern experiment has yet managed to successfully reproduce the conditions that would leave behind the type of evidence that is found in these burned Neolithic sites, had the structures burned under normal conditions”

This is several Weird Venezuelan Happenings ago now, but hopefully you didn’t miss the time last month when a Venezuelan warship attacked an unarmed cruise ship for unclear reasons, but managed to sink itself in the process instead. See also this mock Wikipedia infographic about the battle.

I stumbled across this 2012 Siddhartha Mukherjee piece on depression recently. Even though it’s eight years old, it does a better job than most modern pop science in navigating the successes and failures of SSRIs and the serotonin hypothesis of depression more generally.

Moore’s Law is pretty great, but “in many areas, performance gains due to improvements in algorithms have vastly exceeded even the dramatic performance gains due to increased processor speed”.

The late 1980s saw the “heterosexual AIDS panic”, where people started worrying AIDS would devastate the straight community to the same degree as the gay community. At its peak, Oprah told her audience that “research studies now project that one in five heterosexuals could be dead from AIDS at the end of the next three years”, and the Secretary of Health and Human Services claimed AIDS could be worse than the Black Death, which killed a third of Europeans. I came across this because Michael Fumento, who helped calm the panic and debunk the rumors, is back in the news saying coronavirus won’t be a big deal – which makes me worry he’s less a heroic lone voice of reason, and a more a guy who just really likes dismissing diseases.

Best of new Less Wrong: conflict theory vs. mistake theory as different strategies for general non-zero-sum games

The Taiwan Junket: an unimportant assemblyman in a backwater state legislature gets asked to propose a meaningless bill about Taiwan. When the meaningless bill passes because nobody cares enough to vote against it, he gets hailed as a hero in Taiwan and offered a free trip to the country to attend a dinner in his honor. He concludes that this was a Taiwanese government propaganda effort to dazzle citizens who aren’t paying attention to details.

Best of new Less Wrong: Choosing The Zero Point. If you frame vegetarianism as a moral obligation you’re shameful for failing at, people get angry and won’t do it. If you frame it as a surprising new opportunity to do more good than you expected to be able to do before, people get excited and are more interested. What’s the general case of this?

It’s always been true that for what the state spends on public-schooling your kid, you could hire fancy private tutors with tiny class sizes (for example, in New York you could buy a $100K/year tutor to teach five kids full-time). @webdevmason points out that this is even more relevant now that reopening public schools could help spread a deadly pandemic.

Silly rules improve the capacity of agents to learn stable enforcement and compliance behaviors claims that arbitrary rules (like eg Jewish ritual law) play a useful role by spreading information about which of your neighbors comply with taboos and how often violations get punished. They present a toy model that shows that rules against eating poisonous berries work better when coupled with an arbitrary pointless rule whose violation has no real consequences.

Many people including me have enjoyed reading comments by no_bear_so_low (aka De Pony Sum) on the SSC subreddit. Now he’s released an online collection of his essays.

Average national IQ correlates well with GDP per capita and other measures of development. But is average national IQ really the right number to look at? “Smart fraction theory” suggests we should instead look at the range of top IQs, since the smartest people are most likely to drive national growth by inventing things or starting businesses or governing well. Now Heiner Rindermann and James Thompson (names you may recognize!) have given the hypothesis its most complete test so far, and found that yes, IQ at the 95th percentile correlates better with national development than at the 50th percentile. But I am a little skeptical of their results, because 95th-%-IQ correlates at about 0.97 with 50th-%-IQ, so any signal from the difference would be very tiny and probably swamped by other features. Also, usually when 50th and 95th are really different, it’s because the country is multiracial (eg South Africa had the highest 50th-to-95th-percentile IQ difference in the whole sample) and those countries’ policies and economies depend a lot on unique features of exactly what’s going on racially. Some more commentary here.

Related: the above study shows Kazakhstan as having among the highest IQs in the world. This was surprising enough to me that I looked it up, and although they are middle-of-the-road by most measures, one study found they have an extraordinary number of super-high-achievers on standardized tests, beating out usual titans like Finland, Switzerland, Israel, and the US. They could be using the Chinese strategy (only let your smartest students take the test in order to look good). But also, the USSR also stuck its space program and several other science megaprojects in Kazakhstan, and a lot of the scientists stayed around after the USSR broke up, so maybe there are a lot of really bright Russian kids. Also, Stalin deported a lot of Koreans there for reasons that probably made sense to him at the time, and some of them are still around too. Also, they’ve resisted Western pressure to stop having a gifted program in their education system, which probably helps a lot. So who knows, maybe the numbers are right?

Stephen Wolfram: Finally We May Have A Path To The Fundamental Theory Of Physics, And It’s Beautiful vs. r/SSC commenters: Oh God, It’s Stephen Wolfram Talking About Cellular Automata Again. See also this Scientific American article, which leans towards the second position.

Best of new Less Wrong: Discontinuous Progress In History: An Update. The first nuke was thousands of times more powerful than any preceding bomb; a graph of bomb progress would have looked like a gently sloping line that suddenly shot up a cliff in 1945. How common is this pattern? Is progress along some metric (like bomb power, or ship speed, or…) usually linear, linear with occasional cliffs, or totally random? Katja Grace investigates. Be sure to check out the section on the Shipularity, where ship sizes briefly increased so quickly that a naive best-fit would have reached infinity in 1860, produced a single utterly huge ship (SS Great Eastern) in 1858, then crashed back down again and resumed growing normally. It concludes that trends are usually continuous except in certain unusual situations, such as “when Isambard Kingdom Brunel is somehow involved”. Of obvious relevance to AI singularities, since we’re wondering whether AI capabilities will grow at some constant rate or suddenly shoot up – someone should make sure no Brunel descendant gets a job at DeepMind.

Against exaggerated criticism of Dr. Seuss – no, he didn’t cheat on his wife when she had cancer, and he (probably) didn’t (exactly) drive her to suicide.

“The common assessment is that Cuba’s achievements in lowering infant mortality and increasing longevity are among the praiseworthy outcomes of the regime—a viewpoint reinforced by studies published in US medical journals…we argue that some of the praise is unjustified. Although Cuban health statistics appear strong, they overstate the achievements because of data manipulation.” And “data manipulation” includes things like “sometimes they force abortions on unconsenting mothers because the fetus looks sickly and if it dies after birth it will ruin their infant mortality stats”. See also discussion here. Cf. eg New York Times‘s treatment of the same topic – “Cuba has the Medicare For All that many Americans dream about…we should push for American babies born in low-income families to have the same to have the same opportunities for attentive health care as [Cuban babies] will have.” [EDIT: see comments here]

Stephan Guyenet on peer review: “I see a lot of harsh criticism of the scientific peer review process, and I think some of it is deserved. But as someone who has been conducting deep evaluations of published literature outside the peer review process, I’d like to offer some perspective…” When he reviews popular science books, he finds that they’re often egregiously wrong about basic facts and nobody has noticed, so he thinks peer review is necessary to enforce a minimum standard of honesty.

Reason presents a theory of how US health care got this way, and discusses the “doctor’s cooperatives” that were handling the job pretty well before the AMA regulated them out of existence.

Related: the New York Times asks health economics experts to rank different countries’ health care systems in a bracket-style tournament. Some big surprises – 4 out of 5 experts say the US’ system is better than Singapore’s! Switzerland is the overall winner.

According to polls, trust in experts (of all types) has increased significantly over the past twenty years. But experts keep telling me it’s been going down. Guess I’ll stop trusting them! But that means trust in experts is going down, which means I should have trusted them after all. Aaah, there’s no way out! Help! [EDIT: Only true for the UK]

New paper asks MTurk users and Intro Psych students to take a survey, and includes some text in the middle of a question meant to test whether they are really paying attention. It finds that 22% of Turkers (and 64% of students) weren’t reading the question before answering. Author: “I think MTurk workers are better respondents. All of the other evidence I’ve seen suggests that, too.”

Speaking of surveys, here’s the results from r/TheMotte’s. I am obviously interested in A Ranking Of Everything, From Scott Alexander To Stalin. Yes, I am the top-ranked person on the list and beat eg George Washington. You could sort of explain that by r/themotte being a spinoff from this blog and so heavily selected for people who like me. But note that other top ten responses are things like “I believe capitalism is a better economic system than all available alternatives” and various other controversial things – users are ranking controversial things they agree with higher than obviously good stuff (“Andrew Yang” is above “religious freedom”; “I believe a baker should be allowed to refuse to bake a cake for a gay wedding” is above “Abraham Lincoln”). Check out also the writeup of The Modal Motte User if you want to feel uncomfortably seen.

Related: r/TheMotte survey results on the MBTI test

Reporters Without Borders has built The Uncensored Library on Minecraft, containing all the information banned by the repressive governments of the world. The idea is that repressive-state-citizens behind firewalls that prevent them from accessing traditional websites with banned terms can still play Minecraft and get the information that way. Oh, and the architecture is beautiful, exactly what you’d want for a utopian worldwide library dedicated to freedom of thought built in a virtual world with no resource constraints. If we told 1990 we had something like this, they’d think the future had turned out okay after all.

Lesbians (inhabitants of the island of Lesbos) are suing lesbians (homosexual women) in Greek court for “appropriating their national identity”.

Some discussion (based on this and this study) demonstrating that rich Democratic donors do not drag their party toward the right on economic issues.

YouTube admits boosting mainstream media channels over individuals’ videos even though users watch less of them and it makes them lose money.

LA Times on the affordable housing crisis in California. The government regulates affordable housing so heavily that it’s more expensive to build affordable housing for poor people than luxury houses for rich people, and so affordable houses mostly just never get built.

Related: a story about how Mitt Romney solved a homeless shelter budget crisis in Massachusetts. The old policy was that homeless people would stay in a homeless shelter until it was full, and then anyone else who needed shelter would get put up in a hotel at government expense. But lots of people were getting put up at hotels at government expense and it was really expensive. Romney’s new policy was that if a shelter was full and a new person showed up, the longest-time resident of the shelter could go to the hotel, and the new person would stay at the homeless shelter. The rate of new people wanting to stay at full homeless shelters plummeted.

A newly-discovered microbe can “completely stop mosquitoes from being infected by malaria”, scientists already investigating whether it can be used to help eradicate the disease.

In the 1970s, the Japanese auto industry produced noticeably better cars than the US auto industry at lower prices. How did they manage it, and how come it took the US decades to catch up? This review is the best I’ve found on the subject, but although it has lots of great history I still don’t have a good feeling for exactly what the Japanese advantage was and why it was so hard for Americans to do even when the Japanese basically handed their US competitors all their procedures on a silver platter.

DrugsAnd.Me is an exceptionally good site on recreational drugs, when they are vs. aren’t dangerous, and how to use them safely if you decide to go that route. I find myself learning new things even on drugs I am supposedly competent to prescribe to people. Did you know some people think Asians might metabolize benzodiazepines unusually slowly and should start with half the usual dose?

The Scunthorpe Problem is when innocuous words trigger censorship or spam filters because they contain a suspect string – for example, the English town of Scunthorpe sometimes gets censored because it contains “cunt” (also the English town of Lightwater, because it contains “twat”). Some of these are really funny – apparently it’s hard to discuss socialism on some sites because the substring “cialis” makes the filter think it’s an erectile dysfunction ad.

John Alexander Dowie, legendary charlatan faith healer and messianic prophet, got several thousand of his followers to move to his new commune of Zion, Illinois, a town described as “”a carefully-devised large-scale platform for securities fraud”. He is most famous for getting into a Messiah-duel with contemporary Muslim Messiah-claimant Mirza Ghulam Ahmad, founder of the still-extant Ahmadiyya movement. Ahmad challenged Dowie to a contest where they would each pray to outlive the other; whoever survived longest was the real Messiah. Dowie died a year before Ahmad, so I guess the Muslims win this one. Also, check out his outfit. Stylish!

This entry was posted in Uncategorized and tagged . Bookmark the permalink.

611 Responses to Links 5/20

  1. VoiceOfTheVoid says:

    After searching all sites, the police concluded that earlier estimates of nearly a thousand dead had been exaggerated, and that the final death toll had settled at 924.

    “Ah, rumors of a thousand people dead were totally overblown. Really, it was barely over 900!”

    • crh says:

      That line used to end, “… the final death toll had settled at 778.” At some point someone changed it to 924 without changing the rest of the sentence. The two citations are to NYT articles, one from April 2000, one from July 2000.

      From the April article:

      Die she most probably did, along with what authorities here say at last count is some 923 other people, more deaths than in any cult in modern history. With mass graves turning up nearly daily, the authorities fear that many more dead may yet be found, perhaps even hundreds more.

      From the July article:

      The final death toll in the cult killings has settled at 778, he added.

      Earlier estimates given by the government had suggested the figure might exceed 1,000.

      I don’t know why someone decided to switch to an older estimate of the death count.

  2. Erusian says:

    The common assessment is that Cuba’s achievements in lowering infant mortality and increasing longevity are among the praiseworthy outcomes of the regime—a viewpoint reinforced by studies published in US medical journals…we argue that some of the praise is unjustified. Although Cuban health statistics appear strong, they overstate the achievements because of data manipulation.”

    The other thing that gets forgotten about Cuba (a lot) was that it’s medical system was already really good before the Cuban Revolution. By virtually all accounts, the system has been slowly degrading. Whether you blame Communism or the US blockade or both depends on whether you lean pro-Cuba, pro-US, or whatever else.

    Some discussion (based on this and this study) demonstrating that rich Democratic donors do not drag their party toward the right on economic issues.

    The most interesting thing here is that the Republican Party donor’s big appeal to their base here is basically a combination of cultural conservatism and nationalism. And the Democrat’s biggest disconnect is on cultural issues and globalism. This implies political entrepreneurs have a lot of space in the anti-globalist, unashamedly nationalist space, regardless of whether they’re otherwise right or left wing. That won’t only get them a huge portion of their own party, whatever it may be, but also tempt a lot of defections. Of course, the entrenched elite might resist but so it ever is.

    But that makes sense how you have been getting (for about the past ten years) all these weird scenarios where something happens and you end up with populists on both wings. There had to be some political resource driving that, in this case it’s that that group is underserved. My thought is that if you want to increase system stability you need some way to integrate them, which come to think of it is something Biden’s been trying to do while still paying his dues and getting donor approval etc.

    And, as with most political research, it implies the Libertarians are doomed.

    • Matthias says:

      The most curious fact about politics is how (relatively) libertarian policies are despite the broad electorate mostly preferring something like moderate nationaliat socialism.

      Eg the British public hates, hates, hates the privatisation of passenger rail transport. Yet, it hasn’t been reversed under neither Labour nor Tory governments, and it doesn’t seem to be much of an issue in campaigns either.

      • I’d normally explain it as being due to elites, by various definitions, being more liberatrian than average and political parties needing a mix of broad based and elite appeal.

      • NostalgiaForInfinity says:

        Everyone hates the privatised train system but nobody votes for the government based on transport issues

        • SuiJuris says:

          Everyone hates the current system but nobody would really prefer any available alternative.

          • NostalgiaForInfinity says:

            People definitely say they would prefer nationalisation – whether they actually would prefer whatever nationalised system we wound up with is the issue. It has majority support among all voters and is the most popular even among Tory voters). I’m too young to remember British Rail but people will occasionally mention its varied failings as a reason not to re-nationalise. That said, this poll suggests that support for nationalisation is slightly higher among older groups, who are presumably more likely to have experienced both services:

            http://www.bmgresearch.co.uk/the-independent-bmg-poll-widespread-support-for-renationalisation-of-railways-amidst-continued-disruption-to-services/

          • Simon_Jester says:

            Everyone hates the current system but nobody would really prefer any available alternative.

            The word ‘available’ does a lot of heavy lifting there. See Charlie Stross’s remarks on the beige dictatorship.

            Representative democracies, Stross argues, share a few common traits that have the potential to become failure modes:

            1) In practice, getting elected usually requires party backing, and attempts to found your own party with booze and hookers usually fail. This restricts the limits of the possible in a lot of ways.

            For example, you can’t get elected as a conservative in the United States unless you can figure out how to sell your positions on Fox News and to the median Fox News watcher. Because “get elected as a conservative” means, in practice, “get elected as a Republican, using the pre-existing media infrastructure.”

            2) Professionals usually defeat amateurs in any given field, and politics is not an exception. Once in a while, you get a freak upset. Sometimes, Cliff Young exploits his own seemingly bizarre mutant traits and weird running style to beat all the ‘professional’ ultra-marathoners by running a jillion hours with no sleep and somehow not collapsing. Sometimes, Donald Trump exploits his own seemingly bizarre mutant traits and weird style to beat all the ‘professional’ Republican presidential primary candidates by being a superstimulus to a large slice of the primary voters.

            But the general dynamic here still results in your democracy getting a political class that is far more homogeneous than society as a whole, and that will often simply not consider options that most of the populace at large is interested in.

          • Peffern says:

            Am I the only who finds the “beige dictatorship” significantly better than the alternative?

          • fibio says:

            Am I the only who finds the “beige dictatorship” significantly better than the alternative?

            Our experiments with Red Dictatorships certainly didn’t go any better.

          • Thomas Jorgensen says:

            … I am going to make a prediction here. The brits privatized the electric grid, which.. did not work. That experiment ended with EDF running the electric grid in the UK.

            The brits have privatized the rail grid, which.. also does not work, for much the same reason (quit pretending you can have markets in spaces that are natural monopolies by virtue of the physical infrastructure!). Thus, the logical endpoint is SNCF taking over the whole thing, kit and kabootle.

          • John Schilling says:

            The brits have privatized the rail grid, which.. also does not work, for much the same reason

            But the United States rail grid has been privatized from the start and remains so. And it works very well, delivering more freight faster better farther and cheaper than I think every railroad in Europe combined. Is there some reason particular to the UK, or to Europe generally, that says private railroads can’t work well there but can work well here? Is it only passenger rail that can’t be privatized, for example?

          • Austin says:

            Passenger rail struggles with privatization largely because it’s not competitive with automobiles, planes, and buses for convenience, cost, and speed in the vast majority of locations. It may be a monopoly in the sense of there is only so much rail infrastructure to go around, but it is only a monopoly if you don’t consider all “methods to go from point a to point b”.

            The British rail system has been the most successful in terms of not requiring quite so many taxpayer dollars as other European systems and also having a growing ridership.

            My opinion is based on “Romance of the Rails” by Randal O’Toole, which is a more in depth exploration. He is a Senior Fellow with the Cato Institute, so YMMV.

          • alext says:

            Passenger rail struggles with privatization largely because it’s not competitive with automobiles, planes, and buses for convenience, cost, and speed

            Not sure about cost, if you factor in the cost of the car and the driver’s time and effort. That’s, of course, compared to the cost of the train ticket for the end user. Granted it’s subsidised – public inter-city transport is a public service and thus (imo) worth subsidising.

            Speed isn’t really a contest, trains outrun cars and don’t get stuck in traffic. Comfort is better on any decent train. For distances under say 1000 km, rail beats air at total travel time, if you factor in the time and hassle to get to the airport, pass security, wait for embarkation etc., whereas train stations are usually inside cities and easily reachable with public transport.

            Obviously, I’m from lazy Europe, where car ownership is a necessary evil, not a prerequisite.

            But it’s only a contest if there’s a train going where you want, when you need it. That’s the rail service’s job to provide, and that’s where it usually fails. In my experience, at least.

          • Edward Scizorhands says:

            America has chosen to build a lot of rail for cargo, which requires different optimizations than for people.

            There are few places in America where high-speed rail is in the sweet spot where it’s farther than it’s worth driving [1] and shorter than it’s worth flying. [2] And those places tend to already have it, like the NE corridor.

            [1] When you own a car and need a car in your destination city anyway, you need to get over a significant burden to make it worth driving to a train station, taking the train, and then renting a car.

            [2] A lot of the things that make airplane travel suck are policy choices. And if we are making policy choices to improve things, maybe we can make airports suck less. Also, once policy choices are on the table, someone might decide to make trains suck. I do not think getting a TSA for trains is a good idea, but if someone really starts pushing the idiots in charge of making everything suck might make trains suck, too.

          • alext says:

            What’s different when building cargo rail, rather than people rail? Of course industrial tracks branch into each factory/port/whatever, but most of the track (ie investment) is between cities, right?

            When you own a car and need a car in your destination city anyway…

            If the destination has bad public transport, sure, you have to bring your own. But that’s working around a problem, not the optimal state.

            A lot of the things that make airplane travel suck are policy choices.

            If anything, they are the obvious choices, ie doing it any other way would be dumb suboptimal.

          • Thomas Jorgensen says:

            Cargo rail has to tolerate higher weights, while high speed rail has surprisingly tight tolerances. Neither of which is really an insurmountable problem, you can build rail which is dual purpose, just a matter of solid foundations accurately placed. The problem is that highspeed rail trains go at 300+ km/h while a freight train hitting 100 km/h is considered to be hauling ass.

            This plays merry havoc with scheduling if you try to run both on the same tracks, and trying to get around it by, for example, only running freight at night, makes the freight part very, very slow.
            The only actual solution is separate track – which can go on the same right of way, mostly, but putting down four tracks does rather raise costs.

          • Winja says:

            Everyone hates the current system, but they won’t abandon it unless there’s a very good chance that their policies are the ones to get enacted.

          • Edward Scizorhands says:

            If anything, they are the obvious choices, ie doing it any other way would be dumb suboptimal.

            Airplane security is stupid to the point of being user-hostile.

          • alext says:

            highspeed rail trains go at 300+ km/h […] This plays merry havoc with scheduling

            Don’t think high-speed trains can even run on most freight tracks.

            High-speed trains are airplane competitors. They require stronger tracks with a smooth, straight layout etc. That’s a whole different ball game, and it’s debatable whether it’s worth the cost.

            100-150 km/h is plenty fine for most short-medium distances. Scheduling problems are alleviated, as you note, by laying down more tracks. No idea how expensive that is, compared to widening highways, in terms of traffic capacity per dollar.

            Airplane security is stupid to the point of being user-hostile.

            Of course it is. When’s the last time terrorists hijacked a train? Airplanes are such a high amount of energy in such a flimsy package – they need all the security they can get.

          • Aapje says:

            Most people don’t travel long distance to work or for common other trips. For medium distances, trains work best in regions with multiple bigger urban centers, not too far apart. In the US, that would for example be California and the Boston/New York/Washington region.

            They usually need a last mile solution, like buses or bikes. The Dutch railways very smartly started renting out bikes, which has good symbiosis with their trains. Of course, nowadays you have all kinds of bike & scooter sharing companies that litter cities. Smart city planners should incorporate parking for those in their plans for new and refurbished railway stations.

            Long distance, high speed trains have a hard time competing with planes, because they need lots of expensive rail that adds more and more expensive the further you travel. In contrast, every extra mile that a planes flies is relatively cheap.

            Planes work better than high speed rail when people want to travel to many different far away places, in different directions. The cost of JFK airport is amortized over incoming and outgoing traffic to and from many different airports. Long distance, high speed trains work best when traffic comes and goes in one direction. For example, Japan has a very strong north/south orientation, with many big cities along a north/south line, which is why bullet trains work so well there.

            Of course, you could use a hub and spokes model for rail, just like some airlines use as well. Or you can have a hybrid system. This is more or less already used a bit, as you can get a KLM ticket to go from Antwerp to elsewhere, where you first travel to Schiphol by high speed train and then by plane.

          • Edward Scizorhands says:

            Airplane security is stupid to the point of being user-hostile.

            Of course it is.

            You say you agree with this, but then seem to want even more stupid policies.

            Is this about improving trains or making the alternatives to trains worse?

        • noyann says:

          > But the United States rail grid has been privatized from the start and remains so. And it works very well, delivering more freight faster better farther and cheaper than I think every railroad in Europe combined.

          How does passenger transport compare?

          • John Schilling says:

            The United States transports passengers primarily by road and air, and I believe we do so far better than the Europeans who daftly imagine rails are the best tool for that job. It is because we mostly keep passengers off our rails, that they do such an excellent job of hauling our freight.

            For the few Americans daft enough to want to travel personally by rail, we do have a modest subsidized passenger rail system in the form of Amtrak. The Acela is fairly well regarded; the rest suffers quite a bit from limited use and the requirement to not get in the way of the freight.

          • Austin says:

            The numbers I have seen are ~5000 ton miles of freight per capita vs 500, and 62 passenger miles per capita vs 492 between USA and EU-27.

            That works out to about 6x as much freight by rail total for the US compared to EU-27.

            Moving freight by rail is more energy efficient than trucks, much more so the passengers by rail vs. passengers by car/air, so trading freight capacity for passenger capacity is not environmentally friendly from an energy budget standpoint.

          • edmundgennings says:

            The US and Europe have hugely different geographies and styles of urbanization. New York(state) is larger than England. This influences what style of long distance transit makes sense.
            I think there is basically one line where long distance passenger rail makes sense in the US. Going north south on the east coast. My sense is Amtrak uses this to subsidize the rest of long distance passenger rail in the US.

          • Paul Zrimsek says:

            Geography is destiny for both types of rail transport. Passenger rail thrives on population density: the great majority of passenger miles in the US (32.2b out of 38.6b in 2018) are urban rapid transit or commuter rail. For freight, on the other hand, what you want is a long enough haul for the fuel savings to outweigh the expense of changing modes at either end (or of picking up and delivering the cars for the dwindling number of directly rail-served industries). I believe the rule of thumb in US railroading is still a 400-mile haul for general merchandise, though there are some especially bulky or hazardous commodities for which rail is preferred no matter how short the trip. So the opposing patterns between North America and Europe are pretty much what you’d expect to see.

          • Thomas Jorgensen says:

            The EU, much like China, builds rail because road and air just do not bloody well have the capacity. For funsies, google earth Amsterdam and find the rail lines. Those are all more or less at capacity.

            The overall system is, however, very poorly integrated, because there was no Union when most of those lines were built, and in some cases, people actively went out of their way to prevent integration for military reasons.

            So a lot of stuff that should be on rail is going by road. Not, however nearly as much as you would think from the poor rail numbers – Europe has a lot of coastline, and a lot of cities with ports, as a consequence 40 percent of all interstate trade in the union is sea borne, which is way, way higher than the US, if I am reading the us.gov site right. Which I am not sure I am. Is there really that little commerce up and down the western and eastern seaboards?

          • John Schilling says:

            Moving freight by rail is more energy efficient than trucks, much more so the passengers by rail vs. passengers by car/air, so trading freight capacity for passenger capacity is not environmentally friendly from an energy budget standpoint.

            This, plus freight is much more amenable to requiring an overnight trip to getting to the destination city and then sitting around for a day at the city’s central railyard waiting for last-mile transportation. For mid-range passenger transportation, cars get a big boost from not having to do a last-mile mode shift at each end, and for long-distance travel it’s hard to beat 500 mph airplanes. Both of these are more important in the United States, with its low-density cities and broad swaths of (literal) flyover country.

            You’re definitely right that on pure energy-efficiency grounds, you want to optimize for putting the heavy freight on the most efficient system and getting passengers out of its way even if it means they use less efficient vehicles.

          • bean says:

            Is there really that little commerce up and down the western and eastern seaboards?

            Quite probably. The US has a rather antiquated set of regulations on this kind of stuff, most notably the Jones Act. Any ship trading between US ports needs to be US built and US crewed. Even if European countries have similar rules, they can ignore them in international trade.

          • CatCube says:

            @Thomas Jorgensen

            I can’t for the life of me figure out what site you’re referring to with “us.gov” to look at the statistics, but be sure that you’re looking at the right numbers. They may be breaking out some stats differently than others depending on how they count trips from one to another. The Intracoastal Waterway as a system may have different stats than some port-to-port trips that use the sea rather than the system of canals that travel the coast.

            Overall, though, I’d say that if you’re surprised by how little point-to-point water shipping occurs, you’re probably correct. The Jones Act mandates that only US-built and -flagged vessels can carry commerce between ports in the US. Since it’s really expensive to build ships and crew them in the US (because we insist on much more stringent regulations than, say, Liberia, a common flag of convenience) this is a big bottleneck in the use of seaborne commerce.

            Transloading costs eat you alive in all transport modes, so it really only gets used for bulk shipping where it’s already cheap to transload the cargo. There’s quite a bit of barge traffic carrying commodities like coal, grain, limestone, etc., in the US due to this, but I don’t know if I’ve ever heard of a container ship plying the intra-US trade within the continental US (CONUS), though I’m sure some happens incidental to the container trade that occurs with Hawaii and Puerto Rico. At this point, though, you have to contend with railroads and trucks for CONUS transport of containers, and they’ve got that nailed down to the point I doubt you’d make money on it even without the Jones Act.

            Edit: Ninja’d by bean

          • bean says:

            I don’t know if I’ve ever heard of a container ship plying the intra-US trade within the continental US (CONUS)

            Actually, the first container ship was intra-US, between New Jersey and Houston. But that was a long time ago, and when the railroads and trucks figured out how to ship containers efficiently, US coastal shipping went away.

      • Lambert says:

        I thought Corbyn said he’d nationalise.

        And the current Gov’t seems like it’s willing to replace private train operators with the operator of last resort. The DfT kicked Arriva off Northern and took the reins themselves in March.
        I’m guessing it’s partly because Boris wants to cement the support of the traditionally Labour North and partly pour encourager les autres (i.e. Southern).

        Brits who were old enough to be travelling by train before ’93:
        What was it like before privatisation? Was it better under the Railways Board?

        • spkaca says:

          I do recall British Rail – the time before privatisation. I have a bias as I got to travel for free (family member of staff), and I was still young enough that any rail journey was something of an adventure. That said I think the overall experience has not changed very much, taking into account the big differences that would have happened anyway. For instance the first difference that comes to mind is that trains are all non-smoking now, so they smell better on the whole. Not being able to open windows also annoys. But that I take to be because the lawyers made everyone terrified of lawsuits. On the most important matters, delays and cancellations, I recall more since the ’90s; but then I’ve used them a lot more, so that doesn’t reflect actual prevalence.
          I suspect the reason the overall experience hasn’t changed is that the people actually running the trains didn’t change all that much; organisational cultures perpetuate themselves.
          It still annoys me that they call us ‘customers’ instead of ‘passengers’. I detect the hand of a management consultant there.
          Having said all this, I am one of those Conservative voters who lukewarmly supports denationalisation, but as someone notes above, it doesn’t determine my vote.

          • spkaca says:

            *renationalisation. Autocorrect reversed my meaning.

          • AlphaGamma says:

            Not being able to open windows also annoys. But that I take to be because the lawyers made everyone terrified of lawsuits.

            In the sense of ”not being able to open windows enough to stick your head out and be decapitated by a tree or a tunnel entrance”, perhaps. But then, there are a few windows like that still around– largely because on the older high-speed trains, passengers have to open the window to reach the outside door handle to open the door when the train stops.

            I remember a lot more trains with smaller opening windows that it would be very difficult to reach out of (and impossible to stick your head out of). I think those have been removed because you don’t need them on an air conditioned train…

        • Dead Hour Canoe says:

          It was terrible. Everything was filthy and grimy, trains were late, there was a pervasive air of neglect and decline.

          From memory, the thing that improved after privatisation (and it’s only ever been quasi-privatisation, the whole system is still controlled and closely monitored by the state) was the general atmosphere. Trains were still late, the system still didn’t work, it was still in its fundamentals somewhere between terrible and just about OK; but surfaces were painted or covered in posters, stations and trains were less dirty, station shops were better, you got the feeling someone cared enough to at least put on a facade.

          It all seemed fairly pathetic to me at the time, because nothing had really changed; the cheap visual improvements that were made were of poor quality, and seemed like putting sticking plasters on a gangrenous wound. But maybe that kind of stuff matters more than you’d think, because the startling thing about the current system is how much more popular it is than the old one. Passenger numbers have gone from c.750 million in 1995, (having decreased from c.1,000 million when the service was nationalised after the Second World War), to c.1,750 million now, after 25 years of re-privatisation. That revealed preference is, to put it mildly, not what you’d expect given people’s views on privatisation.

          I don’t think you can read too much into this, however. You have to bear in mind that everything used to be just generally crappy in the UK – not awful, but shabby, dreary, slightly communist Eastern Europe. The rail system has changed in roughly the way nearly everything else has (for better and worse).

        • dark orchid says:

          A book-length answer to this is Christian Wolmar’s “broken rails”. Very brief review from memory: British Rail was created after the second world war and started out on an austerity budget like everything else – dirty and delapidated. But Wolmar claims they had their act together to the point that, when they were privatised, everyone in government expected the new owners to be 10x as efficient or something but it turned out that BR in its last days was actually quite a well-run company.

          Privatisation led to some unforseen consequences. Everyone wondered why the unions didn’t seem to mind too much until it turned out, they’d figured out that ToCs (train operating companies) would be competing for trained drivers which pushed wages up. Railtrack, the infrastructure company, ended up running as a property investment company “with a bit of railway attached” which they neglected to a point that several major disasters occurred, the one at Hatfield was the final straw and sent them into administration. (There’s a case to be made that the country is still dealing with the after-effects of this period of under-maintenance.) A lot of engineering talent left for various reasons, and the new managers were “manager types” not “railway types” who didn’t always have a clue about things.

          Wolmar also makes clear that privatisation didn’t mean the government giving up control – far from it, they regulate everything down to the type of seats allowed (this one through crashworthiness regulations). I know there were a lot of complaints when GWR introduced “ironing board” seats on the new class 80x trains, but the fact is the only bit that GWR had any control over was the colour of the seat covers (they went with dark green).

          Another recent example: before Covid-19 the big railway news (HS2 aside) was industrial action over DOO (driver only operation), that is running passenger services without a second safety-critical staff member on board (the “guard”). Although this was a dispute between unions and ToCs, as experts on railforums.co.uk will point out, the requirement to introduce DOO was written into the relevant franchise conditions by the government.

          Before I stay up all night talking about railways, this rail forums thread gives a few descriptions of how BR was different from today. Example: “A generally muckier and more run-down environment.”

          • Aapje says:

            The Dutch railways were merely more or less privatized, with nearly all of the passenger traffic being by the National Railways (NS). They really do a very solid job compared to the few competitors, aside from the high-speed rail debacle.

            One of the main issues is the extra bureaucracy involved with separating out the infrastructure company (literally called Prorail), so some have demanded that they be integrated again, although this runs into the issue that there is quite a bit a competition on the cargo side of things.

      • Null42 says:

        I always wondered how much of it had to do with the obvious term being tarred with association with the regime that started WW2.

        ‘Socialism for citizens’? ‘Patriotic socialism’? I’m kinda stumped. Especially as I suspect a lot of economically-liberal, socially-conservative people aren’t actually fond of the historical Nazis. Cutting immigration and preserving Medicare doesn’t mean you support death camps.

    • Simon_Jester says:

      I would like to point out that for as long as left-right politics have existed in recognizable form in democratic societies, there have always been populists on both wings.

      Right-wing populists build a power base by leveraging traditions that rally core target demographics against perceived lesser-thans and outsiders. Left-wing populists build a power base by leveraging discontent at economic inequality, a universal human trait that can be found in any time and place.

      You find recognizable ancestral forms of this going back a LONG way.

      So the modern existence of both right-wing and left-wing populists doesn’t necessarily mean anything especially unusual is going on.

      • Nikitis says:

        Populists have been indeed historically prevalent. Hell, even the ancient Athenian democracy had populist demagogues frequently swaying the electorate. But there were essentially no successful populists in the western world 20-25 years ago. It is worthwhile to examine why they are becoming more prevalent again.

        • keaswaran says:

          Does Hugo Chavez not count as part of the “western world”?

          • Nikitis says:

            Not really, no. Perhaps I used the wrong description, but I meant “western world” as a shorthand for “Western Europe and North America” and their economically advanced liberal democracies.

          • keaswaran says:

            I suppose that’s reasonable. But even so, it’s notable that populism was important in many of these economically advanced liberal democracies in nearly every decade since 1900, with the ’90s and ’00s being perhaps the only exception. I’m guessing this has something to do with Francis Fukuyama’s “end of history” thesis and the “great moderation” in economics of 1980-2008.

          • Nikitis says:

            Yes, which is why I specified a period of time where that was not the case. I hope we are in agreement on this?

            I do believe the “end of history” philosophy the neoliberal economis are responsible for the lack of populism of that period, but also laid the seeds for the revival we are currently experiencing.

        • Simon_Jester says:

          @Nikitis

          Populists have been indeed historically prevalent. Hell, even the ancient Athenian democracy had populist demagogues frequently swaying the electorate. But there were essentially no successful populists in the western world 20-25 years ago. It is worthwhile to examine why they are becoming more prevalent again.

          What definition are you using for ‘populist?’ The term isn’t always used consistently, and it would be pretty easy to creatively redefine it to avoid including any major national leaders of 20-30 years ago.

          At a bare minimum, I think @keaswaran is right to point out that the ’90s and ’00s, specifically were basically one big triumphal march of neoliberal capitalist-technocratic politics. You had left and right-wing variations on this theme, but it wasn’t getting questioned by much of anybody. Because, again as Keaswaran points out, this was “the Western World” still riding that post-Cold War high, of believing that history was over and we’d won, aside from mopping up a few stragglers.

          • Nikitis says:

            Would you say then that there were successful populists movements or politicians in those countries at that period of time? Because if not, we do not disagree in anything. All we need to ask then is why this period ended in the way it did.

          • Simon_Jester says:

            @Nikitis

            If populist movements have been common for centuries in many times and places, but suddenly they mostly disappeared for a period of time between, say, 1985 and 2000 or so*…

            Perhaps the natural pressures that always create populist movements were temporarily eclipsed by some other phenomenon, blocking out something that would normally be there.

            In which case saying “Why did the populist movements come back?” is asking the wrong question. Maybe the question we should be asking is “What temporary combination of circumstances caused them to mostly go away?” and then the answer to the previous question is simply “That temporary combination ceased to apply.”

            ____________________

            *(just to pull numbers out of a hat)

          • Nikitis says:

            It is still the right question. If the answer is indeed “a temporary set of circumstances”, then the question just shifts to “what were those circumstances”, “why did they end”, and “how can we replicate them”, assuming that populism is something we’d prefer to get rid of.

        • Liam Breathnach says:

          There was indeed one Western country with a populist leader in this period:

          https://en.wikipedia.org/wiki/Silvio_Berlusconi

        • Squirrel of Doom says:

          The one big change is that we’re now getting our news through social media, not the “mainstream media”.

          That is common to the whole world, and explains why populism is happening everywhere.

          • Does it also explain the increased popularity of socialism, at least among young people in the U.S.?

          • Simon_Jester says:

            @DavidFriedman

            It very well might. Official media in the US still tends to be quite reluctant to endorse socialism, even tacitly. The Internet enables socialists to propagate their ideas by grassroots and word of mouth microphone much more easily.

            On the other hand, it does much the same for everyone, so it’s best modeled as a cause for a general rise in views farther from the mainstream, not as a specific campaign to promote one specific ideology.

        • acertainidiot says:

          The reason populists exist in the first place is a distrust in government. The reason for their absence during the time period you suggested is that trust in government was strong following the “defeat” of the Soviet bloc and subsequent economic growth that lasted (with a couple minor interruptions) for 18 years. The political order, in control of the situation, began to greatly limit the framework of debate to further keep things under control, primarily to prevent left-wing thought from re-emerging in the mainstream (of which they were successful). This kept populists from gaining some power. Which is not to say there were attempts at populism: Most significant of these were Ross Perot, Jesse Ventura, and Howard Dean in the US, Jean-Marie Le Pen, Pim Fortuyn, and Jorg Haider in Europe. But they were often suppressed or failed to make a lasting impact.

          The only time that populism ever fully succeeded in this time frame was in Italy. The reason for this had to do with the Tangentopoli (Bribesville) scandal that occurred in the mid-1990s. It was a corruption scandal that engulfed the entire political order in the country that had been in place since the start of the Cold War/end of the monarchy. In this context, the media magnate Silvio Berlusconi took a populist angle and quickly entered government, riding on resentment from the populace over the excess corruption.

      • Erusian says:

        Sure but those flare ups were usually one sided. When you had the original Populist movement you didn’t have a simultaneous conservative version. Further, this isn’t a strategy being used by established politicians but by political entrepreneurs like Trump. So the question is what gap in the political market, what political resource, is driving it. This appears to point to it being a large base of socially conservative but fiscally liberal voters with strong national identity spread throughout both parties and being underserved by both parties.

        • jumpinjacksplash says:

          I suspect that’s it in a nutshell – that weird fourth box at the bottom of the Libertarian diamond charts (generally called “communitarians” or “authoritarians”) is actually full of people.

          I’d sketch out a hypothesis that a successful political party requires both voters (to vote for it) and elites (to run/organise it). Elites are economically right-wing and socially left-wing (this isn’t really a description of Libertarianism – pre-2020 Bloomberg is much closer to socially a Democrat and economically a Republican), whereas non-elites are socially right-wing and economically left-wing. Elites prioritise which of the two halves they care about differently, and sacrifice the other half to appeal to voters.

          The best evidence I can think of for this is the UK, which has a weird third-party (Lib Dems), who had a hugely unbalanced organisation-to-popularity ratio, similar to Bloomberg. On the other side, whenever someone attempts a socially right-wing, economically left-wing party it tends to be a shoddy disorganised farce (post-Farage UKIP, the Trump administration, the various People’s Parties in Europe). It also explains why, when in power, left-wing parties focus on social issues but right-wing ones focus on economic ones, but they each mostly campaign on the other side of their platform (other than 2020 Hilary Clinton, who went full elite).

          If you wanted a figure-ground inversion, you could argue that the “real” political spectrum is elite opinion vs popular opinion, but as neither of these could ever form a majority government you end up with competing coalitions.

          To avoid getting bogged down in biases, I’d suspect that elite opinion is probably self-serving, and populist opinion is probably somewhat uninformed/simplistic (closer to the loopier extremes of American ‘Movement Conservatism’ than a Tucker Carlson ‘welfare state minus the immigrants’ platform).

          Footnote: I don’t think left-wing economics plus right-wing social policy gets you fascism, because Bernie Sanders’ economics combined with Ted Cruz’ social views would get you medicare-for-all and school prayer (or prayer-and-medicare-everywhere to tie into the Dr Seuss link) but in the real world neither of them are in favour of fuhrers, panzers, gas chambers etc.

          Edit: Just occurred to me that on this model, the rise of populism worked in the US because primaries weakened elite control of parties but not elite support for them, and in Europe they have more popularity due to elites screwing up, but haven’t got very far due to tripping over their own feet (cf. Italy).

          A lot of the rise of European populism could just be a political cycle where mainstream parties move to the centre to get votes, leaving space on the extremes for new parties which ultimately replace them (e.g. Northern Ireland, Germany, the left in Greece, possibly the right in France but that’s more complicated). This would be compatible with all of the above being wrong. Data-wise, YouGov seems to be compatible with it for 2019 Britain, but 538 has America being only 20% “hardhat” (anti-libertarian) in 2016.

          • I don’t think left-wing economics plus right-wing social policy gets you fascism, because Bernie Sanders’ economics combined with Ted Cruz’ social views would get you medicare-for-all and school prayer (or prayer-and-medicare-everywhere to tie into the Dr Seuss link) but in the real world neither of them are in favour of fuhrers, panzers, gas chambers etc.

            What do you think “fascism” means? Mussolini, who invented the term, wasn’t in favor of gas chambers and didn’t have a very effective army.

            To me, the defining feature of fascism is the combination of private ownership of the means of production with government control. The industrialist still has his estate and his limousine, but the government tells him a good deal about what he is to do with his factory.

            It started, as I understand it, because Mussolini, then a socialist, concluded that you were never going to get socialism by a bottom up rising of the working class, so you had to get the same result by a top down strategy.

          • Simon_Jester says:

            @DavidFriedman

            Do you think you can get fascism without the racial mystique and intense, usually violent nationalism? Because even if Italy’s army was pretty weak, Mussolini certainly used that military aggressively in pursuit of perceived national objectives.

            Elites are economically right-wing and socially left-wing (this isn’t really a description of Libertarianism…)… whereas non-elites are socially right-wing and economically left-wing.

            I think either you meant to load that up with “ORs” instead of “ANDs,” or the logic structure is off.

            Could you expand on what you mean by, for example, “elites are economically right-wing and socially left-wing?” Do you mean “elites hold and promote both those sets of positions at the same time, reliably?” or what?

            Similarly, saying “non-elites are socially right-wing” likewise leaves a lot to be desired if we’re looking for unambiguously true statements that can be used as evidence, in my opinion.

          • Filareta says:

            An exception: Polish rulig Law and Justice party is basically this right-on-social-issues-left-on-economy populist movement, and they are visibly well organised, because they have formed government three times, the recent two times after winning two parliamentary elections in the row with 50%+ majority in seats (so no need for a coalition partner, actually they are the first non-coalition government since the end of communist era).

          • @SimonJester:

            Do you think you can get fascism without the racial mystique and intense, usually violent nationalism?

            I don’t see why not.

            A number of South and Central American dictators have been described as fascist, although I don’t know a lot about their actual policies. I don’t think that involved any more nationalism than other countries had.

            FDR was an admirer of Mussolini early on, and the First New Deal was along fascist lines, economically speaking, but without a racial mystique or an unusual level of nationalism.

            Elites are economically …

            Was that directed at me, as the preceding point in your comment was? I’m not the person you are quoting there.

          • Null42 says:

            Anatoly Karlin (who says plenty of other stuff I don’t agree with, but is pretty data-based, at least compared to most of the other guys on ‘Israel is behind the coronavirus, 9/11, and the moon landing hoax!’ Unz) had a chart showing a lot of Eastern Europe had a populist-neoliberal axis instead of the usual liberal-conservative one, essentially being rotated 90 degrees on the political compass.

            The theory was Communism was socially conservative in a lot of ways, encouraging family formation and the like, so the rebellion against it tended to be more purely neoliberal/libertarian. I’m curious to hear anyone with real-life experience in that part of the globe comment.

          • alext says:

            The theory was Communism was socially conservative in a lot of ways, encouraging family formation and the like, so the rebellion against it tended to be more purely neoliberal/libertarian. I’m curious to hear anyone with real-life experience in that part of the globe comment.

            Even better. Communism (ie left-ism) was the conservative political option. Lots of people had figured out the system and were making do, they weren’t really all that eager to change all the rules. The free market, individualism, capitalism, that was the new thing, with which the young and hip wanted to replace their elders’ outdated socialism.

          • akarlin says:

            Yes, if you think about it, “conservatism” is ironically the ideology that is most lacking in core values. It’s basically just the “center of gravity” of social and political opinion at any one time and place, and these can and do vary cardinally.

            So the “conservatives” of 1990s Russia were indeed Communists. Since the 2000s, it has transitioned into Putinism.

          • It’s basically just the “center of gravity” of social and political opinion at any one time

            (Conservativism)

            That’s what the word should mean but it isn’t what it does mean. Barry Goldwater was a conservative, but far from what was then the center of gravity of opinion at the time, as reflected in the election outcome.

          • alext says:

            “conservatism” is ironically the ideology that is most lacking in core values

            Not true. It means being cautious about change. Any change causes chaos, unwanted and unpredictable side effects and takes effort to adapt.

            Moderate conservatives allow that sometimes changes to the status quo are necessary, as long as they are carefully considered and don’t cause too much disruption.

            You could as well say progressivism lacks core values – loving newness for newness’ sake.

          • acertainidiot says:

            I don’t see why not.

            A number of South and Central American dictators have been described as fascist, although I don’t know a lot about their actual policies. I don’t think that involved any more nationalism than other countries had.

            A common threads among post-WWII South and Central American dictatorships (with exception to Cuba under Castro and Argentina under Peron) is a particularly virulent form of anti-communism that imposed extreme forms of violence in the name of “defending the nation.” Its roots were in the so-called “Jakarta Method,” named after Suharto’s coup and the subsequent genocide of over 1 million Indonesians for their (supposed) connections to the local communist party. In the sense that they were anti-left wing to the point of indiscriminate violence, there is an argument to make that they were fascist.

            In terms of economics, the situation gets hazier. Chile under Pinochet, for example, served as the testbed for neoliberal/free-market capitalism through its so-called “Chicago Boys” dictating economic policy along those lines. Argentina post-Peron, and to a lesser extent Brazil, followed suit. But there is an equally compelling argument to make that such a form of capitalism can exist under fascism, on the principle that it’s flexible enough to adapt to the situation.

        • Simon_Jester says:

          Sure but those flare ups were usually one sided. When you had the original Populist movement you didn’t have a simultaneous conservative version.

          Says who? There have been examples of mass popular movements of liberals and conservatives colliding since at least the Vendee uprising of the 1790s.

          Because liberal and conservative populists are often motivated by very different things (social unfairness versus threats to their perceived purity as humans), it’s totally possible for the same set of social conditions to give rise to two such imaginary humans.

  3. VoiceOfTheVoid says:

    Re: “Choosing the Zero Point”, I feel like I’m not quite grasping the concept. Can anyone who thinks they do help come up with some more concrete examples of what it means to “set a new zero point”?

    • MawBTS says:

      See Scott’s Setting the Default.

    • keaswaran says:

      Here’s Tversky and Kahneman 1981 on this (interesting choice of example they used, in the current context!):

      The effect of variations in framing is
      illustrated in problems 1 and 2.

      Problem 1 [N = 1521]: Imagine that the U.S. is preparing for the outbreak of an unusual Asian disease, which is expected to kill 600 people. Two alternative programs to combat the disease have been proposed. Assume that the exact scientific estimate of the consequences of the programs are as follows:

      If Program A is adopted, 200 people will be saved. [72 percent]

      If Program B is adopted, there is 113 probability that 600 people will be saved, and 213 probability that no people will be saved. [28 percent]

      Which of the two programs would you favor?

      The majority choice in this problem is risk averse: the prospect of certainly saving 200 lives is more attractive than a risky prospect of equal expected value, that is, a one-in-three chance of saving 600 lives.

      A second group of respondents was given the cover story of problem 1 with a different formulation of the alternative programs, as follows:

      Problem 2 [N = 1551]:

      If Program C is adopted 400 people will die. [22 percent]

      If Program D is adopted there is 113 probability that nobody will die, and 213 probability that 600 people will die. [78 percent]

      Which of the two programs would you favor?

      The majority choice in problem 2 is risk taking: the certain death of 400 people is less acceptable than the two-in three chance that 600 will die. The preferences in problems 1 and 2 illustrate a common pattern: choices involving gains are often risk averse and choices involving losses are often risk taking. However, it is easy to see that the two problems are effectively identical. The only difference between them is that the outcomes are described in problem I by the number of lives saved and in problem 2 by the number of lives lost. The change is accompanied by a pronounced shift from risk aversion to risk taking. We have observed this reversal in several groups of respondents, including university faculty and physicians. Inconsistent responses to problems I and 2 arise from the conjunction of a framing effect with contradictory attitudes toward risks involving gains and losses. We turn now to an analysis of these attitudes.

      • Ketil says:

        If Program B is adopted, there is 113 probability that 600 people will be saved, and 213 probability that no people will be saved. [28 percent]

        This took me a while to decipher. But I think 113 is supposed to be read as 1/3, i.e. 33% chance, and the bracketed percent (i.e. 28 in this case) is …the fraction of people (out of the N=1500 or so) who chose each option?

        • Kindly says:

          Yes, except that we discover, clicking on the PDF, that the actual N values were only 152 and 155. (Still good enough given the difference in percentages.)

      • No One In Particular says:

        Also T&K (and psychologists in general) refer to it as “anchoring”.

  4. Ozy Frantz says:

    Re: Snowflake–

    Remember that the first black Marvel superhero was Black Panther? Comics are just like this. As a person whose favorite age of comics is silver, I’m glad we’re returning to the classic Marvel campiness and tone-deaf yet earnest attempts at social justice.

    • grendelkhan says:

      Remember when Grant Morrison was doing Animal Man (“The Coyote Gospel” holds up wonderfully), and he had B’wana Beast (a white guy with magical Africa powers, obviously problematic even in the eighties) hand off his powers to a black anti-apartheid activist who named himself “Freedom Beast”?

      There’s a place for subtlety and restraint, and that place is generally not in superhero comics.

      • Deiseach says:

        B’wana Beast (a white guy with magical Africa powers, obviously problematic even in the eighties)

        If it’s Grant Morrison, that was deliberate. He’s a guy who really doesn’t like superheroes as traditionally conceived doing superhero comics and it only works because he loves comics themselves as a medium. The above sounds like an obvious Tarzan deconstruction.

        • J Mann says:

          Have you read Morrison’s autobiography? I’d say he loves superheroes, but he’s a high functioning mystic and loves them because they provide a platform for gonzo creativity.

          • Deiseach says:

            Which is why I said “as traditionally conceived” 🙂 I’d place him midway between Alan Moore, who takes traditional characters and puts a completely different twist on them yet still locates them in the body of the mythos around superheroes and comics (his re-imagining of Rupert the Bear in “The League of Gentlemen” is both transgressive and yet follows logically if you go the route of “okay anthropomorphised animals, we’re talking Doctor Moreau here”) and Garth Ennis, who absolutely hates superheroes who are the conventional origin type of supers (but who can be extraordinarily tender towards the ‘little people’/ordinary people/those at the bottom of the heap in society).

        • grendelkhan says:

          B’wana Beast dates back to the late 1960s; the character had become an embarrassment to DC by the 1980s.

    • Anaxagoras says:

      Slightly more subtle than you might think with Black Panther. The superhero actually very slightly predates the Black Panther Party. Consensus seems to be that it’s actually a coincidence, with neither being named after the other.

      This case is definitely naming the characters after the political terms, but yeah, agreed that Marvel has a pretty established history of campy, earnest, and “really?”-inducing approaches to social justice.

      • Le Maistre Chat says:

        This case is definitely naming the characters after the political terms, but yeah, agreed that Marvel has a pretty established history of campy, earnest, and “really?”-inducing approaches to social justice.

        I’ve seen the initial period of this called “the Relevance era” and it included DC Comics as well, though perhaps to a lesser extent. Marvel had Black Panther, Luke Cage, the Falcon and Ms. Marvel (Carol Danvers, who movie fans wouldn’t know under that name), while DC may have only had Green Arrow’s social justice goatee and the John Stewart Green Lantern.

        • No One In Particular says:

          Presumably, by “that name”, you mean “Carol Danvers”. The Endgame script refers to the character as “Carol”, and the name appears in dialogue in both Captain Marvel and Endgame, and Carol’s Kree name (“Vers”) is based on the name. A casual viewer might not be familiar with the name, but anyone who is paying attention, even if they haven’t seen any of the comics, should be familiar with the name.

          • Le Maistre Chat says:

            I meant “Ms. Marvel”. The character Carol Danvers had been floating around the Marvel universe for decades before being given the name “Captain Marvel”, which had previously belonged to Mar-Vell the Kree, Monica Rambeau, Genis-Vell, and I forget who else (his sister Phyla-Vell?). And of course there was Billy Batson before them all.
            The movies skip all the soap opera of revolving door code names. Though maybe that’ll change with a Disney+ show based on the fallout of Steve Rogers irretrievably going away.*

            *Even then, don’t get your hopes up for a list of Captain Americas convoluted enough to include “Steve Rogers of the 1770s” and “evil fake Steve Rogers who fought Communists”.

      • J Mann says:

        Marvel’s approach is often campily earnest regardless, as in the luck-powered Irish national hero “Shamrock” or their Peter Parker wannabe “Speedball.”

        Let’s also never forget DC and the New Guardians.

        Oh my – I hadn’t followed the New Guardians, but just read the wiki page I linked and lost it at this summary:

        Jet – An English girl who could manipulate electromagnetic fields to a variety of effects. She was infected with HIV after being bitten by The Hemogoblin, a vampire with AIDS. After apparently giving her life to fend off an alien Invasion, Jet is now alive again and leading the Global Guardians.

        • Deiseach says:

          as in the luck-powered Irish national hero “Shamrock”

          How did I manage to miss this? And I was kind of wincing about Banshee! 😀

          • J Mann says:

            Take a look at Marvel’s “Perfect St. Patrick’s Day Read” for more on both Shamrock and Cuchulain the Irish Wolfhound and their opportunity to join the Guardians of the Galaxy. 😉

          • theredsheep says:

            It’s also the name of a character in Worm with very similar powers, so apparently Wildbow missed it too.

          • Deiseach says:

            Dear God, when I read your comment I thought it would be an actual wolfhound named Cuchulain but no, they meant Cú Chulainn the Hound of Ulster (whom they present as a kind of big blond Viking?)

            It’s a weird mix of getting obscure details right and getting the larger picture wrong.

          • MisterA says:

            Don’t forget Banshee’s arch nemesis and evil brother, Black Tom Cassidy, whose mutant power is the ability to create a shillelagh.

      • Ozy Frantz says:

        Huh! TIL.

        I still think not renaming him points to a certain… something.

        • Anaxagoras says:

          For a more extreme but recent example of an innocuous element getting associated with a not-widely-liked political movement, I gather that the spy organization in the TV show Archer stopped being called ISIS some time in 2014 after the Islamic State started its reign of terror.

          Personally, even though I think ISIS is in the upper echelon of awful political organizations, I wouldn’t have batted an eye had Archer decided to ignore the unfortunate alignment and kept the name. Black Panther (the superhero) has quite a bit more in common with the Black Panther Party than Archer’s ISIS does with the Islamic State. On the other hand, though, difference in badness of the political group should defray that a lot.

          I dunno. I have no idea what conversations Marvel had out of public eye in the late 60s.

          • John Schilling says:

            Fortunately for Marvel, they get to sit this one out on account of Isis being a superhero of the DC stable. A derivative version is currently appearing on the TV series “Legends of Tomorrow”, with no mention of her former supernym.

          • Matt M says:

            I was always disappointed that Archer caved on that one. They could have gotten a lot of mileage out of jokes about the name confusion, particularly given that their own ISIS organization was hilariously incompetent.

          • LadyJane says:

            @Matt M: In an interview, one of the writers said they toyed with the idea of doing the whole “these assholes stole our name!” thing and building jokes around it, but decided against it because it felt more like South Park humor than Archer humor.

          • silver_swift says:

            There is, apparently, a Marvel character called Isis, though she’s supposed to be the literal Egyptian goddess.

            Also, looking through the list of things that are named Isis, it’s interesting to see which did and did not change their name after the Islamic state stuff started.

            In particular, there is apparently an organization that calls itself the Institute for the Secularisation of Islamic Society, which has got to be confusing the hell out of everyone that interacts with it.

        • LadyJane says:

          I still think not renaming him points to a certain… something.

          The character was renamed to Black Leopard for a little while back in the 70s, specifically because the writers didn’t want him to be associated with the real-life Black Panther Party. Eventually they changed it back, largely because the real Black Panthers stopped being relevant and faded into obscurity.

      • spkaca says:

        Consensus seems to be that it’s actually a coincidence, with neither being named after the other.

        The original Black Panthers were 761st Tank Battalion, a distinguished unit in WW2 (made up primarily of African-American soldiers, the Army was still segregated). Possibly both names have the same source.

    • theredsheep says:

      Note that Snowflake and Safespace are just the two cringiest of a whole lineup, the New New Warriors. Their teammates are (I am not making any of this up) a guy named Screentime whose power is googling stuff unusually quickly, a Goth vampire teenager named B-Negative, and Trailblazer, a seemingly obese girl with a magic hammerspace backback.

      • NostalgiaForInfinity says:

        These are all lame, but I would assume that many from the ’40s/’60s/’80s were similarly cringy. We just only know about the ones who have lasted (who have also changed through the decades to adapt to contemporary taste e.g. 1960s Batman).

      • J Mann says:

        The best is that they describe it as an effort to recreate the magic of edgy 90s teen skateboarding hero “Night Thrasher.”

        • MisterA says:

          My reaction to this is that once I saw it was the New Warriors I am totally onboard for these names because ridiculousness like this is the New Warriors brand – the classic roster featuring Night Thrasher and Speedball.

          • theredsheep says:

            Those two are silly, albeit less so than the new lineup. The rest of the originals played it more or less straight; Firestar was a flying fireball girl, Namorita was a silly Namor ripoff, Nova I guess flew around and punched things really hard. You could chuck in Silhouette, who IIRC was there to be NT’s mildly disabled GF and didn’t seem to do much because she was Kitty Pryde with crutches.

            EDIT: Forgot Marvel Boy existed. It’s been a while. But he was also not deliberately ridiculous.

          • Edward Scizorhands says:

            The old New Warriors were all old reject characters with dubious origins — dubious in publishing terms of “why do we need this?” Firestar came out of the Spider-Man cartoon, Kid Nova was a dumb sidekick created for dumb reasons, Namorita was clearly a “give Namor a super-girl-cousin, just like Superman and The Hulk.”

            Fabian Nicieza decided to just group them all together and go for broke. And it worked wonderfully.

        • Deiseach says:

          edgy 90s teen skateboarding hero “Night Thrasher.”

          I think I remember him? 😀

          Comics have had a lot of embarrassing “be relevant to the kids” moments over the years, but in 2020 we are supposed to be more aware of problematic elements, so I can’t see this new team line-up as anything more than some convoluted inside joke. Whatever way I look at it, I can’t take it seriously.

      • Ninety-Three says:

        The other characters are useful for giving us an idea what the writers are going for. For instance, Screentime got his powers from exposure to “experimental internet gas” and B-Negative says things like “The world is a vampire…and so am I.” This is not earnest seriousness to be cringed at, it’s camp. They’re being deliberately silly and aiming for the audience to laugh with them.

        • theredsheep says:

          See, I thought “experimental internet gas” went beyond campy-stupid and into “co-writer: the late Sen. Ted Stephens.” If you’re pretending to be a really clueless old person desperately trying to be relevant with a youth scene you don’t understand, I don’t think the fact that you were insincere/ironic about it should earn you points. Especially since I suspect the goal was to provoke discussion by being pitch-perfect stupid in a way that’s differently offensive to both left- and right-wingers. I know I haven’t heard anybody mention the New Warriors since about 1995, when I stopped bothering with comics …

          • MisterA says:

            Yeah. I mean I think it’s totally fair to think this attempt at a joke misses the mark, but I do think it’s definitely an attempt at artificially replicating the sort of weird silliness that happened organically with the original New Warriors.

            Which is probably why it rings a bit false, though. Night Thrasher, the grim skateboarding vigilante, is very stupid, but it wasn’t intentionally stupid, it was just the kind of random weirdness that arose in comics back then all the time.

            It’s the difference between The Room and all the attempts that followed to intentionally create movies that are “So bad they’re good.” Part of the fun is that the weird nonsense happened organically.

          • theredsheep says:

            Am mildly concerned that it was supposed to come out April 15 and all I can find is criticism of the announcement. I wonder if they just quietly axed it? That would be a great loss for schadenfreude.

          • MisterA says:

            Nah, comics all stopped coming out when COVID hit. There was actually some kind of huge drama involving Diamond, the distributor that has had a monopoly on comics distribution to comic book stores for decades and which a lot of store owners have always hated, that I don’t super understand but resulted in no new comics coming out for months.

        • I thought it was a joke at first too, but the writer, Dan Kibblesmith at least seems pretty sincere about it. He says he found previous superheroes to be alienating and that he wanted to make characters with a positive social message for young people. Obviously the names are puns and so on and a certain degree of cheesy humor goes with the territory, but I don’t think the characters were entirely conceived as something to laugh at in total. It’s certainly not being presented as a parody by the author or the marketing, though it’s possible that someone higher up who allowed this to happen is in on the joke.

      • Winja says:

        The backlash against the New New Warriors has been entertaining to behold.

        Not only are they disliked by the typical commentators at the center, right, and hard right political positions, but they’re evidently fairly reviled amongst the very market of young, hip, trendy left-leaning SJW types that the characters are supposed to appeal to due to a perception that they’re clearly meant to be patronizing.

    • The Pachyderminator says:

      Unfortunately, the superhero Black Panther slightly predates the Black Panther Party, and Stan Lee has claimed that the shared name is just a coincidence.

  5. MawBTS says:

    Related: the above study shows Kazakhstan as having among the highest IQs in the world.

    That’s surprising to me too. In this 1973 graph on Soviet scientists by ethnicity, Kazakhs don’t stand out. (Before clicking the link, see if you can guess who’s at the top…and the bottom).

    • John Schilling says:

      But Kazakhstan and ethnic Kazakhs, are not the same thing. Scott’s theory, at least superficially plausible, is that the nation of Kazakhstan has an anomalously high IQ because that’s where the former Soviet Union stashed a big chunk of its space program, its biological-weapons program, and assorted other chunks of science best done in the middle of nowhere, with a bunch of smart but not necessarily ethnic Kazakh scientists to run them.

      • Steve Sailer says:

        New Mexico has a high proportion of very high IQ residents relative to its median IQ due to Kazakhstan-like WW2-Cold War facilities.

      • Joseftstadter says:

        There has been a huge exodus of other ethnic groups out of Kazakhstan since 1991. The Jews, Poles and Germans left almost immediately. The Russians have been steadily moving out for decades. The Korean population is not large enough to move the needle. In 1995 that explanation might have been plausible but not in 2020.

        Having lived in Kazakhstan for several years, and having lived in other parts of the former Soviet Union I find it very hard to believe that ethnic Kazakhs have anomalously high IQs. I honestly would have guessed the opposite.

        However, I wonder though how much of the seemingly low IQ behavior I saw in places like Kyzyl Orda, Karaganda or Atyrau back in the 1990s was actually a result of environmental contamination artificially depressing IQ? Maybe the collapse of a lot of the old line Soviet heavy industry has resulted in a much healthier younger generation.

        • Long Disc says:

          I do not think the space program had much of lasting impact on population. Baiconur was just a launchpad without much permanent non-military presence around.

          However there were some outstanding school and university professors in Kazakhstan for many years and some of their legacy persists despite the mass emigration of non-kazakhs after 1990. The reason for this brilliance of educators was that Kazakhstan was regime favourite dumping ground for exiling all kinds of unwanted groups in 1930s-1950s. Some of these groups were relatively uneducated (such as Koreans or Chechens), others were relatively advanced (such as Germans exiled from their settlements along Volga river in 1941 and also Poles exiled in 1939-1940 from what is now Western Ukraine and Belarus), and some were quite elite (such as the victims of mass political and class based purges in St Petersburg and Moscow.) In many cases, a school teaching job was the best a former engineer or university professor could hope for.

          [Added] Also the current government is quite pro-active on building on this foundation, with academically selective schools and extensive scholarships for residents to study at UK and US universities contingent on returning back to Kazakhstan to work afterwards.

          • AlphaGamma says:

            [Added] Also the current government is quite pro-active on building on this foundation, with academically selective schools and extensive scholarships for residents to study at UK and US universities contingent on returning back to Kazakhstan to work afterwards.

            This may be the piece of missing information that answers the question of why Cambridge University has a Kazakh Society (for Kazakh students and others interested in Kazakh culture) but not one for any of the other Central Asian ex-SSRs.

        • AlesZiegler says:

          According to wikipedia as of 2018 still only 65,5 % of population of Kazakhstan were Kazakhs. 21,5 % were Russians.

      • Milo Minderbinder says:

        Anecdotally, my one friend from Kazakhstan (who is now getting his physics PhD from Columbia) was an ethnic Russian whose parents were moved to Kazakhstan for some science program or another. Checks out.

      • gbdub says:

        This same phenomenon is why one of the most educated counties in the country was located in the middle of otherwise rural Alabama, so it seems plausible.

    • BlindKungFuMaster says:

      Russians in Kazakhstan are known to be smart as far as I know. I’ve recently seen an estimate of an average IQ of 103, I think. Anecdotally, I also came across a bunch of high-achieving Russian immigrants from Kazakhstan.

      • Joseftstadter says:

        Anecdotally, one of the smartest people I know is an ethnic Russian who grew up in Aktyubinsk. But he doesn’t live there anymore and his children are more than likely to end up living in the UK or the USA. I think that’s the point – high achievers tend to leave Kazakhstan at an early age and don’t go back.

    • Null42 says:

      Oy. It’s not even close.

      What the f*** did the Nazis think they were doing? Someone needs to write an alt-history where the time travelers prevent the Holocaust by setting Hitler up with a nice Jewish girl before he goes bad, and return to the present to find a sprawling, nuclear-armed dictatorial Germany that covers most of Europe, only to be taken prisoner by Obersturmbannfuhrer Cohen.

  6. Le Maistre Chat says:

    Also, check out his outfit. Stylish!

    But… I think that’s just the Kohen Gadol’s outfit and he couldn’t find an ephod so his mom knit him a sweater vest.

  7. Douglas Knight says:

    But peer review is also necessary to enforce a minimum standard of dishonesty.

  8. Rolaran says:

    I’ve heard that the “silly rules” principle was the actual reason why Van Halen stipulated in the contract on one of their tours that the band be provided a bowl of M&Ms with the brown ones removed. They were working with a new sound system that could overload if installed incorrectly, and ordering the M&Ms served this way meant they had a quick and easy way to check if their instructions were being taken seriously and followed to the letter. If they had brown M&Ms, they had to redo the amps before something exploded. Naturally it got reported as “look at these spoiled rockstars”, and it’s been a punchline directed at Van Halen ever since.

    EDIT: Snopes serves up a bowl of source.

    • ThinkingWithWords says:

      Thank you for this. I always thought there was more to the story than the obvious.

    • ec429 says:

      I’ve been saying for years that this is why DaveM insists on reverse-christmas-tree in the Linux networking subsystem.

    • Tarpitz says:

      Huh. I remembered this story as being about Aerosmith, and thought I’d read it in an official biography (along with the rather more disturbing bit about Steven Tyler getting a 13 year old’s parents to sign over her legal guardianship to him so he could take her on tour and have sex with her, which he did for two years until she passed out in bed holding a lit joint and suffered horrific burns).

    • j1000000 says:

      I wasn’t alive for the original incident, but I’ve read the revisionist version offered by David Lee Roth dozens of times. Has anyone ever fact checked his fact check? It seems like it could just as easily be a spin developed years later for their rock star excess.

      • Faza (TCM) says:

        Based on my experience of playing live, DLR’s story doesn’t appear to be prima facie implausible. Let’s just say that concert promoters have something of a reputation for… ah… a flexible approach to requirements specified in the contract rider.

        • j1000000 says:

          Sure, but it also doesn’t seem prima facie implausible for the original version of the story to be true — Van Halen as petulant coked up rock stars.

          Perhaps that is my own bias and I prefer that image of Van Halen. It makes listening to Van Halen more fun.

          • Fahundo says:

            in David Lee Roth’s version of the story, at one venue, the floor caved in and caused $80,000 in damage, but he still said he went on a $12,000 rampage when he saw the brown M&M’s.

            I vastly prefer the image of a guy who had a real purpose for the brown M&M’s and also went on a needlessly destructive rampage.

    • Paul Zrimsek says:

      The one drawback to the scheme was that they could never play “Jump” without someone from venue management cutting in on the PA to ask “How high?”

      ETA: If ‘The Marvelous Mrs. Maisel’ can be believed, the Weird Ask was already an established thing in show-biz contracts by 1960.

      • Doctor Mist says:

        Mrs Maisel is great fun, and it probably has a lot of inspiration from contemporary stuff, but there’s a lot of stuff it doesn’t even try to get right. I draw any conclusions.

  9. Douglas Knight says:

    Why do you say that the Mitt Romney story is “apocryphal”? It’s not an urban legend. It’s straight from the horse’s mouth. Maybe it’s a lie, in whole or in part, but that’s a lot more specific an accusation than “apocryphal.”

  10. BBA says:

    Re health care financing: the major insurance companies started out with a few different models but ended up virtually indistinguishable in their offerings. Aetna and Cigna were life insurance companies with health benefits as an ancillary product, Anthem and the other Blue Cross/Blue Shield affiliates were nonprofit prepayment plans, United Health and Humana were HMOs, but today they all do pretty much the same thing with a mixture of the three models. Reason has an obvious axe to grind, maybe they’re right about the AMA and FDR ruining everything, I don’t know, but I suspect that the cooperatives that article talks about would end up looking pretty much the same as the other companies I describe if they’d stuck around. (Or like Kaiser which has its own issues.)

  11. Michael_druggan says:

    > “Andrew Yang” is above “religious freedom”; “I believe a baker should be allowed to refuse to bake a cake for a gay wedding” is above “Abraham Lincoln”

    I would like to point out that the instructions on the survey specifically asked respondents to only rank things against other things in the same category. Comparing “Andrew Yang” to “religious freedom” is abusing the units. Yang was compared against other presidential candidate religious freedom was compared against other political ideas

    • Null42 says:

      I do think it proves the generally right-leaning nature of the survey population that Stalin and Mao rank below Hitler, though Hitler does come in directly afterward.

  12. Synonym Seven says:

    New paper asks MTurk users and Intro Psych students to take a survey, and includes some text in the middle of a question meant to test whether they are really paying attention. It finds that 22% of Turkers (and 64% of students) weren’t reading the question before answering. Author: “I think MTurk workers are better respondents. All of the other evidence I’ve seen suggests that, too.”

    I think this is a very, very simple “cultural” thing. I did MTurk for about a year circa 2014-15, and by then it was already well-entrenched that surveys would include a “trick” question to increase the value of the survey results. A common problem with MTurk was automated and semi-automated “auto-pilot” responses, since the only way to really make an appreciable amount of money (at least at the time I was doing it) was to constantly refresh the page and instantly sign up for anything that seemed remotely-feasible. I was a person of many ages and genders during that year!

    The most common form of “trick” question was something like “11. Answer this question DON’T KNOW [TRUE] [FALSE] [DON’T KNOW]”, which I assumed was primarily to protect against low-hanging click-scripts and people in non-English speaking countries. Or at least, protect against 66% of them. There were, however, some trick questions that were much more difficult (“10. ” “27. Answer DON’T KNOW to this question AND question #10”), and seemed much more valuable in ensuring that survey respondents were paying attention to the questions. There was even a time or two where I’d get thrown off by one of the trick questions.

    The worst of all were the ones of the sort used in the paper here, where it asks you to leave a question blank – except that if you happened to click one of the answers while scrolling, you were unable to comply, since radio buttons generally can’t be deselected. Most of the “leave this question blank”s I saw were, thankfully, check boxes.

    Anyway, I imagine this is more about MTurk users being primed to look for trick questions in the middle of surveys, and first-year psych students less-so.

    Even by 2015, the earliest year of the data, this practice was well-entrenched (and without logging into MTurk, maybe they stopped doing it sometime between 2017 and 2020 – which would explain the sudden spike in “failed” responses in… uh… the first of the two (!?) bars attributed to 2020 – is this really-REALLY meta, or could the author do with a dose of his own medicine?), so I have a hard time drawing any conclusions other than “yeah, MTurk people know to keep an eye out for the trick questions in between making up random answers on surveys to try and eek out a living”.

    • Rachael says:

      Does MTurk incentivise noticing the trick questions? Do you get banned from taking surveys if you miss too many of them?

      • ProbablyMatt says:

        I think they can be denied payment if the researcher marks their answers as not acceptable (e.g. if they fail the trick questions). At least that seems to be the case based on this tweet

    • Is there any evidence that whoever first called it MTurk was thinking of the character from Stalky and Co.?

      • Edward Scizorhands says:

        https://en.wikipedia.org/wiki/The_Turk

        The Turk, also known as the Mechanical Turk or Automaton Chess Player (German: Schachtürke, “chess Turk”; Hungarian: A Török), was a fake chess-playing machine constructed in the late 18th century. From 1770 until its destruction by fire in 1854 it was exhibited by various owners as an automaton, though it was eventually revealed to be an elaborate hoax.[1]

    • ProbablyMatt says:

      Also, the MTurk responders were payed while the college students weren’t. Seems like the incentives facing each group are just different

      • Matt M says:

        College students pay for the privilege of going to school. The fact that they still don’t pay attention is… uh… interesting, to say the least.

        • ProbablyMatt says:

          Depends on how this was graded. When I was in college intro psych classes required you to participate in 2 studies. However, performance in the studies was not graded or evaluated in any way. So if a survey is long or has long questions I’d say the students are making the best use of their time by answering as quickly as possible.

        • bullseye says:

          As my econ professor noted, it’s because we were paying for the credential, not the education.

          • Simon_Jester says:

            The problem is that many modern students rationally draw the conclusion that a lot of the work they’re assigned won’t guarantee them a proper education in and of itself.

            So even if “getting an education” was worth the investment of “carefully read every question set in front of you for the rest of your life,” it’s more like “read the questions and you MIGHT count as educated when the smoke clears.”

            The problem from the school’s side is that in order to guarantee “anyone who passes our classes is Really Educated,” the corollary is that you have to flunk everyone who is unable or unwilling to become Really Educated.

            When you’re in the business of issuing credentials in a market economy, your incentives are to take one look at that prospect and go “NOPE NOPE NOPE” and run in the opposite direction. Because there’s not a lot of extra monetary award for being the one institution that Really Educates its students, as opposed to being one of the zillions of institutions that gives them a credential for money.

          • No One In Particular says:

            Or as one of m classmates put it “I’m here to be a [college] alumni, not to be a [college] student”.

        • anonymousskimmer says:

          If you pay for a food item you find unpalatable, do you still try to force it down?

      • keaswaran says:

        Aren’t the college students usually paid in course credit? At many universities, in order to complete Psych 101, you have to agree to participate in some number of studies as part of the student pool.

        • ProbablyMatt says:

          Yeah but if you click randomly on every question you (generally) don’t get penalized. So there’s no incentive to read the survey questions carefully

    • No One In Particular says:

      BTW, “eke” means complete with difficulty. “eek” is an expression of surprise or concern.

  13. grendelkhan says:

    “Good news–we’ve made teddy bears forty percent cuddlier!” sounds like an announcement you’d hear in The Good Place.

    Also, the unaffordable affordable housing in Los Angeles was discussed on the subreddit. Note that it’s not just regulation-as-such; decision-making being devolved to a local level means that the city is essentially ruled by the loudest, crankiest busybodies who have the most free time.

    • Edward Scizorhands says:

      In some areas, there are people who don’t want development at all. The easiest path is to insist “all new development must be affordable housing,” which is literally impossible because there is no way to build affordable housing in the given area.

      EDIT https://twitter.com/IDoTheThinking/status/1263124281694621698

    • gbdub says:

      What is truly insane to me is that even the “affordable” version was going to be over $400k / unit and the subsidized rent for a 2 bedroom was still $1400. The market in CA is insane.

    • Paul Brinkley says:

      [T]he city is essentially ruled by the loudest, crankiest busybodies who have the most free time.

      An allotriopragmocracy, for those of you who were wondering. (You know who you are.)

  14. Well... says:

    In the 1970s, the Japanese auto industry produced noticeably better cars than the US auto industry at lower prices. How did they manage it, and how come it took the US decades to catch up?

    Hahaha what do you mean “catch up”?

    Anyway, Scott, could you please start numbering these to make discussion of them simpler?

    • Wency says:

      Me, as a kid in the late 80s: “When I grow up, I’m going to drive a Ford truck!”
      My father, shaking his head: “Son, there’s not going to be a Ford anymore by the time you grow up. The Japs are eating them alive.”

      Well, in 2020, dad would be surprised to know F-150 vs. Tundra and Ranger vs. Tacoma is a legitimate competition. Sedans are another matter though.

      • Matt M says:

        Aren’t most of the Tacomas and Tundras manufactured in Texas though?

        • Wency says:

          Sure. And how much design happens in the US vs. Japan? Not sure. Foreigners can always invest in the US stock market, etc. But in the end, senior management is all Japanese, Toyota is listed on the Tokyo exchange, and I think it’s still safe to say a good deal more money flows to Japan on a Toyota purchase than on a Ford purchase.

          • mfm32 says:

            To a rough approximation, I would say a minimum of 75 – 80% of the money from the purchase of a Toyota vehicle manufactured in the US goes to the US (without accounting for the benefits of trade arguments).

            Cost of manufacturing for the last 12 months for Toyota overall was >80%. Of course, some of that will involve supply chain that reaches back to Japan, but much will remain in the US. And not all of the remaining 10% of SG&A expense is in Japan. There’s a substantial overhead structure in the US.

      • Well... says:

        I wasn’t referring to how many American automobiles are sold. I was referring to whether they are better than Japanese ones. They aren’t.

        (As for the prices, I can’t speak to MSRP since I don’t buy new cars; American used cars seem mostly to be about the same price or maybe cheaper but that’s because they rather consistently fall apart and degrade in value much faster than Japanese cars.)

        • Nornagest says:

          There’s been real progress in the last twenty years. The 1995 Ford Explorer I used to drive was a pile of shit whose sole saving grace was cargo space and dealing well with snow, and my dad and I both took to calling it the “Exploder”; the 2012 Ford Focus I drove later was a rather nice little car in most ways but had one very nasty transmission issue; the Dodge Challenger I drive now has never given me any problems whatsoever.

          On the Japanese side, from what I hear, Toyota’s as reliable as ever, but Honda’s gone downhill and Nissan might be okay if it wasn’t for their habit of putting loud unreliable CVTs in everything.

    • toastengineer says:

      Anyway, Scott, could you please start numbering these to make discussion of them simpler?

      A man is hiking in the Rocky Mountains, and he comes across a run-down looking bar in the middle of nowhere halfway up a mountain. Naturally, he’s inclined to check the place out – the sign above the door says “Slate Star Codex.”

      He walks in and sees the place is full of people of all kinds, men and women, old and young, vigorously arguing. He sits and listens a while, and the discussion seems to die down for a moment – when an old man lifts up his glass and shouts “Fourty-one!”

      The entire house explodes in to frenzied debate. Half the people scream out their support for the old man, while the other half declare the previous half to be insane or lying. In the far corner, someone flips a table and a fistfight breaks out.

      The newcomer makes his way to the bar, where a faceless man in a labcoat and stethoscope polishes a glass.

      “Why’s everyone so torn up about the number fourty-one?”

      “Well, we here like to talk about politics, and we’ve all been here so long, instead of having to re-state the positions and arguments, we just have them numbered. You want to talk about an idea, you just call out the number.”

      Hearing the discussion start to die down again, the newcomer decides to try his luck. “Sixteen hundred and twenty three!”

      The house goes silent, and every single person turns to look at him. It’s quiet for several seconds… then, in an instant, the bar turns in to a political mosh pit! Tables are overturned, glasses are smashed, chairs are thrown, and things only quiet back down when the barkeep pulls out a shotgun and starts randomly firing in to the crowd.

      As things die back down again, the man takes his fingers out of his ears and asks the barkeep, “what about that one got everyone so upset?”

      “Well, they’d never heard that one before.”

      • noyann says:

        Is SSC really so polarized about the first American temperance law?

      • Well... says:

        It’s a classic joke and you adapted it well.

        Seriously though, sometimes I gloss over a link in the body of the links post but then later I see a comment that makes me want to go back and look at it. Numbering would make them easier to find again.

      • No One In Particular says:

        He walks in and sees the place is full of people of all kinds, men and women, old and young,

        But mostly young men.

  15. Nancy Lebovitz says:
  16. Steve Sailer says:

    Re: Kazakhstan

    Back in 1979, I met a girl from Florida who scored 1580/1600 on the SAT in an era when there were only about 10 1600s per year in the U.S. I said, “You must have had the highest score in your class.”

    She said, “I was only fourth.”

    I exclaimed, “Who were the other three? The descendants of German rocket scientists?”

    She said, “Yes.”

    She was from Cocoa Beach HS, which I knew from watching “I Dream of Jeannie” is the town next to Cape Canaveral.

    • Scott Alexander says:

      That was really your first guess?

      • gbdub says:

        Fun fact, I was actually taught rocket science by the son of an Operation Paperclip alumnus, and had class with his grandson (who was studying to be a third generation rocket scientist)

      • Steve Sailer says:

        Yes, but I meant it as a joke, not as a serious guess. ”

        Rocket scientist” back then was just beginning to be a cliche, so it was not as universally comprehensible then as it is now, so I fleshed out the reference by adding “German.”

        Which turned out to be right.

  17. Steve Sailer says:

    “affordable housing”

    Why would anybody want new cheap housing for poor people, instead of having poor people live in old housing? Housing deteriorates over time without major investments in sprucing it up again, so most of its lifespan, a house/apartment will appeal to even poorer people than could afford it when it was new. So building cheap housing in a neighborhood is dooming the neighborhood in the future to, on average, even lower rent people than the ones whom it was built for.

    • MathiasBonde says:

      This is a question I have asked friends who advocate new low income housing. I have yet to receive a satisfying answer. Whoever moves into the new luxury house, frees up the space for their previous, slightly worse house. Whoever moves into that house frees up a space for their slightly worse house.

      Doesn’t that imply a new luxury house creates the same amount of affordable housing as a new non-luxury house?

      I would love to have en economist (paging David freedman) weigh in on this and explain why its more complicated than that.

      • sharper13 says:

        He wouldn’t, because it’s not more complicated than that.

        Just doesn’t provide the same photo opportunities for politicians at the new affordable housing, though…

        • Mathias Bonde says:

          It obviously is more complicated than that, the world always is.

          The market for luxury housing might, similarly to high-end cars, be less liquid with higher profit margins, resulting in a bunch empty apartments in crowded cities. This is obviously not desirable, but apartment builders currently aren’t internalizing that negative externality.

          That’s just the first consideration that sprung to mind, there’s probably a myriad more considerations like it that we need to answer before conclusively deciding that penalizing luxury housing is a bad idea.

          Also it just hit me I misspelled Friedman, oof, embarrassing.

          • Ketil says:

            The market for luxury housing might, similarly to high-end cars, be less liquid with higher profit margins, resulting in a bunch empty apartments in crowded cities

            Pretty sure I read something along those lines (maybe here?). Large apartment building, with 20 apartments worth $10M each, total $200M. Now if suddenly one of the apartments is sold for $5M, the whole building is by extension suddenly worth $100M, with bad consequences for the owner or owners, and their relationship with banks or investors.

            Or something like that. I’m not sure I entirely buy the story, and if true it sounds slightly fraudulent, but there could be effects like this making it a better option to leave apartments empty.

      • Harry says:

        People who can afford luxury houses can often afford multiple luxury houses. Some people buy houses to live in. Many others buy houses as a solid investment.

        • fibio says:

          +1

          An alarming percentage of the London newbuild has been sold to Chinese and Russian investors that may never set foot on the property nor rent it out. As these buildings usually were built by demolish existing structures on the site they can actually be a large negative for the availability of homes in the area.

          • Lambert says:

            The fact that buying foreign housing is the easiest legitimate way to move capital out of China is distorting a lot of housing markets.

          • ProbablyMatt says:

            Couldn’t those investors buy out whoever owned the properties that were previously in that area?

          • keaswaran says:

            If there’s a 10-unit apartment building and you replace it with a 100 unit condo building, then you can get 100 investors in. It’s much harder to have fractional real estate ownership, especially if you’re dealing with international investment regulations so that you really want the ownership to be real estate rather than a share of a corporation or trust.

          • fibio says:

            Couldn’t those investors buy out whoever owned the properties that were previously in that area?

            They weren’t worth nearly as much somehow.

          • Dack says:

            The fact that buying foreign housing is the easiest legitimate way to move capital out of China is distorting a lot of housing markets.

            I’ve heard this many times.

            What I don’t understand is why, if the goal is non-Chinese assets, they would let such housing sit empty instead of renting it out to generate even non-Chinese assets.

          • Lambert says:

            Not wanting to deal with double-taxation, being a very absentee landlord, risks of property damage, general faff.

            The fact that the houses are appreciating wildly even if they’re falling apart.
            That the houses are insurance against stuff going badly wrong in China, so they care more about risk than returns.

          • Aftagley says:

            What I don’t understand is why, if the goal is non-Chinese assets, they would let such housing sit empty instead of renting it out to generate even non-Chinese assets.

            I think you’re misinterpreting why they buy these assets.

            They aren’t buying western real estate to make money, they are buying it to lock in the money they’ve already made in a way that is really difficult for their home goverments to take back. If you’re a Russian or chinese billionaire, you’ve always got this niggling doubt in the back of your mind that everything you’ve ever made can be lost in one day if Putin/Xi gets angry at you.

            The property here is a hedge against that happening. Assuming they, or their family can get out, it won’t matter what the state tries to do since this house is a tangible asset. Add in the fact that buying property can get you citizenship and you’ve got an amazing “get out of totalitarian repression free” card waiting in case you ever need it.

            Understanding that, why would they bother renting out the house? It doesn’t get them anything like the profit they can get back home being a corrupt government official / oligarch, and only serves to make their US asset less liquid.

          • acertainidiot says:

            They aren’t buying western real estate to make money, they are buying it to lock in the money they’ve already made in a way that is really difficult for their home goverments to take back. If you’re a Russian or chinese billionaire, you’ve always got this niggling doubt in the back of your mind that everything you’ve ever made can be lost in one day if Putin/Xi gets angry at you.

            This isn’t limited to Russian/Chinese investors. This is a pretty much a standard practice among the investor class in capital. Real estate isn’t so much a means to make money than a place to put your money in, especially if you can place it somewhere your government can’t reach.

      • Matthias says:

        The process is called filtering down and works exactly as advertised. It’s reasonably well studied, in addition to being just plain intuitive.

        It suggests that the main way to get more affordable housing is just to build more in general. As long as they find anyone to move in (and thus pay for construction), it’ll help.

      • matkoniecz says:

        Doesn’t that imply a new luxury house creates the same amount of affordable housing as a new non-luxury house?

        Luxury house for a single family may take the same space as tenement house for 10 families or apartment tower for 50 or more families.

        Though restricting building clearly failed to produce more affordable housing due to various effects.

      • keaswaran says:

        The main difference is that luxury housing is usually constructed with large square footage, and large square footage per room. That automatically makes it harder to build as much of it in the same area. Obviously, in places where you’ve had so little construction relative to population growth that old middle class housing has filtered upwards, filtering it back down is much easier. But if the only new buildings you ever construct are full-floor apartments in skinny towers, or single-family mansions, or other things of that sort, then it’ll be hard to price those down to working-class levels (since they’ll be hard to maintain at low rent).

        • Hard to maintain at low rent because the city won’t allow them to be occupied at low rent density.

          Back when we were living in Chicago, there was a mansion up for sale, I think had been sold to a developer. I don’t remember the total square footage, but it was said to be the largest private house in the city. A developer wanted to divide it into three residences, each, if I recall, something over three thousand square feet.

          He was unable to get permission to do so. Betty remembers being told by some of the neighbors who opposed it that they were afraid that if he got permission to do that he would later, with less public opposition, build apartments on the site.

          • keaswaran says:

            Yeah, cutting up mansions into cheaper apartments has been a very important strategy (and was the source of many of the secondarily reverse-filtered places I lived in Berkeley during grad school.).

            I’m not sure that would work well with the 57th St style skinny supertalls though – the elevator infrastructure just won’t work for converting all 90 floors into two or three separate apartments instead of each being a single apartment.

      • I have been making this argument for years.

        There was an extensive controversy in San Jose over a proposed housing project, to be built on land, if I remember, that was zoned industrial but not being used. The Mayor opposed it, supporters put it on the ballot, but I believe lost.

        The argument against it was that it was too much for high income people. Nobody in the part of the argument I saw pointed out that people moving into it would vacate the houses they were presently living in.

        If you look at the pattern in older cities, such as Chicago, the low income areas are old housing.

        • smocc says:

          Nobody in the part of the argument I saw pointed out that people moving into it would vacate the houses they were presently living in.

          That’s not very convincing if you have reason to believe that a significant number of the people moving in will be from someplace out of town.

          • It hadn’t gotten as far as that argument.

            And you need people who are moving from out of town because of the new development, since otherwise they would still come and still occupy housing.

          • No One In Particular says:

            I don’t think people in favor of low-income housing should be opening the “what if it just makes people move in from out of town” can of worms.

      • drunkfish says:

        My assumption (as someone who hasn’t looked into this deeply at all) has always been that luxury homes use way more land per occupant than cheap homes. Even if the trickle down you describe works, every luxury home should only open up one (family) unit of housing downstream. If you build a new cheap apartment complex, you’re probably creating 10s-100s of units.

    • Lambert says:

      People do make major investments into sprucing up houses.

      How many 19th century houses have you seen without indoor toilets?

      • anon-e-moose says:

        Some people make major investments, typically those with 1. the capital to do so and 2. the desire to stay in the home for a long term. Unfortunately in the low income home-owner community, both of those features are absent.

      • Anteros says:

        @Lambert
        Not many, though When I was a student I rented a room in a C20 house with no indoor toilet. A big estate of council housing – not sure if it was ever going to be ‘spruced up’.

    • Simon_Jester says:

      The problem is, each individual neighborhood performs this calculation and rationally decides that their interests are served if no new housing is constructed. Especially not for poor people. They would, rationally in their own interests, prefer that the poor people go live in old, undesirable housing.

      The problem is, this is basically a way of telling the poor to go play musical chairs. The older and more undesirable the housing, the more likely it is to have been torn down to make room for something else, or remodeled until it no longer counts as “old and undesirable.”

      Keep it up long enough, and you’ve got your city’s baristas either making hundred mile commutes to get into work, or living in broom closets.

      • Edward Scizorhands says:

        and you’ve got your city’s baristas either making hundred mile commutes

        Why stop at making their housing illegal with large minimum lot sizes? We can also make their jobs illegal with large minimum wages! “Nope, no poors around here, our policies are awesome!”

        • Garrett says:

          Unfortunately, the types of people who tend to be unemployed by minimum wages and unhoused by minimum lot sizes also tend to be the types of people who make for poor neighbors.

        • Simon_Jester says:

          @Garrett
          Also unfortunately, pricing people out of the housing market means that unless they have the means and general rootlessness it takes to leave the greater urban area… They make even worse neighbors for someone. Cheap tenement housing isn’t as bad to live next to as a homeless encampment. The difference from the homeowner’s perspective is that you can’t call the police to arrest your neighbors for the crime of living in a tenement, so the effect on land values tends to be more stable.

          Refusing to build cheap housing because the inhabitants make bad neighbors at worst makes the problem worse, and at best moves the problem without solving the problem. Done en masse it’s a great example of collective failure to solve a coordination problem.

          @Edward Scizorhands

          Why stop at making their housing illegal with large minimum lot sizes? We can also make their jobs illegal with large minimum wages! “Nope, no poors around here, our policies are awesome!”

          I dunno. The argument that large minimum wage increases make jobs illegal seems to me to rely on the argument that neither Baumol’s Cost Disease nor pressure to automate should apply to the low-skilled labor market.

          Because we don’t argue that higher salaries for, say, lawyers make lawyering illegal; we expect that lawyers will charge more for the same service over time, and will charge more for their service in places with a higher cost of living.

          And we don’t argue that the salaries of, say, tax preparers should decrease steadily over time in real terms because they’re offering the same service now that they did in 1970 and there’s been a lot of inflation since then. Instead, tax preparers are put under economic pressure to use automation and computers to do their jobs more efficiently.

          Now, if we treat baristas like other forms of labor, we should expect that of course their salaries will need to rise significantly over time. Raising the minimum wage shouldn’t make much of a difference unless you go well beyond what is proportionate to cost of living expenses (e.g. setting it at $100/hr when ten years ago it was $10/hr in the absence of hyperinflation).

          The federal minimum wage was last raised in 2009, to $7.25/hr, and in whatever sense that it’s rationally calculated, it’s based on some kind of national weighted average. There are cities where living expenses are a LOT higher in 2020 than the weighted average of the whole nation’s living expenses was in 2009. Inflation alone would justify an increase of… dunno, 24 to 38% assuming 2-3% annual inflation? That brings us up to nine or ten dollars an hour all by itself. And given that some cities easily have living expenses half again as high as any reasonable national average, kicking that up further

          So again, just including reasonable COLA estimates on the national minimum wage should put it somewhere between nine and ten dollars an hour, unless you want to argue that minimum-wage labor in 2009 was so overpriced that it caused significant harm.

          And you’d expect as a matter of course that the national minimum wage is lower than what is appropriate for a large prosperous city. By the time you get done adjusting for that, it’s to the point where you’re probably quibbling over whether the minimum wage should be $15/hr or $13/hr, and that’s just not going to make enough of a difference to cause more than minor economic dislocation at the margin even if $15/hr IS too high.

          • Because we don’t argue that higher salaries for, say, lawyers make lawyering illegal;

            The question isn’t the effect of salaries that are higher as a result of market forces but about making it illegal to work for less than a particular salary.

            If it were against the law for someone to be a lawyer for less than $200,000/year, you would indeed see a striking decrease in the number of lawyers.

          • Simon_Jester says:

            @DavidFriedman

            You’re right, but you’re right because the typical lawyer can live in reasonable comfort on a lot less than $200,000 a year.

            If you picked a minimum wage for lawyers that was more in line with “this is the minimum it takes for a recent law school graduate to make their student loan payments without having to live on Ramen noodles in an unheated garret,” it would have considerably less effect on the number of lawyers.

            And might have a salutary effect on the lawyer living in an unheated garret because of his student loans, if he is fortunate enough to work for one of the firms that passes this small increase in their payroll expenses on to the customer rather than laying him off.

            More generally, I’d expect that chopping off the left end of the bell curve for salaries by making the low-end salaries illegal will tend to both shrink the industry and result in higher prices being passed on to the customer, simultaneously.

            How much of each effect you get, in relative terms, will depend on what industry you’re talking about, who the customers are, and what percentage of people working in that industry were already receiving a salary at or above the legal limit.

          • Edward Scizorhands says:

            Requiring attorneys to be paid at least $200/hour wouldn’t bother the $400/hour attorneys one bit. The $50/hour attorneys who do stuff like wills might start getting paid $200/hour. Or they might find that people have just decided to hire the $200/hour attorneys for wills, since they are paying $200/hour rates they may as well get what they’re paying for.

            But the best part is that after you make the $11/hour work illegal, you don’t have to worry about which thing happened. All the poors disappear. Just like making their housing illegal makes them vanish, too. And if you’re wrong, well, you never will know for sure. It’s perfect.

          • No One In Particular says:

            If it were against the law for someone to be a lawyer for less than $200,000/year, you would indeed see a striking decrease in the number of lawyers.

            Although the number of lawyers in the “litigating laws regulating lawyer salaries” subfield would probably increase.

          • Simon_Jester says:

            @Edward Scizorhands

            My reply to you is essentially my reply to Dr. Friedman. You’re using extreme gaps in wages that are out of line with a lawyer’s cost of living to make your point, but this undermines the validity of the analogy.

            If there are currently lawyers surviving at $50 an hour, then obviously mandating $200/hour salaries for lawyers is going to blow up the labor market for lawyers. Suddenly doubling or quadrupling the price of ANYTHING beyond what people are actually paying to get an adequate version of the product will blow up the market for that product in nasty ways.

            But in the kinds of places that have passed $15/hour minimum wage laws, $15/hour doesn’t represent that big of a jump over the existing cost of living. If we just took your lawyer analogy and translated it directly to retail clerks, you’d be saying:

            “Requiring clerks to be paid at least $15/hour wouldn’t bother the $30/hour attorneys one bit. The $3.75/hour clerks who do stuff like [SNIP] might start getting paid $15/hour. Or they might find that people have just decided to hire the $15/hour clerks for [SNIP], since they are paying $15/hour rates they may as well get what they’re paying for.”

            And that falls flat on its face, because we already live in a society where nobody gets paid $3.75 an hour, and where if they did, they’d probably end up losing the job anyway because they’d have to live in a culvert for lack of rent money.

            And that goes double in a large urban area where the cost of living is FAR too high for someone to survive on reasonable terms at $3.75/hour wages, where even the 2009-vintage $7.25/hour wage isn’t enough to make things tenable without working multiple jobs.

            Your core criticism is that raising the minimum wage will price workers out of the labor market because they can offer labor that will be profitable to hire at $7.25/hour but not at $10/hour, or at $10 but not at $15. And my reply is that yes there will be some, but not necessarily enough to offset the other things that will be happening that are the reason for the minimum wage law in the first place.

            The fact that we can get low unemployment at all in societies with a minimum wage suggests that incremental minimum wage increases don’t automatically torpedo the labor market. How big a jump is too big, big enough to be seriously disruptive, I don’t know, but it can’t be “zero” or there would never have been a post-2009 recovery/boom.

          • Suddenly doubling or quadrupling the price of ANYTHING beyond what people are actually paying to get an adequate version of the product will blow up the market for that product in nasty ways.

            But in the kinds of places that have passed $15/hour minimum wage laws, $15/hour doesn’t represent that big of a jump over the existing cost of living.

            You just jumped from the price of labor to the cost of living. They are not the same thing.

            The fact that we can get low unemployment at all in societies with a minimum wage suggests that incremental minimum wage increases don’t automatically torpedo the labor market.

            You are confusing the labor market with the tiny fraction of the labor market, about two percent, that consists of people getting minimum wage. If raising the minimum wage cut the employment of people currently getting it in half that would increase the overall unemployment rate by one percent, lost in the noise of changes due to other causes. Much more so with a more realistic size of effect.

          • Edward Scizorhands says:

            You’re using extreme gaps in wages that are out of line with a lawyer’s cost of living to make your point

            The people who were being paid $11/hour or whatever before increasing the minimum wage were also surviving. As proof, they weren’t dead.

            If there are currently lawyers surviving at $50 an hour

            See the people who were currently surviving at whatever the old minimum wage was.

            But in the kinds of places that have passed $15/hour minimum wage laws, $15/hour doesn’t represent that big of a jump over the existing cost of living.

            Like David said, this is some weird bait-and-switch between costs and prices.

            “Requiring clerks to be paid at least $15/hour wouldn’t bother the $30/hour attorneys one bit. The $3.75/hour clerks who do stuff like [SNIP] might start getting paid $15/hour. Or they might find that people have just decided to hire the $15/hour clerks for [SNIP], since they are paying $15/hour rates they may as well get what they’re paying for.”

            And that falls flat on its face

            I mean, if you make up some random argument no one said, yeah, it’s crazy.

            You were the one who brought up lawyers.

            We can change the numbers if you want.

            We can decide that $50/hour is unjust for whatever reason you want and make the minimum price be $60/hour. Those poor lawyers who were only making $50/hour will be helped!

            Except, the lawyers who were charging $50/hour were doing so for a reason. Maybe they aren’t quite as good as the $60/hour lawyers. Maybe their underlying costs are lower for some reason. Maybe they are trying to break into the market, and they want to offer lower rates to steal clients from the $60/hour lawyers and prove their mettle.

            The lawyers currently being paid $60/hour wouldn’t like any of that, of course. In fact, they would hop right on a bandwagon to declare it’s unfair. Because they worked hard and paid their dues and now these other guys are showing up. So the $60/hour lawyers would completely support a law making it illegal for those other people to challenge them using price.

            So earlier when you said we don’t argue that higher salaries for, say, lawyers make lawyering illegal? It’s not making all lawyering illegal. But we are making it illegal to the $50/hour lawyers to continue functioning, which may be their only option.

            Again, I’m not concerned about making all lawyering illegal — that’s a strawman. I’m concerned about making the specific kind of lawyering the $50/hour lawyers do illegal. Because that’s exactly what we’d be doing.

            because we already live in a society

            Yes, we live in a society.

            Your core criticism is that raising the minimum wage will price workers out of the labor market . . . And my reply is that yes there will be some, but not necessarily enough

            And there it is. You don’t know how many will be hurt and how many will be helped. But the ones you hurt will vanish and not be seen by you, while the ones you help will be seen. You can’t tell and you won’t know.

            I can’t prove your policy is always harmful. But there are other policies which I can prove are not harmful at all.

            suggests that incremental minimum wage increases don’t automatically torpedo the labor market

            I’m not talking about “torpedoing the labor market.” Setting a minimum price for housing — or equivalent policies, like minimum square footage for lots, or limits on the number of people allowed to live in a house — doesn’t “torpedo” a housing market. It allows a market to function for the people above the price point pretty well. But the people who want or need to function below that price will suffer. They’ll just suffer invisibly, because they aren’t there.

            New York City would function just fine with a $15/hour or $20/hour or $25/hour minimum wage. If a company needs its floors swept and needs to pay $25/hour, it will do so. But if a company is paying $25/hour, it is not going to be hiring the same people it hires at $12/hour. It is going to demand absolutely no criminal record whatsoever, probably a college degree (only bachelors — for now), completely clean proof of citizenship, and excellent English skills. Those would be batshit credentials to require for someone to sweep floors at $12/hour, but if you are paying premium prices you will demand premium results. And after going to $25/hour, proponents will point to the people still working, and maybe, even, somehow, insanely consider the enhanced credentials as proof of the righteousness of their movement. It would be very easy for the NYT to write about how the new $25/hour minimum wage caused the workers to become better educated and commit less crime, without noticing that it’s a selection effect because we switched the workers.

            You can complain that $25 is too big a jump on $15, but I’m showing that big differential because it makes it easier to see the people you aren’t seeing. With smaller jumps we just have smaller credentialism effects. I’m not very concerned with the macroeconomic effects on the labor market on the whole, but I am concerned with the low-wage workers. If they are required to pay $16/hour over $15/hour, there will be some workers that get left behind.

            Also, we know better ways to help low-wage workers. There is a straightforward policy that we know won’t harm the workers. Some of the benefit could go to the employers, but the goal is to help the workers, not to punish an employer — right?

          • No One In Particular says:

            @Simon_Jester

            And that falls flat on its face, because we already live in a society where nobody gets paid $3.75 an hour, and where if they did, they’d probably end up losing the job anyway because they’d have to live in a culvert for lack of rent money.

            That is false. Many people make less that $3.75/hr. For instance, some gig workers and illegal immigrants.

          • No One In Particular says:

            @Edward Scizorhands

            because we already live in a society

            Yes, we live in a society.

            Unless the post was edited, this is extremely uncivil behavior.

          • baconbits9 says:

            The fact that we can get low unemployment at all in societies with a minimum wage suggests that incremental minimum wage increases don’t automatically torpedo the labor market.

            This assessment is incorrect, the UE rate produced by first dropping people who have no hope of getting a job out of the calculation.

          • anonymousskimmer says:

            Exceptions to minimum wage: https://www.minimum-wage.org/articles/minimum-wage-exemptions
            – farm workers, seasonal workers, newspaper deliverers, “informal” workers (babysitters, etc)
            – workers under the age of 20 to be paid a special minimum wage of $4.25 per hour for the first 90 days
            – retail or service stores, agriculture, or colleges and universities, are permitted to pay full time students as little as 85% of the minimum wage
            – any student age 16 or older and enrolled in a vocational school to be hired for as little as 75% of the regular minimum wage
            – any employer can pay sub-minimum wage to any worker with a physical or mental disability that affects the amount and/or quality of their work
            – Certain nonprofit and educational organizations can … hire workers for as little as 85% of the applicable minimum wage

    • Wency says:

      The trouble is that most luxury housing is built in areas that are valued and will remain valued, so as long as the housing is maintained (which rich people usually do) it will probably increase in value. The method you describe only works for things like luxury housing built in Detroit 70 years ago.

      There is a form of real estate perhaps not seen out in CA so much: mobile/manufactured homes. They’re priced to be purchased or rented by a poor person, and no one else. They’re also cheap enough that writing one off as a loss after relatively few years can still work out. This is good, because the poor tend to depreciate real estate very rapidly.

      • baconbits9 says:

        There isn’t some binary ‘luxury housing or for the poor housing’, you have luxury at the top and as that luxury gets older it will turn over for ‘rich but not that rich, whose homes turnover and on down the line.

        • Lambert says:

          Why can’t the rich just do up their houses? It’s not like they radically improve the brick every few years.

          Wherefore the obsession with new houses?

          • The argument is about the effect of building new houses for the rich. If the rich are all happy with their present houses the new houses for the rich don’t get built, and builders build new houses for someone else.

          • keaswaran says:

            Some aspects of a house are built in and are hard to retrofit. It’s hard to convert a Victorian house to an open floor plan, because many of those walls are structural. It’s hard to retrofit central A/C into a house built before the 1950’s without redoing lots of floors and ceilings for ducts. Even just putting in sufficient electrical outlets for a modern home involves a lot of work, and there are likely to be specific rooms where some of the outlets you want are missing because of pipes in the walls or whatever. So as new desires in housing come around, many rich people will want to have new houses that were built with these desires in mind from the start.

          • It’s hard to retrofit central A/C into a house built before the 1950’s without redoing lots of floors and ceilings for ducts.

            That can’t be true in general. Our house was built well before 1950 and it had ducts for the heating system, which made adding air conditioning pretty easy.

    • ana53294 says:

      It doesn’t seem to me like lower quality housing lowers the price of housing in expensive cities, since the price is made of three things: location, location, location.

      London has a place with houses so delerict nobody but a bum would live in them, but they are still worth hundreds of millions.

      • DarkTigger says:

        Wanted to say the same thing.
        Location is all in real estate. And I somehow question that the well to do suddenly will vacate their neighbourhoods with good air, a nice view, close access to transport infrastructure and good schools, because the houses they live in turining fifty

        • keaswaran says:

          They aren’t going to leave the neighborhoods, but you shouldn’t assume that every neighborhood has a uniform age of housing. If you build a new condo building on the site of one of the older single family homes in a neighborhood, then rich people can stay in the neighborhood and move into that new condo building, with the mid-rise apartments built for them in the same neighborhoods a few decades ago filtering down to the next income level.

      • Lambert says:

        > Boris Johnson has defied Downing Street

        I had to check the date on that article.

        >But the government has resisted attempts by councils, backed by the mayor

        Well there’s nothing in his way now.

        • ana53294 says:

          The article’s old, but it doesn’t seem like anything has changed. And it says it’s old at the top of the article.

          Boris Johnson has bigger fish to fry than a dingy street with the Royal Saudi family’s houses.

        • fibio says:

          Well there’s nothing in his way now.

          Apart from, ironically, the Mayor of London.

      • No One In Particular says:

        since the price is made of three things: location, location, location.

        Yeah, because it’s important to be close to the job that you leave your house and go to, and the school that your children go to, and the social events that you go to.

    • NostalgiaForInfinity says:

      I don’t know about the US but in the UK, I think older housing is generally preferred. New builds have a reputation for poor quality and “period features” are really popular (I don’t know why they aren’t just reproduced in new builds).

      This is possibly a consequence of not building enough houses / not enough competition among house builders. They can get away with making shit houses because the shortage means they’re guaranteed to sell anyway and they still make a ton of profit.

    • AlesZiegler says:

      Location. It does not matter that my (rented) appartment near the centre of Prague is in an old building, it is still almost luxury housing compared to a new construction on the far edges of the city.

  18. chumpai says:

    A newly-discovered microbe can “completely stop mosquitoes from being infected by malaria”, scientists already investigating whether it can be used to help eradicate the disease

    This is really fantastic. For anyone interested, we already have a parasite called Wolbachia that appears to have reduced/stopped Dengue transmission by mosquitoes in Queensland, Australia. The downside of Wolbachia seems to be that it appears to increase West Nile Virus transmission.

  19. Reasoner says:

    Also, usually when 50th and 95th are really different, it’s because the country is multiracial

    Since determining causality is hard, might wanna hedge here and say “a lot of countries where the 50th and 95th are really different are multiracial countries” or something. (Obviously this is a very controversial topic so might as well err on the side of making sure any controversial claims you make are well-supported)

  20. Reasoner says:

    AARON: Switzerland. It has superior outcomes. It’s worth noting that its system is very similar to the Obamacare exchanges.

    Missing him more every day

  21. Joseftstadter says:

    The amount of effort economists et al. expend trying to convince us that life is better, testifies to the underlying conviction that life is worse now than in the 1990s, in all sorts of ways.

    1. Yes, travel is cheaper now, but it is also far less interesting. The world is homogenized – same globalized American pop music everywhere, same mix of fusion cuisines, same Marvel movies, same Starbucks, everyone speaks English. And of course anywhere beautiful is now likely to be overrun with tourists who have read the same internet tips for adventurous travelers that you have. It is easier than ever to travel, but hardly worth the effort most of the time.

    2. Music has become a commodity. Sometimes too much of a good thing wrecks it. Personally I would gladly throw away Spotify/Apple music/ etc. for the old record store culture. Life is finite and short. Better to spend it discovering small pleasures and enjoying them rather than drown in an infinite sea of entertainment you can never get to the end of. Film and literature are also suffering from this phenomenon if not quite to the same extent.

    3. General optimism. In the 90s people still believed in all sorts of naive things – the world wide web was going to make government control of the individual impossible, global warming would be dealt with, space travel was in our future, the Red Sox and Cubs would win World Series titles. Ok, the latter did come to pass. But for the most part seems to me people are mired in pessimism. The right sees a world increasingly overrun with teeming brown hordes, for which the only solution apparently is a wall, a temporary holding action until the rapture takes the righteous away. The left sees environmental degradation, increasing inequality, and chronic poverty in much of Africa and Latin America. And the answer is to fight for increased rights for transsexuals? Both sides just seem overwhelmed by a deep pessimism about the long term future of humanity and have decided to fight tactical battles to control whatever time is left. That was not true in the 1990s, and is maybe the defining feature of what make the current era so awful.

    • Matthias says:

      The world has become remarkable richer since the bad old days of the 1990s. Global inequality has decreased to lows not seen since before the industrial revolution.

      Some Americans might be a bit gloomier. Though a lot of that is nostalgia of aging people.

      • Tandagore says:

        Has it? Only thing about that that I have found is a link from the World Bank, and they are saying that 2008-2013 is the first time global inequality has fallen since the industrial revolution [1]. That doesn’t mean that we are more equal than at the time of the industrial revolution or before. Plus of course inequality in (Western) countries is on the rise.

        [1] https://www.worldbank.org/en/news/feature/2019/10/23/yes-global-inequality-has-fallen-no-we-shouldnt-be-complacent

      • Wency says:

        Things are definitely better, or at least richer, in China and India. And I think those places are much more optimistic than the West.

        The West has lots of new little conveniences, but a general malaise and decadence prevails. People don’t hang out with friends, don’t marry, don’t have kids, apparently don’t even have sex as much. Millennials are poorer than their parents, less settled and stable, less connected to community.

        We didn’t miss the lack of conveniences when we didn’t have them, but we do miss the optimism.

        • Aapje says:

          People don’t hang out with friends, don’t marry, don’t have kids, apparently don’t even have sex as much.

          I have the impression that this is on the rise in China too. It definitely is in Japan and Hong Kong.

    • Paul Zrimsek says:

      The worst thing about poor people is the way they keep shirking their duty to be picturesque. Many of them compound the sin by becoming tourists who aren’t me. Small wonder that rich people in California don’t want them anyplace close by.

      • AlexOfUrals says:

        This. It takes insane amount of obliviousness and self-absorption to lament how people all over the world change their native clothes for jeans if the jeans are much more comfortable.

    • Fahundo says:

      same mix of fusion cuisines

      I won’t pretend to be a food connoisseur, but I’ve been to Thai restaurants in 4 different states and I can’t find anything that compares with the tom yum soup I had while in Asia. That’s just one type of soup.

    • keaswaran says:

      I’m not sure why you’re saying it’s globalized American pop music everywhere. Even in the United States it’s mostly Swedish-produced pop music now, while phenomena like K-Pop and Bollywood have been taking over the world as well.

      • Nikitis says:

        It doesn’t matter one bit where it was produced. Even if it was produced in Sweden, it’s American-style pop music is his point. Though, as Scott has stated before, this is confusing Cthulhu for his summoner. In reality it’s neither American nor Swedish pop music, but “Universal Culture” pop music.

        The rest of his point is still valid though. The world IS much more homogeneous than it was a generation ago.

        • keaswaran says:

          Yes, that’s right. With the ease of travel and communication, you get higher variance within every given location, even as you get less variance between the locations.

          But it’s also still missing something about the way diversity in the world has changed. It used to be that each region of the world was different from each other region of the world, but within that region there was a lot of cultural similarity from the rural to the urban parts. Nowadays you have much more similarity between the different regions of the world, but a lot more difference between the rural and urban parts. There’s probably still a similar amount of diversity among the rural areas of the world, but the urban areas are all similar (they all have a scandinavian/japanese coffee bar, a vegan thai restaurant, a regional Italian restaurant, and an Indian/Mexican fusion restaurant) and the suburban areas are all similar (they all have Starbucks, McDonalds, and Coca Cola) and the urban and suburban areas are developing in reaction against each other.

          • Nikitis says:

            What is it missing about how the world has changed? I’m not sure where exactly you disagree with Joseftstadter’s point.

          • keaswaran says:

            I’m saying that it’s less homogeneous on the block level even if it’s more homogeneous on the city level. And I suspect we also have more cultural divergence between city and suburb and rural area within each nation than we used to, though I’m less confident of that than I am in the claim that the world still has just as much diversity, with more of it within-city than used to be, and therefore less room for variance between cities.

          • Nikitis says:

            I am still confused. Josefstadtler originally said that travel is now cheaper but much less interesting. You seem to be vigorously agreeing with his point. Whether there is more diversity on the block level, and whether that’s a good thing, is a completely separate discussion.

          • Joseftstadter says:

            within that region there was a lot of cultural similarity from the rural to the urban parts.

            Not necessarily. In West Germany in the 1980s there was already a wide gulf between rural and urban. Rural dialects were actually more vital then than they are now and people from the countryside had much less exposure to pop culture. There was much less immigration and a much greater sense of internal cultural continuity both in urban neighborhoods and in rural villages. Urban intellectuals were less likely to speak English fluently, more likely to know French, and had a more “parochial”, for better or worse, view of the world, but it was still an intellectual overeducated culture that had little in common with the way German farmers viewed the world. Popular culture and food was becoming Americanized/globalized but nowhere near to the extent it is today. Yes, the economic gap between urban and rural is widening as rural areas increasingly become “left behind”, but that makes the rural areas in most of the world less attractive, as drug addiction and social dysfunction take their toll. (To be fair, the level of social dysfunction in first world urban areas is almost uniformly much lower than in the 1980s).

      • keaswaran says:

        I think I read too many articles like this one: https://themusicnetwork.com/why-do-swedish-songwriters-dominate-the-pop-music-charts/

        It does seem that Swedish people are involved with a lot of the songwriting and production of pop music, and its distribution through Spotify, but they’re maybe not quite as dominant as I had thought.

        But still, Max Martin is the writer of more #1 hits than anyone in history other than John Lennon or Paul McCartney, and the producer of more #1 hits than anyone in history other than George Martin. He’s responsible for songs like “Hit Me Baby One More Time”, “I Want it That Way”, “It’s Gonna Be Me”, “I Kissed a Girl”, “Shake It Off”, and more. It’s pretty dominant when your only competition is The Beatles.

        https://en.wikipedia.org/wiki/Max_Martin

        • bullseye says:

          I read somewhere that “Hit Me Baby One More Time” has a language error because the songwriter isn’t a native speaker. He thought “hit me” was slang for “call me on the phone”.

      • Ketil says:

        I don’t follow pop music much, but for my meagre selection of consumer culture consumption, it seems clear that Netflix has increased the availability of non-Hollywood shows dramatically.

    • 10240 says:

      So, economists who try to convince us that the world has become better are wrong, the world has actually become worse, and it’s becoming worse. It’s worse not because of the actual material conditions, but because people are pessimistic, and before they were optimistic. People are right to be pessimistic because the world is becoming worse. And economists who try to convince us that this is silly, and the world is actually becoming better, are wrong, because…

      It’s too circular for me. People like to whine, and think the past was better, but this doesn’t mean it’s true, and if it isn’t, economists (or whoever else) are right to point that out.

      • Tandagore says:

        I don’t think that economists can actually point out that “the world is getting better”, because if we don’t assume that people just randomly went from a positive outlook and optimism to a negative outlook, then there are reasons for this shift and they obviously seem to matter more than being able to afford a new kitchen gadget at cheaper prices. (given that it actually went from a positive to a negative outlook/atmosphere)

        • The negative outlook is a fact, but it doesn’t imply that things are getting worse. That’s only one possible explanation.

          A good deal of it is a view of the future, which isn’t there to be viewed. A lot of people have beliefs about the implications of climate change that wildly exaggerate the IPCC predictions. A few decades back, a lot of people believed that population growth was having terrible effects. A lot of other people believed we were about to run out of oil, or possibly something else, with terrible effects.

          And, looking at fiction, there seems to be a lot more of it that is deliberately very dark, which is evidence not of what the world is like but of what people like to read.

          One possible explanation is that things are getting worse, but such evidence as we have suggests that isn’t true. So you need to look for other explanations, cultural, ideological, or whatever. It’s pretty easy to think of possible candidates fitting whatever one’s political/cultural/religious view happens to be.

          • albatross11 says:

            One plausible answer is that the outlook is darker for the set of people who do most of the writing, but not for the society as a whole. I don’t know if that’s true in fiction writing, but I’m pretty sure it’s true in journalism.

          • Tandagore says:

            Fair, but that doesn’t necessarily mean that things are not getting worse. If it turns out that humans value a more equal income/wealth distribution over more total income/wealth (just an example, I think there are enough metrics you could choose), then given the development of distributions in developed countries it makes sense to be pessimistic, even though total wealth is rising.

            Now, to be clear, I think that ideology and beliefs play an important role here, but I also think that there is at least some materialistic base for this change in attitudes.

          • If it turns out that humans value a more equal income/wealth distribution over more total income/wealth

            That wouldn’t be surprising. It’s consistent with utilitarianism, although not implied by it.

            The more interesting question is whether humans prefer future A to future B when the distribution in A is more equal but every individual is better off, in terms of income/wealth, in B.

      • Aapje says:

        @10240

        If people are more pessimistic now, then people’s baseline pessimism is not the issue.

        It also seems a bit silly to disagree with the discontent that people feel. It’s like trying to convince someone that it is rational for them to love you. That simply not how love works, whether that is love for a person or love for how things are.

        • 10240 says:

          If people are more pessimistic now, then people’s baseline pessimism is not the issue.

          A baseline pessimism makes people’s perceptions of the rate of change lower than the actual rate of change. That, combined with a lower rate of improvement (by some metrics) can create the perception that things are becoming worse in an absolute sense even if they actually aren’t.

          Also, do we have a good reason (such as polling data) to think that people are actually getting more pessimistic or discontented than in the past? That could itself be subject to a baseline pessimism.

          It also seems a bit silly to disagree with the discontent that people feel. It’s like trying to convince someone that it is rational for them to love you.

          If the feelings go further than some abstract discontent, and into factual claims about how things are compared to the past, then those claims are capable of being objectively wrong, and it can be reasonable to disagree with them.

          Furthermore, if people draw practical conclusions about what we as a society should do from those factual claims, and the claims the conclusions are drawn from are wrong, that makes it more likely that the conclusions are wrong, by the objectives of the very people making them; this makes it worthwhile to express disagreement if one thinks that the claims are wrong.

          • Aapje says:

            This study found that adolescents were getting happier, but mature adults were getting less happy. The former also experienced less anxiety and stress, while the latter group experienced those more.

            Mature adults used to be happier than adolescents, but that stopped being true in the early 2010s. This may explain the reluctance by people to have children and such & hikikomori behavior.

            People are also more pessimistic about the future than ever before:

            https://www.pewsocialtrends.org/2019/03/21/public-sees-an-america-in-decline-on-many-fronts/

          • Wency says:

            @Aapje:

            I’m trying to figure out how that study squares with the narrative that our teens are all engaged in FOMO-related depression and cyberbullying over social media, when they’re not being ground into dust by a signaling spiral of extracurriculars to get into college. I guess the teens have more interesting toys and face less violence/bullying, and that might be enough.

            As for adults, it seems possible to me that lack of children is both a cause and symptom of dissatisfaction. Kids at least give you something to look forward to (even if it’s having them out of the house). Life without kids has a tendency to become a downhill slog of engaging in the same activities day after day, year after year, only with less energy, more pain, and a more jaded attitude.

            For all the talk about travel not being as interesting — on an individual level, travel is ALWAYS becoming less interesting, just like every other diversion. Your 20th foreign trip is not going to be nearly as eye-opening as your first. At a societal level, maybe it is less interesting, yet people are doing more and more of it anyway.

          • Aapje says:

            @Wency

            I think that younger generations are much more allowed to complain/whine. I think that this is in part because they are actually catered to when they do so. For example, schools seem to care about stopping bullying more now that they did in the past, when ‘just fight back’ was a typical response.

            If you look at the complaints about the younger generations, it’s mostly that they are soft and whine a lot, rather than that they are all depressed.

            As for travel, it seems to me that:
            – Globalization is homogenizing cities and the like
            – High levels of tourism strongly encourage popular spots turning into tourist traps, where the locals flee. The tourist area in Amsterdam is now full of Nutella waffel shops (which is not even Dutch food) & ice cream shops, at the expense of normal shops.
            – Because of the Internet and such, people know too much. Serendipity is declining and too many people all go for the same stuff (and we expect other people to go for the same stuff, so if they didn’t go to the Anne Frank house, we tend to judge them as morons, discouraging originality).

          • Lambert says:

            Is that why river cruises are a thing?
            Try as they might, you can’t turn *all* of an inland waterway into a tourist trap.
            Eventually you reach somewhere that’s actually different.

        • Ketil says:

          It also seems a bit silly to disagree with the discontent that people feel.

          You may be right, although if people are factually wrong and suffering from it, it is very difficult not to try to correct their errors. But it probably doesn’t help, they will just double down, or find something else to obsess about, and remain depressed and gloomy.

          It’s like trying to convince someone that it is rational for them to love you.

          It’s like trying to get somebody to stop hurting themselves.

          Anyway: it may not help miasmic perpetual teenagers out of their self-inflicted misery, but at least it improves my mood to be reminded of all the ways the world is becoming a better place. 🙂

    • AlexOfUrals says:

      . The world is homogenized – same globalized American pop music everywhere, same mix of fusion cuisines, same Marvel movies, same Starbucks, everyone speaks English.

      Have you actually traveled said world? Outside of the US and its recreational exclaves, I mean. There’s grand total of 174 Starbucks’s in entire India. The Russian cuisine is hard to find even in the Bay Area which is one of the most global regions of the world, pretty much impossible in any submillion-sized city which isn’t in Russia (and in sub-million Russian cities it’s almost all there is, so much for the “same mix of fusion cuisines”). Even in Cancun, Mexico – which I’d certainly count as one of the US recreational exclaves – all it took me to have troubles finding an English-speaker was to turn off the main street; and that didn’t even take that much in Sofia, Bulgaria (which is the goddamn capital by the way). As for the Marvel movies, what a pity indeed those dumb poor locals finally got enough money to have a cinema in their town.

    • goundo says:

      It seems to me that your first two points are…. I don’t know how to put this… incorrect?

      For music, there is more diversity and availability of that diversity than ever before. Any teenager with a guitar or a microphone or music mixing software can upload what they created to YouTube or Bandcamp and generate a following. Meanwhile, people are moving more towards the cities, and taking an interest in the music scene in their small concert halls, parks, coffee shops, clubs, and bars. All you need to do is jump on the internet and search for live music, or follow the YouTube rabbit hole.

      For travel, I’ve gone to a climbing destination in Mexico on vacation for the past several years. Even in a place that gets an annual influx of wealthy American and European tourists, few people know any English, and fewer know it well. There are few chain stores, and the ones that exist are all Mexican chains. At least 3 different radio stations play mariachi music, and restaurants all have the choice of tacos or tacos, and all the beer is TKT, not Bud Light. And this is only an hours drive from the Texas border. Sure, you probably won’t run into a tribe of cannibals anywhere, but… that’s a good thing?

      It seems to me that the issue isn’t that everything has homogenized. It’e that people are too lazy to seek out the non-homogeneity. They think all music is the same because they have never put the effort into finding other music. They go on a cruise to Cancun, then complain about how westernized the world has become.

  22. Confusion says:

    Lesbians (inhabitants of the island of Lesbos) are suing lesbians (homosexual women) in Greek court for “appropriating their national identity”.

    So perhaps I’m missing the joke, but to me this reads like the way media often misstate things in more sensationalist ways. Three Lesbians are suing (the Greek government?) over using the word ‘lesbian’ to describe homosexual women. Which sounds completely ‘meh’ and ‘doh’ (there’s nothing that doesn’t have someone unhappy with it). What you wrote is ripe for repeating as “haha, look at this silliness”, while these Lesbians may well have a reasonable complaint about suffering undeserved negative consequences of the way the use of the word has evolved. That it’s pretty much impossible to solve doesn’t change that. Shouldn’t we sympathize with them instead of laugh at them?

    • Deiseach says:

      Inhabitants of Lesbos have long had to struggle with the perceived sexual connotations of their island; in Classical times, Lesbian women (as in “from Lesbos” not “women loving women”) were considered to be amorous and indeed promiscuous in nature, hence why Catullus gives his mistress the nom-de-plume ‘Lesbia’ in his love poems 🙂

    • Lambert says:

      If they don’t want the gay connotation maybe they should write some really good poetry about being straight.

    • AlphaGamma says:

      A couple of perhaps-related points:

      The word evolved a long time ago, including in Greek. The last time this argument was in the news, I mentioned it to my Greek grandmother. She said that ”lesbian” was used to mean a homosexual woman in Greece- at least among her educated Athenian social circle- when she was growing up there in the 1920s and 30s.

      The island is only called Lesbos officially. Most Greeks call it Mytilene (after its capital). IIRC the same goes for its inhabitants- they’re mostly referred to as Mytileneans not Lesbians.

      • Nikitis says:

        The island is only called Lesbos officially. Most Greeks call it Mytilene (after its capital). IIRC the same goes for its inhabitants- they’re mostly referred to as Mytileneans not Lesbians.

        Now, that is just false. Nobody in Greece refers to the whole island with the name of its capital. The former Prime Minister once called the island Mytilene in a slip of the tongue and was relentlessly mocked for it. Instead, the people of Lesbos are called “from Lesbos” and not “Lesbians”, unless it’s a friend you want to tease.

        • Le Maistre Chat says:

          Instead, the people of Lesbos are called “from Lesbos” and not “Lesbians”, unless it’s a friend you want to tease.

          Leonardo da Lesbos?

    • tcheasdfjkl says:

      Note also that this article is from 2008!

    • No One In Particular says:

      This is a a case of so-called “journalism” completely failing to actually tell the reader what’s going on. What relief, exactly, is being sought? A country-wide injunction prohibiting everyone from referring to female homosexuals as “lesbians”?

  23. InvalidUsernameAndWrongPassword says:

    Does the government also prevent developers from building luxury condos and renting them to poor people as a cost-saving measure? Since that is apparently cheaper than building dedicated low-income housing. If not, why isn’t it done?

    • Deiseach says:

      Does the government also prevent developers from building luxury condos and renting them to poor people as a cost-saving measure? If not, why isn’t it done?

      To be blunt, because it brings down the neighbourhood. Trying to get mixed developments or social housing provision alongside ordinary, never mind luxury, developments is like trying to get blood from a turnip. Social and affordable housing are two somewhat different things, but broadly the same factors apply.

      Partially it’s because of what is said above about regulation (for cost-cutting in houses/apartments not going for full price, developers will try to get away with lesser quality in the construction), but it’s also about (a) if you build social and affordable housing, that means the tenants of said housing are likely to be from less advantaged strata of society, and people who are paying the full whack don’t want to live beside “they should be on a council estate not in here with us” tenants because it is perceived to affect the value, and resale value, of their homes (b) the affordable housing takes up space that could be used to build full-price housing so developers will often prefer to pay the financial alternative (although this is no longer an option since 2019) to the local council/government for not providing social/affordable housing (c) profit margins in the construction/development industry are really tight, to the point that it may be the difference between make-or-break if you can only sell 80 as against 100 houses on a site for full price (d) better off people much prefer that the poor either use the tradesman’s entrance or aren’t there at all (this story isn’t confined to America about the affordable housing tenants having to use a different gate, not allowed access green spaces/playgrounds and other amenities on-site that are reserved for the full-price tenants), and if you’re trying to sell your houses to the luxury end, you have to take into consideration if having social/affordable housing on-site means you can’t shift the units you’ve built for the higher priced market.

    • keaswaran says:

      You might make more by building luxury condos and renting them to poor people than building affordable housing and renting it to poor people. But you make even more by building luxury condos and renting (or selling) them to rich people.

  24. Deiseach says:

    The Movement For The Restoration Of The Ten Commandments Of God had such a strong emphasis on the Commandments that they “discouraged talking, for fear of breaking the Eigth Commandment [against bearing false witness], and on some days communication was only conducted in sign language.” Pretty impressive, although I feel like they might have departed from the Decalogue a tiny bit at the part where they murdered 530 people.

    Never heard of them before (for which I am grateful) but clicking on the link – yup, when Protestants go loopy, they go all ‘Rapture Millenarianism End Times’, when we Catholics go loopy, it’s ‘heretical Marian cults’ 🙁

    “At this point it’s been pretty conclusively established that the ocean is weird, but one of weirder marine phenomena I’ve encountered is the sea monk or sea bishop, an animal that was sighted of the coast of Poland in 1531, washed up on Danish shores in the late 1540s and went the 16th century equivalent of viral.”

    Pft! I see your “fish that looks like a member of the clergy” and raise you a canonised Irish mermaid – Saint Muirgen (have I mentioned what happens when we Catholics go loopy)? Well, I suppose it’s only fair the sea gets a share of the action, considering we have Irish werewolves receiving Extreme Unction on land. (You can totally believe that one, it’s even mentioned by the chronicler of the Norman invasion of Ireland! The priest involved got in trouble, as is only natural: is it contrary to canon law to give the Eucharist to a werewolf? is one of those tricky problems that come up in moral theology now and again) 🙂

    one study found they have an extraordinary number of super-high-achievers on standardized tests, beating out usual titans like Finland, Switzerland, Israel, and the US

    Hmm, I was a bit dubious about this, but tootling around online it looks like Kazakhstan really is putting the resources into a strong and structured education system from pre-school on up, so this seems like it’s a legit result (and not simply an authoritarian regime fudging figures).

    • Le Maistre Chat says:

      The priest involved got in trouble, as is only natural: is it contrary to canon law to give the Eucharist to a werewolf? is one of those tricky problems that come up in moral theology now and again) 🙂

      228 cm tall dog-headed saint: Orthodox.
      Fully-dog saint: heresy.
      Did they address the question of whether a werewolf is fully dog and fully man? 😛

      • Deiseach says:

        Did they address the question of whether a werewolf is fully dog and fully man? 😛

        Gerald did wonder about this (see chapter 19 of “The Wonders and Miracles of Ireland” in the linked document for the full story), and although he successfully dodged having to appear at the Synod to settle the question (he wrote a letter advising them to kick it up to the Pope), tried the usual recourse: when in doubt, appeal to St. Augustine (can’t appeal to Aquinas yet since he’s writing 37 years before St Thomas Aquinas will be born) 🙂

        It cannot be disputed, but must be believed with the most assured faith, that he divine nature assumed human nature for the salvation of the world; while in the present case, by no less a miracle, we find that at God’s bidding, to exhibit His power and righteous judgement, human nature assumed that of a wolf. But is such an animal to be called a brute or a man? A rational animal appears to be far above the level of a brute; but who will venture to assign a quadruped, which inclines to the earth, and is not a laughing animal, to the species of man? Again, if any one should slay this animal, would he be called a homicide? We reply, that divine miracles are not to be made the subjects of disputation by human reason, but to be admired. However, Augustine, in the 16th book of his Civitate Dei, chapter 8, in speaking of some monsters of the human race, born in the East, some of which had the heads of dogs, others had no heads at all, their eyes being placed in their breasts, and others had various deformities, raises the question whether these were really men, descended from the first parents of mankind. At last, he concludes, “We must think the same of them as we do of those monstrous births in the human species of which we often hear; and true reason declares that whatever answers to the definition of man, as a rational and moral animal, whatever be its form, is to be considered a man.” The same author, in the 18th book of the Civitate Dei, chapter 18, refers to the Arcadians who, chosen by lot, swam across a lake and were there changed into wolves, living with wild beasts of the same species in the deserts of that country. If, however, they did not devour human flesh, after nine years they swam back across the lake and re-assumed the human form.

        So the answer would be: despite the bestial outer form, werewolves are fully human, no admixture of dog 😀

      • Jaskologist says:

        Three Historical Accounts of the Dog-Headed Men // Marco Polo, Ibn Battuta and Sir John Mandeville

        Somewhere in the distant seas past India, these three famous explorers from completely different backgrounds came across a race of men called the Cynocephaly: a remarkable people with the “heads of dogs”. Though Sir John Mandeville’s account (and very existence) is heavily questioned by historians, quite what brought the otherwise reliable Marco Polo and Ibn Battuta to corroborate the ancient legends of the Dog-Headed Men is less clear…

        • Deiseach says:

          This was (allegedly) a question by a missionary going out to Foreign Parts about “what happens if I encounter some of these dog-heads, are they human, can I baptise them?” and if I’m remembering rightly, the advice was “okay, they are reported to wear clothing, so they can’t be mere animals, so if they have a sense of shame and modesty then they are rational and you’re okay” 🙂

          From Wikipedia on Cynocephaly:

          The ninth-century Frankish theologian Ratramnus wrote a letter, the Epistola de Cynocephalis, on whether the Cynocephali should be considered human (he thought that they were). If human, a Christian’s duty would be to preach the Gospels to them. If animals, and thus without souls, such would be pointless. Quoting St. Jerome, Thomas of Cantimpré corroborated the existence of Cynocephali in his Liber de Monstruosis Hominibus Orientis, xiv, (“Book of Monstrous men of the Orient”). The thirteenth-century encyclopedist Vincent of Beauvais acquainted his patron Saint Louis IX of France with “an animal with the head of the dog but with all other members of human appearance… Though he behaves like a man… and, when peaceful, he is tender like a man, when furious, he becomes cruel and retaliates on humankind”.

          Okay, it looks like I am thinking of the letter of Ratramnus to his friend Rimbert who was posing the question. I love the illustration here in this article of three dog-headed men discussing their sacks of grain 🙂

          In the 9th century a churchman called Rimbert – who later became the archbishop of Hamburg-Bremen – was planning to leave on a missionary journey to the northern reaches of Scandinavia. The idea of converting Scandinavia to Christianity had been enthusiastically pursued by the Emperor Louis the Pious and Archbishop Ebbo of Rheims in the 820s. In preparation for the journey Rimbert wrote to Ratramnus, a monk of Corbie in Picardy, asking for information regarding the dog-heads, whom he thought he might encounter. Ratramnus had been sent a dossier Rimbert had put together which informed him that the dog-heads lived in villages, practised agriculture and domesticated animals. In response Ratramnus wrote his Epistola de Cynocephalis a work which would answer the question of whether the dog-heads were worthy of evangelism. The issue hinged on whether the mysterious creatures could be considered rational.

          It’s good to know that if we ever do encounter aliens, the theological spadework has already been done 😀

          • thasvaddef says:

            If animals, and thus without souls, such would be pointless.

            No-one told St. Francis of Assisi!

  25. OptimalSolver says:

    The “Cucuteni-Trypillian culture” built the largest cities in history up to that date, then burned them down every 75 years or so, consistently, for centuries.

    Reminds me of Isaac Asimov’s Nightfall.

  26. Reasoner says:

    Reporters Without Borders has built The Uncensored Library on Minecraft, containing all the information banned by the repressive governments of the world.

    I suppose the closest American equivalent is Unz.com’s collection of books banned by Amazon?

  27. Lambert says:

    My guess is that some disease vector lived in abandoned houses.
    Maybe plague rats/fleas.

    • Le Maistre Chat says:

      Definitely not plague rats and their fleas per se. The brown and black rat species didn’t arrive in Europe until Late Antiquity or the Middle Ages. Latin pedants even said you should call those species mus because “rat” was a vulgar neologism.

      • tsutsifrutsi says:

        Maybe not the rats, but probably a previous carrier for the fleas. Yersinia Pestis has been found in remains from the Cucuteni–Trypillia culture, and so is implicated as a potential cause of the Neolithic Decline (i.e. the period when said culture went extinct, despite being a seemingly-prosperous early civilization that had spread throughout Eastern Europe.)

        • Le Maistre Chat says:

          Maybe not the rats, but probably a previous carrier for the fleas. Yersinia Pestis has been found in remains from the Cucuteni–Trypillia culture, and so is implicated as a potential cause of the Neolithic Decline

          Cool, thanks!

    • AC Harper says:

      That’s a very modern way of looking at possible reasons. Perhaps it is the purity/corruption feeling at work? Over time the house (or town) attracted ‘corruption’ (bad spirits, curses, unexplained deaths, full midden heaps, hauntings, failing construction materials, and, yes, disease) until the sanest option was to cleanse the house (town) with fire and start again.

  28. Mayfear-Writer says:

    Regarding the “burned house horizon”: Irish Travellers have a funeral rite in which, when a member of the clan dies, their caravan is burnt (as memorably depicted in the Gabriel Byrne film Into the West). The rite has persisted into the modern era, to the point that some settled Travellers living in one-storey houses have, in some instances, been known to do burn down houses after the resident’s passing (this happened in a Traveller housing estate just a few kilometres away from where I grew up).

    It’s been theorised that this rite may have emerged at a time when infectious diseases were very common, so that if a person died of a disease it made intuitive sense to burn all of their possessions in hopes of preventing the disease from spreading. Naturally due to cultural evolution, the rite has taken on a life of its own (so to speak).

    • theredsheep says:

      I’m pretty sure the Irish Travelers and the Romani (nee Gypsies) are two different ethnic groups with similar lifestyles.

      • Mayfear-Writer says:

        Thank you, corrected.

      • AlphaGamma says:

        Though the Irish Travellers have AFAIK adopted some of the Romani customs. For instance, they sometimes specify their caravans without toilets or sinks because of originally-Romani purity taboos. And I’ve certainly seen accounts of Romani funerals which involve caravan burning.

        So I’m not sure whether caravan burning was a Romani custom adopted by the Irish Travellers or evolved independently (it seems unlikely to be an Irish Traveller custom adopted by Romani, as Romani outside Britain and Ireland also do it). As practised by the Romani, it does seem to be related to their purity rules.

        • Deiseach says:

          The discussion of Irish Travellers is very fraught, because it is entangled with racism and a whole heck of a lot of bad history on both sides.

          Older generations of Travellers (who used to be referred to as “Itinerants” as a replacement for “Tinkers” because that was now adjudged to be a slur) up to, say the 60s and 70s, would have had many of those customs. They would have lived in caravans (some remaining horse-drawn ones but mostly replaced by motorised ones) by the side of the road in poor conditions, but tellingly often the conditions of settled people, especially those living in rural areas, were not hugely different: when I was a very young child, at times there were tinkers’ caravans camped under the railway bridge when they were on their travel circuits and both they and we got our water from the same pump in the field.

          Today, there is a huge divergence. Modern Travellers, particularly the younger ones, have fallen away from their traditions and are often living under the worst of both worlds, adopting bad customs from liberalised settled society while retaining poor elements of their own culture e.g. while traditional Traveller culture would be judged as (by modern liberal social standards) as sexist and conservative, things like out-of-wedlock births would not be tolerated. Today, there is a lot of marital separation, breakdown, single parents, etc. just like settled society!

          And the Siberian Mammoth in the room is, of course, criminality. I’m not going to address that because it is so dangerous to discuss and could get us all in real trouble.

          What I am going to say is that modern-day caravan and house-burning very probably has much less to do with Traveller cultural customs, and much more likely to do with – not to put a tooth in it – fraud and scams. Let me give you an example, naming no names and certainly making no allegations about anyone doing anything, is that clear?

          So, part of the local council’s remit is to operate halting sites and amenities for Travellers, and part of that is to help source and pay for caravans as necessary.

          It is alleged that sometimes scams are said to happen when it comes to this, e.g. the expensive new caravan paid for by the council will be sold on by the recipients and the money kept by them while they will continue to live in the old caravan.

          All I can do is relate an anecdote I was told. That the usual request was put in for a new caravan for a family that said their old one was no longer suitable. That this request was turned down. That an old and very poor condition caravan was witnessed, by workers who were on the halting site for another reason, being driven in and replacing a caravan which was then driven off.

          That the next day the office was informed that “Our caravan went on fire and is now all burned up, we need that new caravan we asked you for”. That the old and bad condition caravan wasn’t visibly present, that nobody could actually swear to which caravan it was that had mysteriously gone on fire when examining the remains of the burned one, and that suspicion was not enough, so the request was granted.

          That caravans and houses often mysteriously go on fire when requests for upgrades/better replacements are refused.

          And even sharing that anecdote is enough to get me into trouble, so I’ll say no more. Travellers do have a general bad reputation, much of that is prejudice from the settled community, Travellers do live in bad conditions and under hardships, the women are mainly the ones trying to improve things, and that it’s unfair to tar them all with the same brush of criminality.

          But on the other hand, there is definitely a bad element which does engage in crime and violence, and which does invoke the protection of activists and the soft-hearted amongst the rest of the community to respond with counter-accusations of racism and prejudice.

      • No One In Particular says:

        How are they “nee” Gypsies?

        • I think he means that they used to be called gypsies, are now often called Romani.

        • Deiseach says:

          “Gypsies” is now considered a derogatory term or even a slur, especially by the younger woke crowd. Back in 2013 when “Pacific Rim” came out, one of the Jaegers was called “Gipsy Danger” and some people insisted, when talking about it, as spelling it “G*psy Danger” or even re-naming it to “Lady Danger” so they would not be propagating a slur.

          I’ve also seen terms for Romany spelled several ways: Romany, Romani, Roma, Rom, Rroma, etc. (again, back in 2011-13 or so, there were a ton of kids on Tumblr claiming to be of Roma descent or heritage, even if it were most likely they were every bit as white as I was – “I had a Romani granny” was the then-equivalent of the “One of my ancestors was a Cherokee Princess” and involved a lot of lecturing about prejudice and not using slurs and so on).

        • No One In Particular says:

          But “nee” is French for “born”. It refers to the original term, not a previous term.

          • theredsheep says:

            It was a sloppy use of language, typed quickly with an eye towards conservation of keystrokes. I had no idea what the etymology of “nee” was.

          • littskad says:

            I don’t know French well enough to know about how “née” is used there, but etymology is not destiny. In current English usage, “née” just means “formerly called”.

          • mcpalenik says:

            In French, it means born (part tense).

    • DarkTigger says:

      Well, you know that would be a useful explanation, but those people did not only burn single houses.
      They took whole walled towns, hundered of houses, and burned them down.

      And not “there was a battle, and during the fight half of the town burned down”. Because a) there is oftened no evidence for a fight, and b) best explanation for the evidence we see is “they filled all the availabe space in that town with fuel, every house every street, about a mans hight. And then on a dry day with favoriable wind, they set the whole thing ablaze.”

      Also that culture was ended by one of the first pandemics we have evidence for. So if it was an measure against invectious diseases, it wasn’t very effective.

  29. Deiseach says:

    Unlike the Spanish conquistadors, who had the advantage of using guns while facing Stone Age empires, Afonso generally faced enemies as advanced (and sometimes more advanced) than himself. How did he do it?

    I couldn’t resist looking him up on Wikipedia since I was curious to know if he was/this was the same family that the city of Albuquerque was named after (short answer: no, it’s a Spanish duke) and the explanation seems to be in part that he had a good chunk of experience fighting such enemies before he started making his way from Africa to India:

    Afonso served 10 years in North Africa, where he gained military experience in fierce campaigns against Muslim powers and Ottoman Turks.

    In 1471, under the command of Afonso V of Portugal, he was present at the conquest of Tangier and Arzila in Morocco, serving there as an officer for some years. In 1476 he accompanied Prince John in wars against Castile, including the Battle of Toro. He participated in the campaign on the Italian peninsula in 1480 to rescue Ferdinand II of Aragon from the Ottoman invasion of Otranto that ended in victory. …In 1489 he returned to military campaigns in North Africa, as commander of defense in the Graciosa fortress, an island in the river Luco near the city of Larache, and in 1490 was part of the guard of King John II, returning to Arzila in 1495, where his younger brother Martim died fighting by his side.

    …[In] 1503, after a long military career and at a mature age, Afonso was sent on his first expedition to India together with his cousin Francisco de Albuquerque. Each commanded three ships, sailing with Duarte Pacheco Pereira and Nicolau Coelho. They engaged in several battles against the forces of the Zamorin of Calicut and succeeded in establishing the King of Cohin securely on his throne. In return, the King gave them permission to build the Portuguese fort Immanuel (Fort Kochi) and establish trade relations with Quilon (Coulão, Kollam). This laid the foundation for the eastern Portuguese Empire.

    So this isn’t some guy who just hopped on a boat and set sail for the exotic East, this is a seasoned and experienced military and diplomatic commander who has come into contact with non-European rulers before. He also seems to have had a refreshingly practical approach to problems:

    He arrived at Ormuz on 25 September and soon captured the city, which agreed to become a tributary state of the Portuguese king. … Afonso began building the Fort of Our Lady of Victory (later renamed Fort of Our Lady of the Conception), engaging his men of all ranks in the work.

    However, some of his officers revolted against the heavy work and climate and, claiming that Afonso was exceeding his orders, departed for India.

    (So he naturally raids the coast for supplies, heads off to India, is opposed by the Viceroy who has the disaffected officers on his side, and then goes and wins a huge naval battle for something to do while waiting for the political winds to blow in his favour):

    On 3 February 1509, Almeida fought the naval Battle of Diu against a joint fleet of Mamluks, Ottomans, the Zamorin of Calicut, and the Sultan of Gujarat, regarding it as personal revenge for the death of his son. His victory was decisive: the Ottomans and Mamluks abandoned the Indian Ocean, easing the way for Portuguese rule there for the next century.

    (The winds do eventually blow in his favour and our guy heads off to capture Goa, and when he’s done that he once again adopts the hands-on method of problem-solving):

    Albuquerque founded at Goa the Hospital Real de Goa or Royal Hospital of Goa, by the Church of Santa Catarina. Upon hearing that the doctors were extorting the sickly with excessive fees, Albuquerque summoned them, declaring that “You charge a physicians’ pay and don’t know what disease the men who serve our lord the King suffer from. Thus, I want to teach you what is it that they die from” and put them to work building the city walls all day till nightfall before releasing them.

  30. johan_larson says:

    Those of you who remember the cracked.com that was before its radical downsizing in 2017 may be interested in the podcast Quick Question with Soren and Daniel, a podcast by two of the better writers from that site, Soren Bowie and Daniel O’Brien. It’s a relaxed conversational podcast by two good friends who also happen to be professional comedy writers; Soren writes for American Dad, and Daniel for Last Week Tonight.

    https://qqwithsorenanddaniel.podbean.com/

  31. Ben J says:

    Typo in the “DrugsAnd.Me” link – I think the last sentence should be “…Asians might metabolize benzodiazepines unusually slowly and should start with half the usual dose?”

  32. Rachael says:

    The LW post about conflict/mistake theory and game theory was really interesting and thought-provoking.

    I’ve previously thought that the tradeoff between how big the pie is and how evenly the pie is shared out is the essence of the economic left/right axis (i.e. the left would be willing to accept a smaller pie if it meant it could be shared out more fairly, whereas the right would be willing to accept a more unequal distribution if it meant the pie could be bigger overall). Would you say that’s fair/accurate?

    But that in combination with the LW post would suggest that those on the economic left are conflict theorists and those on the economic right are mistake theorists, and I don’t think that’s accurate. (It miiight be true as a weak correlation, but I think the implication of the LW post plus my paragraph above would be that it’s universally or inherently true.) Scott’s original formulation of conflict/mistake theory was about whether you think your opponents are mistaken or immoral, whether you think their means or their ends are flawed – and I think both perspectives are found all over the political spectrum.

    • Wasserschwein says:

      I don’t think that that is accurate. Many on the right value fairness over aggregate production. The pie argument would be that if I bake a pie, it’s only fair that I get to decide what to do with it.

      My impression is that the liberal right is mainly driven by a conviction that the use of force is generally wrong. I think that consequentialist arguments are mostly used in an attempt to seem more objective and to be able to persuade those who have a more favourable view of the use of force in principle, rather than being anywhere near the core of the conviction itself.

      • thisheavenlyconjugation says:

        Who are the “liberal right”? I would say that the right are definitely more in favour of violence (in different ways for different shades of right). Unless by “liberal right” you mean libertarians and “force” you mean their definition of coercion.

        • Wasserschwein says:

          I’m referring to proponents of classical liberalism, as opposed to social liberalism on the one hand and conservatism, fascism, &c. on the other hand. When discussing their opinions I would be using their definitions of the terms.

    • Nikitis says:

      I don’t think that is accurate either. Someone on the left would reject the correlation between more inequality and a larger pie entirely. It’s a disagreement on facts, not on ends.

  33. Jarl Gertz says:

    DrugsAnd.Me is an exceptionally good site on recreational drugs

    It seems to lean a little bit… pro-drug?

    If I try LSD, will I experience LSD flashbacks?
    Simply not true. Scientists still don’t agree on with what HPPD and flashbacks are, but some studies suggest that the chances of developing these symptoms is around 4% of users.

    Seems like ‘Possibly!’ is the correct response to the above question?

    Will taking LSD make me mad and insane?
    The evidence to refute this myth is boundless. LSD is actually one of the safest drugs.

    And yet! https://slatestarcodex.com/2016/04/28/why-were-early-psychedelicists-so-weird/

    • Tarpitz says:

      I know someone who had to be institutionalised after a summer in which she took a lot of LSD, and still needs antipsychotics 15 years later (though she is pretty functional with them). I’m not saying don’t do LSD, but I am saying maybe be a bit sensible about how and how much you do LSD.

    • No One In Particular says:

      The second at least seems to be just pro-LSD. I mean, logically speaking, it’s equivalent to “almost every drug is even more dangerous than LSD”.

      • Jarl Gertz says:

        I guess I take more issue with the first part, since taking lots of LSD really does sometimes make people go insane.

    • RavenclawPrefect says:

      Also, this advice is slightly controversial:

      You should place the tab under your tongue for around 10 minutes, after that, you can safely swallow the tab.

      Sublingual absorption is faster, but riskier, depending on how much you trust your source. Real LSD will work if swallowed immediately, just with a slower onset, but some more dangerous drugs that may be marketed as LSD won’t, so swallowing minimizes your risk of having taken something less safe (which any other drug is likely to be). Not sure how likely this is to occur, but it seems to be the standard wisdom in the more detailed/cautious guides that I’ve seen.

  34. frygtbaeven says:

    Tangentially related to the Scunthorpe Problem: 4chan’s /pol/ had a relatively short lived campaign to transition to using “Google” as a replacement for the N-word, in the hopes that if they were successful Google would have to censor its own name.

    • Aapje says:

      That’s a pretty dumb idea. For it to work the word should be rare, yet something that they don’t want to censor, like ‘disney’.

  35. Scott says:

    The ‘Against Exaggerated Criticism of Dr. Seuss’ comment got deleted, so here’s a mirror.

  36. 9Lo0p says:

    RE: Venezuelan warship attacking a cruise ship

    This could be related to “Operation Gideon”, the failed coup attempt made by some former Green Berets and Venezuelan expats. The Maduro regime claims to have had intel on the coup attempt from the beginning which might be true because one of the leaders of Gideon was named in a raid by Colombian authorities 6 days before the sinking (this raid was related to organized crime in Colombia). The group behind Gideon seems to have had terrible OPSEC and even Tweeted about the operation while it was in progress. It seems to have been known that Gideon would be using small boats to land in Venezuela. The Venezuelan Coast Guard moved 2 interceptor boats to La Guaira, and Venezuelan security forces had temporarily blocked off the highway between La Guaira and Caracas with shipping containers. This indicates that details of the coup attempt may have been known to Maduro forces and measures were in place to try to stop it. This sheds new light on the actions of the crew of the vessel Naiguata; they were presumably on alert for a seaborne commando raid. The cruise ship Resolute regularly carried small boats that could potentially be used to land troops on shore. It could be reasonable for Naiguata to assume Resolute was involved in Gideon and attempt to board her or escort her to a Venezuelan port, and Naiguata’s aggressive approach resulted in the collision.

    A couple of sources:
    https://www.thedrive.com/the-war-zone/33322/breaking-down-the-absolutely-batshit-botched-coup-attempt-against-venezuelas-maduro
    https://www.bellingcat.com/news/2020/05/07/the-venezuela-silvercorp-usa-saga-keeps-getting-weirder/

    • keaswaran says:

      Wikipedia claims it is in fact part of Maduro’s response to a coup attempt by an American private military.

      https://en.wikipedia.org/wiki/Macuto_Bay_raid

      Apparently the only people more hilariously incompetent at military adventures than the Venezuelan government are the people who try to take it down. All you need to do is to actually hire a cruise ship for your plan.

    • John Schilling says:

      Your second cited source states, correctly, that there is no evidence that the Resolute was involved in Operation Gideon or any other paramilitary operation. And it would seem most unlikely that a German-owned, Portugese-flagged vessel with an established history of legitimate entertainment cruises would be used for any such purpose.

      • Nikitis says:

        It could be reasonable for Naiguata to assume Resolute was involved in Gideon and attempt to board her or escort her to a Venezuelan port, and Naiguata’s aggressive approach resulted in the collision.

        Quoting the relevant part. He’s not saying that the Resolute was involved. He’s saying it would be reasonable for the patrol boat’s captain to assume it was involved.

        • sp1 says:

          I’d also argue that it’s extremely reasonable to assume a “German-owned, Portugese-flagged vessel with an established history of legitimate entertainment cruises” would be used for such a purpose because that’s exactly the kind of vessel you’d use for such a purpose.

          If you’re trying to infiltrate a country for coup purposes there’s good reasons you wouldn’t want to fly commercially or charter a plane (too noticeably military / bulky equipment, no ability to disguise identities, can’t easily slip away from an airport without being searched or noticed, etc.).

          You could try entering overland from a neighboring country but that just moves the problem to either getting past security in one of those nations or having the cooperation of their government. There are obvious reasons why a neighboring government might not be keen to have even the appearance of their willingness to aid in a coup and if your coup-team isn’t backed by a larger government (i.e., the United States) then you likely don’t have the contacts and funds necessary to secure that permission anyway.

          Infiltrating via sea avoids these problems and also gives you the ability to land and strike from unexpected places. However, despite the shipping world being notoriously opaque there aren’t fleets of ships that would work well for this sort of thing sitting around waiting for hire. A luxury cruise vessel works well because it’s known to carry small boats, has an irregular sailing schedule, and a big wad of cash would be pretty tempting for a specialized cruise company that has recently been in serious financial trouble. These plotters weren’t great at opsec but another advantage of using this kind of transport is that the regime now has to wonder if every boat near their coast, down to the smallest fishing trawler, is problematic.

          I don’t think the Resolute was involved but if I’m in charge of a coastal patrol craft and I know that there are active coup attempts against my government then it’s reasonable to at least try and search her. When the Resolute didn’t immediately follow my instructions I’d get extra suspicious, possibly to the point of ramming to signal my seriousness. It’s not the patrol boat’s fault that they didn’t know about the ice hardened bow.

          • bullseye says:

            It’s not the patrol boat’s fault that they didn’t know about the ice hardened bow.

            Surely a boat ramming a much larger vessel would have damaged the boat more seriously than the ship, even without the ship’s armor? It’s my understanding that the idea was not to sink the Resolute but to push its bow in order to point it in the prefered direction. I guess they hit it harder than they intended.

            I find the rest of your post persuasive.

          • bean says:

            I’d also argue that it’s extremely reasonable to assume a “German-owned, Portugese-flagged vessel with an established history of legitimate entertainment cruises” would be used for such a purpose because that’s exactly the kind of vessel you’d use for such a purpose.

            No, it isn’t. The way you’d do it is the way the plotters actually did do it, with smaller, privately-owned boats operating from whatever nearby country you can sneak this under the nose of. It’s cheaper and a lot more secure.

            Even if you have suspicions, the way the Venezuelan Navy handled this was dreadful. I’ve looked a bit into the relevant international law, and under recent protocols to UNCLOS, there’s at least a vague case for boarding on the grounds of “we suspect you’re involved in a coup”. But that is not what Venezuela tried to do, and the law in question does not allow you to haul a ship off the high seas and into your territorial waters to do so. These are very different things, and given Venezuela’s international reputation, I don’t blame Resolute’s owners and crew one bit for declining to go along with them. At which point, the Venezuelans managed to get themselves rammed and sunk. This does not reassure me of their professionalism either.

        • John Schilling says:

          I’d also argue that it’s extremely reasonable to assume a “German-owned, Portugese-flagged vessel with an established history of legitimate entertainment cruises” would be used for such a purpose because that’s exactly the kind of vessel you’d use for such a purpose.

          Except for the slight problem that it’s a purpose the ship’s actual owners, operators, and supervising authorities would never ever this side of the Hadean glaciation allow their vessel to be used for. Reasons left as an exercise for the student.

          • sp1 says:

            How could you know the minds of the owners? Remember – this ship was only involved in an incident because it was out to sea. Where was it before getting underway? Buenos Aires, where it had been held due to debts for months following the cancellation of an in-progress tour cruise which was cancelled because they couldn’t pay for fuel. Does being German mean you’re impervious to the temptation of money?

            And again, I don’t even think they were involved. And assume you know none of that. You’re a patrol boat captain at night, with limited information, who has been warned about possible water-borne incursions at a time that the government is taking this seriously enough that they’re blocking highways with shipping containers. You see a foreign vessel with the physical capability of launching and recovering small boats. They claim they’re a mere cruise ship but have no passengers onboard. They’ve been drifting near your territory, supposedly to perform engine maintenance. Do you really refrain from searching that ship because there’s no way they’d be so unprofessional to allow themselves to be hired out by a group who wants to do adventure tourism in Venezuela?

          • John Schilling says:

            Does being German mean you’re impervious to the temptation of money?

            A fairly modest sum of money, that will likely get you thrown in jail by the German government and any legitimate maritime venture you might hope to run in the future run out of most every port in the developed world, unless your Portuguese captain goes along with the plan and your Bay-of-Pigs reject customers maintain perfect OPSEC? I don’t think that’s going to be a huge temptation to the average German businessman.

          • keaswaran says:

            I think one might reasonably suspect that the world of cruises is in a very unprecedented situation the past few months, given how many prominent incidents there are featuring the majority of people on a boat being infected with a disease and then refused port everywhere they go. One might suspect an unorthodox exit strategy given the uncertain future of the industry.

          • sp1 says:

            Coopting legitimate institutions by corrupting one or more of its agents, even at what would seem to be outsized risk to their personal and professional reputations, is a tale as old as time for criminal organizations, schemers, and intelligence agencies of all stripes. None of the people at Deutsche Bank, for example, should have risked the fines and damage to their careers in the finance industry to launder money but they did. This happens all the time in every industry. Shipping, coincidentally, is notoriously poorly regulated and involved in organized crime.

            My point isn’t that you could get any given ship for some clandestine mission or dodgy activities, it’s that you could certainly find someone in that world willing to turn a blind eye. Could I call up Royal Caribbean and propose chartering one of their ships for a quiet mining expedition in Venezuela (or some other flimsy cover story that provides enough plausible deniability for the owners)? Probably not. Could I get a ship from any specialty excursion company that is deeply, deeply in need of cash? I like my odds. Sure, it’s a risk, but so are coups and this is the least risky infiltration choice given my options.

        • baconbits9 says:

          I’d also argue that it’s extremely reasonable to assume a “German-owned, Portugese-flagged vessel with an established history of legitimate entertainment cruises” would be used for such a purpose because that’s exactly the kind of vessel you’d use for such a purpose.

          The problem with this logic is that now every boat is exactly the kind of boat you would use for such a purpose. US navel ship? Check. Former US navel ship that was sold and repurposed? Check. US ally former navel ship that was sold an repurposed? Check. Ship with absolutely no ties to US navy? Somehow also check. Ship capable of easily launching other smaller boats for an invasion? Check. How about ships not capable of easily launching smaller boats? They wouldn’t expect that one coming….

          • sp1 says:

            I must not have explained myself well if you took my argument to mean that it would be equally inconspicuous to rent a US destroyer and run it into the harbor nearest to Caracas at flank speed. A naval ship is a great idea if you have the government who operates it backing you up, which these people did not, and if you don’t care who sees you because you have overwhelming force, which these people also did not. If you have overwhelming force or international support you don’t have to care about your opponent’s ability to intercept you.

            A ship that was previously a military vessel and was repurposed to the point that it no longer appeared to be a military boat could work – and was, in fact, exactly what Castro did with the yacht Granma – but Venezuela has been anticipating some sort of violence for so long that anything vaguely military would be immediately stopped.

            You could indeed use another type of commercial ship – the whole point of using a non-military vessel is that it dramatically increases the search surface for the regime – but there is the pesky problem of it still needing to accomplish the mission. This type of ship – and again, I am not saying that they were in any way involved – is particularly good because it’s configured for routinely launching small boats, its crew would be good at it, these types of ships don’t have the regular schedules and deadlines of cargo vessels, and it isn’t flagged or owned by countries the regime suspect of plotting against it. That doesn’t mean you couldn’t use another commercial or pleasure type craft but depending on your requirements (size of the group, how far they have to travel, the effectiveness of the regime’s coastal surveillance and ability to respond, etc.) this is a particularly good one. I’m not sure how pointing out that because other things could also work, although not as well, invalidates that.

          • baconbits9 says:

            I must not have explained myself well if you took my argument to mean that it would be equally inconspicuous to rent a US destroyer and run it into the harbor nearest to Caracas at flank speed.

            No, my point is pretty simple. Was this a covert mission for the US? Lets look at the ship: does it have ties to the US government/military somehow? If yes then its evidence for it being a covert mission run by the US. If no, well that is exactly the type of ship you would use for a covert mission. This is classic conspiracy theory thinking (in the pejorative sense) any vessel that ended up in a collision like this could be turned and claimed evidence of US involvement.

          • sp1 says:

            You must have misunderstood, I never said or implied that this was a US covert mission because it wasn’t. I even specifically mentioned that a coup backed by the US could probably just cross a land border if they chose to do so.

            My points about the suitability of the ship were around its theoretical benefits for covertly landing (and mayyyyybe picking up, if necessary) a group and equipment where the group doesn’t have the backing of a government. Not some sort of US backed invasion armada. If you have government backing – especially if the government is willing to overtly support you – then you have significantly better options.

          • Deiseach says:

            To be fair, if your government is in a state of high alert, there is a confirmed attempted coup about to be underway, and you’re a naval officer, do you want to take the chance of “nah, I won’t bother checking this ship out because there’s no way it’s involved?”

            You’ll look pretty stupid if you let it slip by and it turns out to be involved, much more stupid than you’ll look if you try checking it out. Better to look a little stupid for stopping an innocuous cruise ship than a lot stupid for not stopping the coup ship.

            Though ramming is perhaps a bit over-zealous as a way of “let’s check this ship out” 🙂

          • bean says:

            Though ramming is perhaps a bit over-zealous as a way of “let’s check this ship out” 🙂

            There are rules for this sort of thing. You’re potentially allowed to board and check papers of a suspicious vessel outside of your territorial waters. This is not what happened. They asked Resolute to enter their territorial waters, which is rather like the police randomly coming up to you and asking to arrest you without a warrant or anything. It makes a huge difference legally, and it was entirely reasonable for Resolute to say “no, we’re actually going to go somewhere else”. They actually played along until it was obvious that the Venezuelans weren’t going to do the normal boarding. The ramming was just the icing on the cake of incompetence they’d been baking.

          • sp1 says:

            Yes, there are rules for how these things are done. There’s the Convention on the Law of the Sea which Venezuela is not a signatory to and the Convention on the High Seas, which they signed in the 1960s. I don’t know if the current government feels bound by the latter but as far as I know they haven’t repudiated it and their statements about the incident seem tailored to fall within its terms. The Convention on the High Seas specifically allows member states to seize pirate vessels wherever they’re found. Venezuela has claimed that the ship was engaged in pirate activity. It’s true that the convention generally requires boarding first but there are some good reasons why the Naiguatá may not have wanted to risk it, which we’ll cover.

            All of this is still purely academic, though, for two reasons:
            1) Venezuela was in a state of emergency and saw itself as fighting for its survival and it’s absurd to think they’d care at all for the details of international treaties. No country would.
            2) Portugal, where the Resolute was flagged, and Germany, its operator, both recognize the Guaidó opposition government, in whose name the coup was being carried out, as the legal government of Venezuela. If those countries already consider your regime illegitimate why on Earth should you worry about upholding international treaties with countries that don’t recognize you? We’re way past the point where anything other than raw power matters.

            But why not board the Resolute at sea?
            Boarding a non-compliant ship is a very high risk exercise. Boarding a non-compliant ship that you suspect – try to remember that the Naiguatá doesn’t know at this point the ship is innocent, they only know that their country is under attack by a group that you sincerely believe might have the backing of larger countries – is engaged in armed hostilities is near suicidal. You’re an underfunded, poorly armed patrol ship. You’ve fired warning shots at a ship that refuses to follow you back to port and you have reason to believe that ship has 100+ armed plotters/mercenaries. They could have heavy machine guns or even a crew served weapon.
            It’s midnight, you’re facing a non-compliant ship during a time your government is blocking highways, the coup has been publicly announced, and you have some pistols and maybe a couple of rifles among your poorly equipped, funded, and trained crew of around 30. Your patrol craft does have a couple heavier guns but although you suspect the ship you probably aren’t ready to start blasting away with those (assuming you have ammunition) and you might be shot to pieces anyway if you do. Do you:
            -Try to board knowing there’s a reasonable possibility that whoever’s on that ship can overpower your crew and kill you all?
            -Take them at their word and wander off, possibly robbing you of a hero moment on national TV and definitely landing you in jail if you’re wrong?
            -Escort them back to port where you can call on local police, army reinforcements, anything to allow you to surround it in port and force the surrender of anyone onboard? If they’re actually innocent, no harm done. If they are a heavily armed landing force and try to fight their way out then you’ve at least put your side in a better tactical position.

            I can tell you from experience that even a U.S. Navy destroyer wouldn’t try to board that ship given the risk unless there was absolutely no other choice, and they’d only do it because they have the overwhelming firepower to destroy her if the boarding party was fired on. They’d shadow and wait for more reinforcements. The Naiguatá didn’t have that choice.

            I’m not saying that Venezuela is blameless and I don’t support the Maduro government but they were behaving pretty reasonably given the circumstances and it’s how I would expect any other military to behave. Any notion that they should stop to consider international law is a failure to try to understand the mentality of the people involved.

          • bean says:

            Look at it from Resolute’s POV. They’re in international waters, and Venezuela has no authority to order them to go into port under any piece of international law I’m aware of. More than that, Venezuela is currently in chaos, and there’s a nonzero chance that they’ll get blamed for something. Far better to say “we’ll clear off” and go somewhere else, and to do whatever you can to get away.

            And you’re ignoring a fourth option the Venezuelans had. Say “OK, we’ll escort you out”, and keep an eye on them to make sure they don’t double back and start launching zodiacs full of commandos. This has the advantage of being legal, and lets you intervene if they do start actually attacking you.

            Ultimately, this is the police asking you to come down to the station, when you’re not even in their jurisdiction right now and you have a reasonable fear that they’ll stitch you up. Resolute was legally and morally in the right to do what she did. Naiguatá’s failure at the very basic skill of not running into other ships was legally probably piracy, and definitely stupid. I mocked the USN when they failed at that, and I like them a heck of a lot more.

          • sp1 says:

            I completely agree that the Resolute did nothing wrong and, with the knowledge they had, was acting rationally as well. The ship had just left Buenos Aires after months of being impounded. I don’t know if the crew was onboard during that time but it’s not uncommon for the company to be unable to repatriate them so they’d have to sit on the ship for months. When the Naiguatá showed up they probably thought, “Jesus, not again” and wanted to avoid going through another impounding. A ship without that history is more likely to follow Naiguatá into port. I also doubt they knew much about what was going on in Venezuela beyond “crazy stuff” and didn’t realize the danger they were in.

            My objection isn’t that the Resolute did something wrong, my objection is with the idea that the Venezuelans were acting completely outside the bounds of normal behavior or incompetently. They picked from a set of bad choices, did a risky maneuver (it’s abnormal but in no way unprecedented for a military vessel to try to nudge another ship in a preferred direction) and got bit by a very unexpected feature of Resolute. (That’s assuming that the ramming was intentional. I think it probably was, albeit probably faster than expected because of suction, but I know Venezuela disputes it.)

            I do think your law enforcement analogy confuses more than it reveals. Military action in a setting of active hostilities is nowhere near the standards of law enforcement. That’s why I don’t think escorting Resolute away from Venezuela would have worked. Law enforcement – or a nation with more capability to conduct coastal surveillance / interdiction – could think the vessel was suspicious but let them go with the mindset that they’ll get the bad guys next time. If you think you may have stumbled upon a group trying to kill your leader that’s not a good choice – the next time that group is spotted might be when they’re giving a victory speech in Caracas. The risk is just too high to let that ship leave without being boarded.

          • No One In Particular says:

            @Sp

            Venezuela was in a state of emergency and saw itself as fighting for its survival and it’s absurd to think they’d care at all for the details of international treaties.

            Correction: the Madura regime was in a state of emergency and saw itself as fighting for its survival. And it’s not absurd to expect a country to respect international law, even in an emergency. Nazi Germany didn’t o around ramming US-flagged ships willy-nilly prior to the US entering the war.

            @bean

            Naiguatá’s failure at the very basic skill of not running into other ships was legally probably piracy,

            Piracy applies to private ship. If a navy ship attacks another ship, that’s not piracy.

          • anonymousskimmer says:

            @No One In Particular

            Nazi Germany didn’t o around ramming US-flagged ships willy-nilly prior to the US entering the war.

            Wikipedia: “American warships escorting Allied convoys in the western Atlantic had several hostile encounters with U-boats. On 4 September, a German U-Boat attacked the destroyer USS Greer off Iceland. A week later Roosevelt ordered American warships to attack U-boats on sight. A U-boat shot up the USS Kearny as it escorted a British merchant convoy. The USS Reuben James was sunk by German submarine U-552 on 31 October 1941.[23] ”
            https://en.wikipedia.org/wiki/Military_history_of_the_United_States_during_World_War_II

            War wasn’t ‘declared’ between Germany and the US until December of 1941.

  37. tgb says:

    The article on “Choosing the Zero Point” mentioning the “bottomless pit” reminded me of the rationalist fanfic I wanted to write but never will. Know the Myst series of video games? In them, you write books that you can then enter. Some of those worlds have humans in them. So one character was going to devote their life to writing books with worlds full of terrible suffering that could be quickly averted (by, say, averting a climate crisis by introducing a new oxygen-releasing fungus). But then you have the question: do these worlds exist prior to writing the book or not? If not, then writing that book created that human suffering and alleviating their suffering wasn’t worth it. So that would be the second main character, who thought the first was committing the worst crime imaginable by causing billions to suffer. He should instead be writing books full of the most bountiful and wonderful worlds possible.

    But how can you resolve this? You can’t ask the inhabitants whether about how long they’ve existed since the writing of the book might have created their history and memories as well.

    You might be able to, thanks to an argument like the Doomsday Argument. Suppose all those human worlds do exist before their book is written. And the book-writing technology existed here on Earth for many centuries and millions of books had been written. But no one had ever encountered people from another world coming to Earth by writing a book that links here. So book-writing technology must be incredibly rare among worlds, since a book-writing civilization links to millions of human-containing worlds but Earth hasn’t been linked to so you expect that less than one in a million of human-containing worlds have book-writing technology. But this would be too special – you would be assuming that your culture is one-in-a-million. Since we have no reason to expect ourselves to be in a special civilization, we conclude that in-fact those other worlds did not exist until the books were written.

    And then I guess there’d be a sequel where they discuss what the implications of the “self-indication assumption” are.

    • Nisan says:

      If I recall correctly, the D’ni brought Book-writing technology to Earth 10,000 years ago. So from their perspective, the anthropic argument is sound and Writing creates worlds. But a human would have to conclude the opposite! You could make your two characters be D’ni and human, so they rationally have to disagree, despite having access to the same evidence.

      Or you could combine the two characters into one character of mixed human and D’ni ancestry, like Gehn and his descendants. They’d find themselves coming to different conclusions based on which of their ancestral civilizations they were taking the perspective of!

      • Paul Brinkley says:

        As far as I can tell, this is correct. The D’ni first linked to (a huge cavern beneath the surface of) Earth about 10000 years before Earth’s present day. That’s when they became the D’ni; before that, they were called the Ronay. All worlds exist in what they called the Great Tree of Possibilities (what comics call the multiverse), and a person can reach one of them by writing a Descriptive Book.

        This is all according to a site that only exists on the Wayback Machine now (drcsite.org), and was intended to show Earthlings’ attempts at figuring out what was going on, so it was deliberately sparse, similar to a historical account of Khafre’s Egypt or something. In turn, even the D’ni weren’t quite sure how The Art worked, so this was like an origin story of an origin story. The evidence is incomplete, and could support multiple accounts.

    • CatCube says:

      The question of “Is the author of a Book creating a world, or merely linking an already-existing one?” was the major plot point in the Myst novels, and IIRC sort of carried over into the games, but in a subtle way. This question was the big divide that drove apart Atrus (the main NPC you interact with in the first few games) and his father, Gehn, the villain of Riven. Gehn believed he was a god because his writing created Riven, and that he deserved to rule over its inhabitants–often cruelly–while Atrus believed in the linking theory and that therefore no such divine right existed.

  38. caryatis says:

    If De Pony Sum is reading this, please add page numbers.

  39. ConstantConstance says:

    Improvement in my lifetime: Sanitary napkins (though the name is still sort of stupid.) You used to have to wear a special belt with these weird clips or else wear special underpants with tabs to feed the ends of the napkin through. Now they have adhesive and stick to your underwear without too much difficulty. They’re also WAY less bulky than they were in 1977. I don’t know if tampons have improved much, but they probably have.

    • caryatis says:

      Not to mention menstrual cups and absorbant underwear.

    • Lord Nelson says:

      Someone needs to tell the schools to get with the times. I had to buy one from the dispenser machines in jr high (mid 2000s), because I stupidly forgot to restock my purse. What I got was just a bunch of cotton balls shoved together.

      It was a traumatic experience and also the biggest waste of 50 cents.

    • No One In Particular says:

      Sanitary napkins (though the name is still sort of stupid.)

      You could call them menstrual pads if you prefer.

  40. Ninety-Three says:

    I came across this because Michael Fumento, who helped calm the panic and debunk the rumors, is back in the news saying coronavirus won’t be a big deal – which makes me worry he’s less a heroic lone voice of reason, and a more a guy who just really likes dismissing diseases.

    It is a curious standard by which having been right in the past is considered disqualifying.

    • keaswaran says:

      This is what Wikipedia quotes him as saying about coronavirus:

      In a January 23, 2020, New York Post opinion article, “Don’t buy the media hype over the new China virus,” referring to concerns about coronavirus disease 2019 during the COVID-19 pandemic as “tinfoil-hat paranoia,” Fumento wrote that “there appears to be nothing very special about this outbreak of the 2019-nCoV or Wuhan virus… we have been through these hysterias before…. An infected American is reportedly doing well…. Wuhan will be a lot worse in China, simply because health care there is vastly inferior. It appears that, like flu, Wuhan usually kills through often treatable secondary infections. Well, treatable in the West… there is no evidence that Wuhan… is more contagious than influenza or spreads differently…. So if you want, buy a (probably worthless) surgical mask to play “twins” with those “courageous” TV newsmen.”[51][52][53] In a March 8, 2020 followup opinion article, he decried the “pure hysteria” about the virus,” noted that there had been only 19 deaths in the US, and maintained that “the spread of the virus continues to slow.”[54][55]

      Saying “there’s nothing special” about this new virus so that people in the west had nothing to worry about from China, and that it was slowing in the US at the time when there had been 19 deaths, seems to be evidence that he has gotten it vastly wrong this time. I think Scott’s point was that if we just looked at the AIDS discussion, we’d think he got things right. But now that we look at two data points, see that he made the same contrarian prediction about both, and was wrong about one, we should think that maybe he just likes this particular contrarian prediction, rather than being particularly right.

    • Scott Alexander says:

      It’s not that past rightness disqualifies present comments, it’s that present [seemingly bad] comments cast a new lens on past rightness.

    • AllAmericanBreakfast says:

      But was Fumento right even on HIV/AIDS?

      Over the decade from 1987-1990, there were about 6 million deaths from HIV/AIDS. ‘Bowen, who made the “plague” comparison here, estimated a death toll in the “tens of millions” by 1987, “if we can’t make progress.” The most charitable literal interpretation is that he estimated a minimum death count of 20 million in the worst-case scenario where no advances were made. AZT was approved that same year. Given that even with that obvious progress, the death toll was almost a third of Bowen’s lower bound, I think his prediction seems to have been accurate.

      I’m not able to find data of HIV prevalence among hetero vs. homosexual people, but this decade, 30% of cases are among heterosexuals. I believe this figure is specific to the US.

      According to UNAids, 30.6 million people were living with HIV/AIDS at the end of 1997.

      In 1999, over 5 million people were infected with HIV/AIDS. This is a rough calculation, but if there was an average of 2 million new cases per year from 1987-1997, and 5 million cases from 1997-2007, that’s 70 million new cases, making Bowen’s estimate of “50 million to 100 million people worldwide could become infected with the AIDS virus in the next two decades” accurate.

      I think that if Fumento was being a contrarian about Bowen’s estimates, he was wrong both about AIDS and about COVID-19. If he was just responding to overblown public panic about contact-transmitted or airborn AIDS, he was epistemically right (trivially; this was known for years already by experts) and successful in calming the panic.

      My takeaway is that if we’re evaluating an expert’s record on accuracy, it’s important to be able to identify and evaluate their precise predictions for ourselves. When we do this, we’re compiling our own list of topic-specific unofficial superforecasters. We shouldn’t rely on a reputation for “getting it right on AIDS,” because our culture doesn’t know the difference between epistemic accuracy and public messaging success.

      • Douglas Knight says:

        I don’t know what Fumento said in his 1990 book, but you can read his 1987 article. He is focused on the percentage of infections in non-IV heterosexuals. Bowen predicted 10% by 1992. Fumento attacks similar predictions as ungrounded, but doesn’t quite make a prediction himself. He attacks several ways of making such predictions. One is to say that it doubled from 2% to 4% and then extrapolating. However, the 2% and 4% refer to the same population, measured different ways, not change of time, so extrapolation is an error. There is a CDC report that predicted 9% in 1991, but he disputes it and seems to endorse (by not attacking) a parallel CDC report predicting an increase from 4% in 1986 to 5.3% in 1991. But even if we had a CDC report from 1992, there is the problem of interpretation: did it use the 2% method or the 4% method?

        ————

        As for Bowen, AZT was approved 2 months later, but it was in the news, indeed, available outside of trials (Fauci cameo) 4 months before his speech, so that doesn’t count as surprising progress. Also, I don’t think that patented AZT was available in most of the world before 1997, so it isn’t very relevant to his global prediction, which was probably mostly about Africa.

  41. Edward Scizorhands says:

    Related: r/TheMotte survey results on the MBTI test

    To save other people from searching, because none of the primary links describe it, https://en.wikipedia.org/wiki/Myers%E2%80%93Briggs_Type_Indicator

  42. Simon_Jester says:

    The big issue with the “just hire private tutors with tiny class sizes instead of spending the money on public schools idea” is… well. Several big issues. Now, mind you, I’m not saying these issues are insurmountable, but they are issues.

    1) Take the example of New York and the benchmark of “it would be cheaper to hire one 100K/year full time tutor for every five kids in the state than to run this school system.” The first question is, where do you find the tutors? You’d presumably fire and rehire-as-tutor every existing teacher in the state, but given that typical teacher:student ratios are in the general neighborhood of 1:20, you’re going to need to, oh… Somewhere between triple and quintuple the workforce directly engaged in educating children. Since ‘teacher’ is already a pretty common profession, this is going to have knock-on effects in the entire labor force, and it is a relevant concern whether there are even a large enough number of suitably educated individuals willing to do the job in the first place, even at the offered salary.

    2) What about the physical logistics? Are the parents all expected to drop off their kids with the tutors? Are the tutors driving around house-to-house and seeing each child for (40-N)/5 hours a week, where N is the number of work hours the tutor spends on the non-student-facing parts of the job like preparing lessons and driving to tutoring sessions? Is there subsidized transport infrastructure to make sure children and tutors can meet even if the parents can’t drop them off? Because it sounds like it’s either that, or a large fraction of American parents have to give a tutor a key to their home to enter it and teach their otherwise latchkey child in the mornings while they’re at work.

    3) Do we pay the tutors more for children no one in their right mind specifically wants to teach? Who negotiates the “your child is obtuse/obnoxious/chronically hyper-disorganized” premiums? Clearly a market can resolve this kind of thing, but the resolution will make a mockery of any idea of a standardized “we are paying all the tutors X dollars per year” system. One advantage of the current system is that your educational expenses don’t abruptly skyrocket by five thousand dollars a year because your teenager goes through a “is an obnoxious little punk” phase. Historically the solution to that problem was either “send my child to a rather brutal ‘reform school’ ” or “stop educating my child at all.” It is not clear that these were superior options.

    4) What about specialists? Schools frequently employ psychiatrists and paraprofessionals whose job is to provide services a lone tutor meeting a child in their house cannot.

    5) What about physical plant? Schools frequently provide children, including those who cannot afford it themselves, meals, library access, computer labs, and so forth. Again, this is something we COULD provide by a different mechanism

    By the time you’re done pricing these issues into the system, either the tutors are getting paid a lot less than 100K/year (probably to the point where you’re struggling to find educated tutors in adequate numbers if you weren’t already), or the overall system is costing the average taxpayer a lot more than expected when we suggested this.

    It’s like pointing out that for the price of a restaurant meal, you could easily buy far superior premium ingredients and pay the per hour wage of a short order cook of your own and have the meal made for you in your home. It sounds great… but in practice it doesn’t work very well and for some reason nobody who couldn’t afford their own mansion tries this in modern times.

    • keaswaran says:

      I was going to say something like this, but phrase it somewhat differently. A tutor is cheaper than a public school if you don’t count the costs of the real estate where the child meets the tutor, and all the other costs subsidized by the general lifestyle of the kind of people that hire tutors, but that the school department has to officially budget for. Obviously if you just count the cost of paying the educators, the public school comes out far lower cost per child than the tutors. It’s only once you add in all the hidden costs of education that the school district becomes more expensive.

      • Matt M says:

        I’m pretty sure that if I were to try and calculate the actual costs associated with feeding and providing an extra 20 square feet of space (my guess is the sqft/student ratio in most schools is lower than this) to a small child, it wouldn’t move the needle on this equation.

        • keaswaran says:

          If it costs $24,000 per student to educate children in the New York City school system, and the student-teacher ratio is 13 to 1, then the expenditures seem to be over $300,000 per teacher. Since I don’t think the average teacher makes half that, I expect that all the other costs beyond the educators do in fact “move the needle” on this equation.

          If you want to hire tutors for your kids, then you either need an extra room in the house for tutoring, or the tutor needs an office somewhere. If the tutor is going to tutor several kids, then the tutor will either need to hire administrators to coordinate schedules, or will need to spend a substantial fraction of the day doing this coordination. I don’t know what other hidden costs there are associated with doing schooling at home, but the school district is certainly spending its money on *something*.

          • Matt M says:

            The school district is spending its money on a veritable army of administrators to do all sorts of things that contribute highly to cost disease and virtually nothing to actually educating children.

            It isn’t spending all its money on food and real estate.

            The point is that most of the stuff it spends money on isn’t actually necessary. The costs are “hidden” specifically because they are unnecessary bloat.

          • albatross11 says:

            It seems like your argument here proves too much. Suppose we lived in an alternate world where the NYC school system spend $240,000 per student–wouldn’t your argument ead to the conclusion in that world that parents must spend about $240,000 to educate their kids with private tutors?

            I think the whole point of the discussion is the suspicion that the NYC public school system wastes a lot of money–that they spend a lot more money than is needed to provide the education that’s their alleged mission to provide. And while I don’t think “replace the public schools with private tutors” is an obvious slam-dunk replacement for the school system, it’s probably useful to get a sense of how much waste there is in the system, or alternatively, how much the line-item of education in the budget is really covering providing free babysitting and recreation, providing various social services, etc.

            One place to look for comparison here would be test prep schools. Does anyone know how much it costs per hour to send your kid to, say, a good SAT prep school in NYC? That’s a competitive business with few credential requirements, so it’s probably a good guess about the actual cost of a tutor and a workspace to teach in. Anyone know what kind of hourly rates they’re charging? Another place to look might be private music lessons, or sessions with a personal trainer.

          • The $24,000 probably does not include an imputed rental cost for the school building, since that belongs to the city.

          • keaswaran says:

            That’s right – to evaluate this properly we’d need to see what functions the school system is actually serving and what resources are needed for that. But this all seems like a Chesterton’s fence. I would think the burden of proof is on someone who claims that we could replace the school system with a similar army of tutors for lower cost at no loss to argue specifically for that no loss point.

            (I hadn’t thought about the fact that the real estate probably isn’t included as a cost here, even though it obviously is one, not least because it’s not taxed.)

          • albatross11 says:

            One fairly simple way to experiment around the edges with this is to phase in vouchers, maybe to 5% of the households per year chosen at random. The voucher allows you to stay at your public school or move to any other school that will take you in exchange for the $24K of yearly education budget.

          • Simon_Jester says:

            @Matt M

            Why don’t we start by specifying which things the school district is wasting money on, and targeting those as a waste of money?

            I mean, I could start off the top of my head with:

            1) Paying licensing fees for too many different digital platforms that do the same thing, more than any one teacher can feasibly use.

            2) Extensive/intrusive standardized testing regimens, which are expensive both in terms of manpower and in terms of lost instructional time.

            @keaswaram

            The city isn’t paying real estate taxes, but it IS paying upkeep on the building and any associated facilities.

            @albatros11

            One fairly simple way to experiment around the edges with this is to phase in vouchers, maybe to 5% of the households per year chosen at random. The voucher allows you to stay at your public school or move to any other school that will take you in exchange for the $24K of yearly education budget.

            Complication: As I discussed, what about the “your child is a jackass” tax?

            I think back to the thousand or so students I’ve had, in a district that spends something like $16,000 per capita on them. And I ask myself “If I were a private school principal working on a for profit basis, would I take this child for that sum of money?”

            The answer is usually ‘yes,’ but sometimes ‘no.’ Either because successfully educating that child will take additional resources that make them personally unprofitable to teach… Or because they’re such a fool or such a punk that I wouldn’t willingly allow them into my institution at any price if I could help it.

            But then the argument reduces to “schools would run more smoothly if they were free to refuse entry to the students they deem likely to be troublemakers or unable to keep up academically.” Which is true, but then why are we talking about defunding the public schools instead of simply letting them do that?

            If the argument is “the government/public would never allow public schools to simply kick out troublemakers and those who cannot keep up with the coursework,” then that invites the question of why we should either:

            1) Expect the government/public to allow an end-run around the system via private schooling to have the same effect, OR

            2) Arguably, why we should even be trying so hard to end-run around what seems in broad like a popular mandate for universal education, even for fools and jackasses.

            Now, there are other responses to the problem of “what about fools and jackasses.” But all the ones I’ve ever been able to think of- and it is not a matter I think of seldom- are things we could equally well have the public schools do, if only the popular support were there to do it at all in any capacity.

            Until then, vouchers just mean that private schools keep taking on the kids who are relatively easy to work with, while the public schools keep the residuum. Which in turn means [i]higher[/i] per capita expenses at the schools and lower outcomes, which in turn means the same people questioning their utility now will be questioning them harder afterwards.

          • mtl1882 says:

            Until then, vouchers just mean that private schools keep taking on the kids who are relatively easy to work with, while the public schools keep the residuum.

            Yeah. I don’t believe we can meaningfully address much else until we bring ourselves to acknowledge and hash this part out.

          • Thomas Jorgensen says:

            .. Most actual solutions to this are, however, either long term or goddamn expensive. Because consider, why is the child a jackass?

            1: Brain damage, due to lead (or equivalent) – This is very cheap to fix compared to the economic damage it averts. Near as I can tell, every country on earth, possibly excepting Norway is way underspending on lead abatement. It is also going to take at least a decade for any effects to show up in classrooms… Which is why everyone underspends. That is two election cycles out. How to fix… I dunno, public awareness campaigns until the public pressures the politicians into doing more?

            2: Malnutrition. Even parents who love their children and do their best do not necessarily have a clue how to feed them right. I suppose we could just go to a system where every child gets breakfast and lunch on the public dime (It has to be universal, because a program aimed only at the poor would inevitably suck). This is going to involve some sticker shock, though really, it is just moving money from parents grocery budgets to their tax bill, and sucks to be you if you are lifelong childless. France does this, and it might explain a fair bit about why France has such good health stats, but pisa ranks it right square in the middle, which.. honestly confuses me? Is some other aspect of French education terrible? Because an intervention that shows up in public health stats really damn well ought to do some good for learning!

            3: Abusive household. Yhea. What the hell is the school supposed to do about that? Have a system of boarding schools you can ship kids of to to get them out of hell? What about when parents turn down that offer? Do it against their will? That is going to go over well. Not.

            4: Just something wrong with the kid, physically, psychologically. More school head shrinkers?

          • mtl1882 says:

            .. Most actual solutions to this are, however, either long term or goddamn expensive. Because consider, why is the child a jackass?

            Well, one way of dealing with it is the traditional way of keeping most in public schools (no vouchers), which provides a place for the jackass, but doesn’t become a school centered around the jackass. Sure, he or she causes some disruption for other students, which sucks, but it may be manageable enough. But if students start leaving in large numbers, there is a precipitous race to the bottom.

            Let’s say for a second that by jackass we mean obnoxious and not violent or anything totally out of control. So you’ve got various levels and degrees of obnoxiousness left, and the behavior of more marginal ones almost certainly gets vastly worse. There’s almost no chance that any of them might eventually mature and straighten out, because they’re no longer really a part of functional society.

            The public school system becomes an asylum. We could agree that we want to do this—there are people who probably see this as the most realistic option. But that’s definitely not the narrative many Americans buy into about public schools, and would require some convincing. The fact that there isn’t an inexpensive way to do it would help with that. But it can also be handled by refusing to stratify the student body in the first place, which has its upsides and downsides. Or by not requiring everyone to go to school. Or by hoping some charter evolves to compete with the obnoxious.

            As for why, while I think all the things you listed play a role, I believe a lot of people just have obnoxious personalities, or at least go through a phase where they do.

          • John Schilling says:

            But then the argument reduces to “schools would run more smoothly if they were free to refuse entry to the students they deem likely to be troublemakers or unable to keep up academically.” Which is true, but then why are we talking about defunding the public schools instead of simply letting them do that?

            Because “Public schools must provide equal (and not separate-but-equal) education to everybody” is too strongly defensible a position, whose defenders will include parents and other advocates of obnoxious punks as well as people tactically allying with such.

            “Nobody can achieve unequal education by opting out of public schooling” and “Private schools cannot be subsidized with public money” are in principle also strongly defensible positions, but the first never took hold in the United States and the second is in the process of falling.

            Everything else, like how good public schools have to be and how well-funded and what sort of private-school subsidies are allowable, is a quantitative debate with no Schelling points to organize a defense around, no hills to die on, just an infinite featureless plain to be pushed back on.

            There are an awful lot of people, probably including a vast majority of parents of school-age children, whose first priority is to create a system where the vast majority of children can have a real education not crippled by the presence of obnoxious bullies in their classrooms. The path to achieving that goal on the private-school side, involves some mopping-up on the vouchers front, and then an advance across the infinite featureless plain. The path to achieving that by having public schools throw out the obnoxious jerks, involves taking a heavily-fortified hill defended by the parents of all the obnoxious jerks, and everyone else who fears being next on the list of people denied truly equal public education. So the people who just want to make sure their kids get a decent education, and the people who want most kids to get a decent education and have given up on “all” in despair, will likely find private-school vouchers easier and more attractive, and the people still devoted to truly equal education for all will likely be fighting the harder battle with few allies.

          • Edward Scizorhands says:

            It’s not necessarily a given that private schools will reject hard kids.

            Neither my nephew nor my son are the most pleasant kids to have in your classroom. They aren’t violent, but they resist a lot of traditional methods of teaching.

            My brother found a private school for his son, and it wasn’t some ultra-expensive elite school willing to tolerate jackasses in exchange for alumni dollars down the road. They just had more flexibility and manpower to deal with him, freed from public school rules.

            OTOH, public schools ended up working better for my son. But a friend of ours had a horrible time getting his son to fit in the same public school system, which refused to adapt to his special needs. They ended up moving for a job, and serendipitously the new school system was awesome for his needs; he went from barely passing (or less) to excelling.

            People need to be given room to explore and find options. “Everyone must fit into the round holes in my prescribed solution” tends to harm lots of square pegs.

      • Douglas Knight says:

        if you don’t count the costs of the real estate where the child meets the tutor

        The school budget doesn’t count that cost either. It does count the cost of janitors. A very concrete proposal is to fire 90% of the administrators and reuse the schools.

    • John Schilling says:

      One advantage of the current system is that your educational expenses don’t abruptly skyrocket by five thousand dollars a year because your teenager goes through a “is an obnoxious little punk” phase.

      One disadvantage of the current system is that everyone else shares that cost because your teenager is an obnoxious little punk, and twenty other kids basically have to give up on formal education until your obnoxious little punk grows out of it.

      Historically the solution to that problem was either “send my child to a rather brutal ‘reform school’ ” or “stop educating my child at all.” It is not clear that these were superior options.

      It is very abundantly clear to me that brutal reform school for the obnoxious punks and real education for everyone else, is superior to the current system.

      • NostalgiaForInfinity says:

        It is very abundantly clear to me that brutal reform school for the obnoxious punks and real education for everyone else, is superior to the current system.

        I’m a leftist and I tend to (reluctantly) support this idea. I went to a comprehensive (non-selective state school) and it’s remarkable how much a lesson can be derailed by just a few students. I’m not sure it’s great in practice for kids who get sent to the brutal reform schools, but they didn’t do all that well in the mainstream schools either.

      • Fahundo says:

        It’s not abundantly clear to me that the lesson that would have been taught if not interrupted by the punk kids is worth the cost of having the brutal reform schools, or that public education is doing much more than providing day care.

        • John Schilling says:

          I’m pretty sure public education is mostly just providing day care because public education is constrained to include obnoxious punks in almost every class and it is nigh impossible to do actual education when there’s more than one obnoxious punk in the room.

          • Fahundo says:

            I’ve been in enough classes where no one interrupted anything to know that isn’t the only reason.

          • Randy M says:

            It is a big reason. The inability to cater to 30 different levels of comprehension is another one. And the pacing needed to make sure all the students are on track may not be allowed by the amount of required course content.
            But being able to remove the obnoxious punks would solve a lot of the problems in a lot of class rooms, and is the low hanging fruit. Better sorting would help on some of the remainder.

          • Fahundo says:

            How many high school teachers have you met though? In a completely quiet classroom most are content to just say “read the next chapter from the book” or some shit like that. I could do that on my own at home, if I cared to.

          • Randy M says:

            I haven’t discussed the issue with every teacher I know, but I did have time to think about it while pulling my hair out and wondering why my life was so useless during the time I taught high school science for a year.

          • In a completely quiet classroom most are content to just say “read the next chapter from the book” or some shit like that.

            You remind me of Adam Smith’s description of teaching in his day:

            If the teacher happens to be a man of sense, it must be an unpleasant thing to him to be conscious, while he is lecturing his students, that he is either speaking or reading nonsense, or what is very little better than nonsense. It must, too, be unpleasant to him to observe that the greater part of his students desert his lectures, or perhaps attend upon them with plain enough marks of neglect, contempt, and derision. If he is obliged, therefore, to give a certain number of lectures, these motives alone, without any other interest, might dispose him to take some pains to give tolerably good ones. Several different expedients, however, may be fallen upon which will effectually blunt the edge of all those incitements to diligence. The teacher, instead of explaining to his pupils himself the science in which he proposes to instruct them, may read some book upon it; and if this book is written in a foreign and dead language, by interpreting it to them into their own; or, what would give him still less trouble, by making them interpret it to him, and by now and then making an occasional remark upon it, he may flatter himself that he is giving a lecture. The slightest degree of knowledge and application will enable him to do this without exposing himself to contempt or derision, or saying anything that is really foolish, absurd, or ridiculous. The discipline of the college, at the same time, may enable him to force all his pupils to the most regular attendance upon this sham lecture, and to maintain the most decent and respectful behaviour during the whole time of the performance.

            The discipline of colleges and universities is in general contrived, not for the benefit of the students, but for the interest, or more properly speaking, for the ease of the masters. Its object is, in all cases, to maintain the authority of the master, and whether he neglects or performs his duty, to oblige the students in all cases to behave to him, as if he performed it with the greatest diligence and ability. It seems to presume perfect wisdom and virtue in the one order, and the greatest weakness and folly in the other. Where the masters, however, really perform their duty, there are no examples, I believe, that the greater part of the students ever neglect theirs. No discipline is ever requisite to force attendance upon lectures which are really worth the attending, as is well known wherever any such lectures are given. Force and restraint may, no doubt, be in some degree requisite in order to oblige children, or very young boys, to attend to those parts of education which it is thought necessary for them to acquire during that early period of life; but after twelve or thirteen years of age, provided the master does his duty, force or restraint can scarce ever be necessary to carry on any part of education.

            He doesn’t specify a university in that passage, but earlier remarks that:

            In the university of Oxford, the greater part of the public professors have, for these many years, given up altogether even the pretence of teaching.

          • Deiseach says:

            How many high school teachers have you met though? In a completely quiet classroom most are content to just say “read the next chapter from the book” or some shit like that.

            I worked in a secondary school and if the teachers had been like that, they would have been reprimanded by the principal. Maybe if they’re supervising a free class or study period, they tell the students “be quiet and read the next chapter”, but teaching a subject class? No way.

            (I was even, for my sins, way back in my 20s once hired for a couple of weeks as a substitute teacher-lite by my old school even though I am totally unsuited with no teaching qualifications, because I had a science diploma qualification, and while supervising free classes I did do the “read quietly the next chapter of the text book”, I did ‘teach’ science classes including doing the practical lab classes, as well as some religion ones).

      • Simon_Jester says:

        I actually agree with you in broad about the need for separate track schools to deal with students who present disciplinary problems… with the caveat that most of the punks need psychiatric care and not beatings, whereas historically the response was more likely to give them beatings.

        The catch is, this isn’t an inherent problem with public schools. Public schools used to coexist, and in some places still do, with exactly the kind of institutions that serve the purpose you describe. They could again, if there were a popular mandate for it.

        But as long as there is no such popular mandate, it doesn’t matter very much whether we start privatizing schools or not; we’ll still have the problem.

        • John Schilling says:

          Private schools generally get to choose whether or not to accept any particular student, and so long as that is the case only the ones specifically advertising themselves as “we’ll whip your obnoxious punk into shape” academies will accept the obnoxious punks. By evaporative cooling, the vestigial public schools will become de facto reform schools.

          By the time private schooling becomes the norm, this solves the problem of the median student being stuck in a classroom with 2+ obnoxious punks and thus no possibility of real education. It does, unfortunately, raise the new problem of a class-based educational system where parents too poor to afford any private school and too busy to homeschool, will have to send even their diligent and studious students to a de facto reform school.

          Whether the new problem is better or worse than the old depends on whether you prioritize education or equality. If the former, “most but not all students are educated” is preferable to “everybody equally uneducated”. If the latter, this is reversed.

          • It does, unfortunately, raise the new problem of a class-based educational system where parents too poor to afford any private school and too busy to homeschool, will have to send even their diligent and studious students to a de facto reform school.

            Not if the private schooling is being supported by vouchers. At the per student amount presently spent by the public schools, there should be no shortage of private schools happy to take students who are diligent, studious, and poor.

        • mtl1882 says:

          I actually agree with you in broad about the need for separate track schools to deal with students who present disciplinary problems… with the caveat that most of the punks need psychiatric care and not beatings, whereas historically the response was more likely to give them beatings.

          If you go back a little further, historically the response was really more “don’t make them go to school.” In my experience, most kids who present disciplinary problems don’t need psychiatric care. Some definitely do, and most of the worst cases do. But most are basically restless. As someone who did well in school, and has worked in education, I can’t escape the conclusion that mass education is really just not appropriate for some significant number of kids, who cannot focus and behave so compliantly for such a long period of time.

          And we’ve made it harder and harder to get away with exiting the system, and tolerate very little misbehavior—I’ve volunteered in a public elementary school where it was treated as a major crisis that 7 year olds were sometimes disorganized and lost homework, or talked in the hallways. Like it was some unheard of pathology. There was no emergency: they were children. They’ll probably figure it out eventually with some assistance. Some will never be masters of paperwork organization, and that is okay. If they get their assignments in, great. If they don’t, maybe they should pick a career that doesn’t require a lot of that. What shouldn’t happen is making sure they feel maximal shame at age 8 because they didn’t put something in the right folder. I don’t have a good way to fix this, but it’s just not a reasonable system. I think it is incorrect to assume the average child is a model student and anyone who deviates has a problem. Kids are at all different maturity levels at that age, and it’s not that big of a deal. The anxiety over it seems to me related to the fact that public schools feel like they have to indicate a serious commitment to universal excellence, but it’s totally dysfunctional. As a standardized exam tutor, I work with a lot of young people from all different schools. The private schools generally seemed healthier to me for a long time—more chill. Now they seem to be reenacting the weird behavior I saw at public schools, especially elementary and middle, to prove sufficient competitiveness and zero-tolerance of everything.

          I’ve had mixed experiences with charters, but basically they seem to be more chill because they believe that with freedom from regulations, they can reach excellence in non-traditional ways. I support this philosophy, but my experience has been offputting. At a liberal suburban charter school I taught at, which emphasized more holistic learning, the kid seemed pretty happy and some were naturally bright but had no discipline. They were baffled by the entire concept of taking a test. They could not systematically learn material, sit still for 20 minutes, or follow instructions. When they reach college, they will be doomed. It might work for kids who don’t have to enter a systematic environment, and there are definitely some kids who simply are not built for such environments but who are intelligent and can probably find some sort of career. My experience with a competitive inner-city achievement-oriented charter was creepy. I appreciated what they were trying to do, but the entire thing was so corporate and career-oriented and condescending. They were mostly trying to build up the kid’s resumes, and these were impressive kids, but it was all very cookie-cutter in a way that didn’t work. It was all about style over substance, and, even with support, the backgrounds of many of the kids didn’t lend themselves to work environments where corporate norms about “presentation” are everything. But they definitely had the potential for a good career in a more flexible environment, if their substantive talents were brought out. And it appeared to me they had overwhelmingly selected for the most agreeable kids, rather than that the environment had made that much of a difference. I’m glad those kids had the opportunity to go to that school, and they will probably do pretty decently due to the emphasis on career placement and their own positive traits, but it’s not a model that’s going to fix anything on a larger level.

          If I ever have kids, my instinct now is to homeschool them. I have found that risks similar issues as the liberal charter school, though, in that many kids simply don’t pick up systematic habits. Mass education, for better or worse, teaches formulaic obedience to somewhat arbitrary instructions in a way that gets through to even the most undisciplined kids, and a lot of our society requires this. I’m sure there are fantastic homeschooling parents who are able to teach these things effectively and in a healthy manner, but I worry about it if I’m the teacher. Because I’m not naturally organized, I think school and the nagging of very organized parents were necessary to make me functional in the real world.

          Education policy strikes fundamentally at people’s values and ideas about life, and most people try to obscure this, for the understandable reason that it may not be something we can really talk about as a society. Separate tracks require admitting that you believe some kids are unreformable or not capable of getting to a certain point. Many would call this basic realism, but there is a huge segment of the public that needs to believe every kid can succeed if we just do a few things. And no doubt, there are concerns with tracking when you have such a low tolerance for misbehavior. This implicates a lot of kids from “respectable” families. A lot of these restless kids will get by okay or eventually get over it if kept within the regular system. If they are stigmatized early, they probably will get worse. There is a sensible way to deal with education if ideals of perfection are given up, and other emotional narratives lose some sway. Other countries are able to handle it. But we have to majorly broaden the conversation and be honest about value disputes, instead of trying to frame everything within idealized narratives or pretending it’s solely an individual issue. The entire system is predicated on the assumption that it is not an individual issue.

          On top of that, I think norms regarding careers and higher education are beginning a major shift, and we will have to talk about what exactly it is we’re preparing kids for, and that there can be a variety of answers to that.

          • Edward Scizorhands says:

            I like this comment.

            One issue with tracking is that, in many ways, it locks kids into a track. When, instead it should just be a way of schooling that is adapted to what the kid is good at and what the kid needs help with. Switching tracks should be relatively straightforward if we realize that the kid wants/needs something else. But if you have been leveling up along one class path for 3 years, you can’t just switch another class path with your exact age peers.

            That gets to another major problem with the schooling system. It is set up to, first and foremost, be easy to manage, for obvious reasons that it is set up by the people managing it. Like a command economy, it seems to work for a little while, but lots of pieces start falling apart.

            Has anyone done a serious attempt at abandoning the concept of grade-levels? It wouldn’t be such a hideous idea to switch someone from grade 9-track 1 to grade 9-track 2 if we didn’t worry about the concept of “grade 9.” Kids would take classes with people of similar starting knowledge/skill, similar knowledge/skill end goals, and similar style needs, and if we think those style needs start to change, we could change that for one/some of their classes.

          • Has anyone done a serious attempt at abandoning the concept of grade-levels?

            The unschooling approach, as in Sudbury Valley School, doesn’t have grade levels.

            One problem with the standard age segregated model is that it groups kids of very different intellectual ages. Another is that it puts kids into an environment where they are direct competitors. A twelve year old doesn’t have to prove that he is bigger and stronger than a nine year old, but he may have to prove that he is bigger and stronger than another twelve year old.

            It’s also, of course, a very unrealistic model of future environments.

          • Simon_Jester says:

            Anecdotally, I know two graduates of unschooling systems (including one whose unschool-school explicitly advertised use of a Sudbury Valley model).

            One is grossly unsatisfied with the education they got because they weren’t told to study things they now wish they’d been told to study.

            The other should have been thus unsatisfied, because they were appallingly ignorant of facts that no one with their IQ would have been ignorant of, had they attended the local public schools.

    • MilesM says:

      It’s not really a great analogy. 90% of kids go to public schools, but 80% of meals are prepared at home.

      If the average family bought 90% of their food (either directly, or delivered by government programs paid for by taxes) at current restaurant prices, the idea of making alternate arrangements wouldn’t sound so outlandish.

      • Simon_Jester says:

        And that would work pretty well for food because most people’s food preparation costs are broadly comparable. Lots of people need to have (or avoid) specific kinds of food, but not a lot of people need special cooking techniques that cost three times more per capita plus regular interventions by a college-educated specialist.

        Also, most people don’t have their cooking routinely tested at their own expense by outside experts who get paid to perform rigorous blinded taste-tests.

        I’m not saying the desire to save money on schooling is illegitimate, but I think that public school reform is a better path to that objective than privatization. The only reason privatization seems to work is because we turn our back and let the private schools do things that public schools would get sued into smoking craters for doing, in the name of being more cost-effective.

        • albatross11 says:

          Simon Jester:

          I definitely see your point. To the extent that private schools are a dodge to allow people to put their kids in schools that are allowed to keep order, track kids by ability, and kick out the disruptive kids, it would probably be cheaper to just allow the public schools to do those things.

          On the other hand, if I live in a school district where the public schools can’t or won’t do those things, it sucks for my kid to have to keep going to a school where they can’t do a proper job running the school for political reasons. Many places seem to have spent decades not being able to run a decent school system, during which whole generations of kids get sent to crappy dysfunctional schools.

          It also seems nuts to me that 30% or more of your home’s value depends on the school district, which happens here. And I think creating an incentive for everyone with any choices to leave the city as soon as they have kids, because only a madman would send their kids to the local DC public school system, is similarly nuts. But having whole generations that don’t get decent schools and crazy school district:house price interactions and people having to move as soon as they have kids so they don’t have to send their kids to a dysfunctional school–all that’s a *policy choice*. I sure don’t see why it’s a policy choice we need to keep making.

        • MilesM says:

          I already said it was a bad analogy, I don’t need you to explain why it’s bad…

    • albatross11 says:

      The tutor thing is a thought experiment to clarify that we spend a surprising amount of money per student in public schools, not a fleshed-out proposal to replace public schools. I suspect some of that money per student is basically going to corruption and featherbedding, and it’s surely *possible* to have less of those things in a school system than exists in many big-city schools in the US[1].

      I also have the impression that:

      a. There are a smallish number of students who cost a lot to educate, particularly students with various special needs. The average spent per student is probably quite a bit higher than the amount spent educating the median student.

      b. There are (as others have pointed out) some students who basically make it impossible for anyone else in their class to learn anything, or who spend their spare time terrorizing their fellow students for fun. Probably no tutors want to teach them anything–that may end up meaning they’re extra-high-cost, or just that they can’t find a tutor.

      c. Schools provide a ton of other services besides teaching–daycare, free lunches and breakfasts, running sports and band and plays, etc. Ideally, that stuff would be on a different line item in the budget than education, but the way the world is now, we bundle them all together.

      Still, it’s hard to justify spending enough per student to have a class size of four, but still having class sizes of 30 pretty routinely.

      [1] A few years ago, I remember stories in the local papers within a few months of each other, one talking about shameful conditions in some DC public school where the roofs leaked and the kids didn’t have books, another talking about a big scandal where it turned out there were a bunch of “ghost employees” collecting money from the DC school system without ever actually doing any work. It’s not so hard to suspect a causal link between those two….

    • riskybizniz says:

      The discussion on private tutors misses half the point of public school. Its not just about getting the population to some baseline general knowledge plus providing the smart ones with more difficult coursework if budgets allow for it. Its also about providing kids with a structured socialization environment so they know how to act once they come of age and go into broader society. All the costs we’d eliminate by going to some well-paid tutor-based system isn’t “money saved.” Its just eliminating one of things that we currently rely on our education system to do (and I’d argue that the US system actually does this pretty well). Plus, having some kind of shared cultural experience is nice. In the US, the only thing the majority of us can really hold onto as shared *is* high school. Even if you hated it and never participated, its useful for us culturally to have touchstones like prom, high school football, etc.

      Don’t get me wrong, I think a lot of fat could be trimmed in public education budgets and returned to teachers who deserve higher pay. My own experience (again, US) tells me that central school district administrations have issues with bloat similar to university administrations. Shifting to a system that delegates significantly more authority to school principals in lieu of the current hoards of bureaucrats common in larger school districts would help with this. Of course this also introduces its own risks (an incompetent principal can do way more damage this way), but ideally a change this radical would be thoroughly thought through to mitigate new risks.

      • John Schilling says:

        Its also about providing kids with a structured socialization environment so they know how to act once they come of age and go into broader society.

        Gosh, nobody here has heard that before,

        Well, except for those of us who have heard it a thousand or more times before, have spent a good chunk of our lives watching or experiencing public schools completely fail to accomplish and in many cases fail to even attempt to do this w/re a large fraction of their student body, and yet still keep hearing proponents of public education brag about this vital service they are allegedly performing.

        You seem to be new here, so fair warning: This is not the place where you’re going to get good results by telling the nerds how thankful they should be for prom and high school football and the like. At very least, you’re going to have to spell out and defend that argument far better than you have here, and it will be an uphill slog against an apathetic and skeptical audience. I’ts also not the place where “hey did you know public schools are useful for socialization?” is anything close to being a novel insight.

      • SEE says:

        If you were actually trying to socialize people to broader society, you’d actively expose them in controlled ways to broader society. Ideally in circumstances where they’re always outnumbered by the already-socialized; say, taking them into the world in small groups (never more than five or so) under the supervision of an adult.

        You certainly would not gather up large groups of the unsocialized persons and largely isolate them from broader society; that’s what you’d do if you were actively trying to prevent socialization to the broader society.

        • You also wouldn’t age segregate them, since that isn’t how the greater society works.

          To what extent does this defense of public schooling depend on assuming that families don’t work? I would have thought that in most past societies, it was the family where kids learned the basics of interacting with others.

          • SEE says:

            Mmm. For past societies, I’d say less “families” and more “band or village” depending on whether you’re hunter-gatherers or agricultural. Family was important, but substantial contact with non-family adult members of society would be the norm. In towns or cities, the pattern would tend to be “family/neighborhood, then apprenticeship”, the latter involving adolescents in contact with non-family adults regularly on a individual/small group basis.

            It is, of course, perfectly fair to point out that modern society isn’t structured to make the historical approach possible, and thus we potentially need an institution to do it. But that’s a long way from showing that the institutions that we have actually help do it, rather than being neutral or hindering it.

            That people were bled, and that people who were bled recovered from disease, didn’t actually constitute evidence that bloodletting helped people recover from disease. Similarly, that people are schooled, and people get socialized, does not constitute evidence that schooling socializes people.

        • Edward Scizorhands says:

          You certainly would not gather up large groups of the unsocialized persons and largely isolate them from broader society; that’s what you’d do if you were actively trying to prevent socialization to the broader society

          It’s what we do in prisons.

  43. Bobobob says:

    The Taiwan Junket: an unimportant assemblyman in a backwater state legislator gets asked to propose a meaningless bill about Taiwan. When the meaningless bill passes because nobody cares enough to vote against it, he gets hailed as a hero in Taiwan and offered a free trip to the country to attend a dinner in his honor. He concludes that this was a Taiwanese government propaganda effort to convince citizens who weren’t paying attention that they were successfully promoting Taiwanese interests abroad.

    This sounds amazingly like a Rick and Morty episode, in which dad Jerry insists to his son that Pluto is a planet, then gets invited to Pluto as an “eminent earth scientist” since the Plutonians don’t like being downgraded to an asteroid.

  44. proyas says:

    Gwern on hard to notice ways that things have improved during his lifetime (ie since the late 1980s). Some things are big technology, like the Internet and electric cars. Other things are tiny improvements in everyday objects, like self-adhesive stamps, power windows in cars, wheeled luggage, TVs you don’t have to adjust the antennae on, and computer mice you don’t have to remember to clean. Radios stopped being staticky, air travel got cheaper, showers don’t run out of hot water. Still others are vast vague improvements in whole areas of life, like cleaner air and water, or Amazon-style improved logistics. Recommended.

    I’d like to add flashlights to the list of improved goods. I remember pretty much all flashlights in the 1990s using incandescent light bulbs and non-rechargeable alkaline batteries, with the brightness being positively correlated with the mass and volume of the flashlight. Smaller flashlights that you could fit in your pocket were weak, dim, and didn’t cast light uniformly so there were usually annoying light/dark patterns. Today’s flashlights are vastly superior.

    The shoebox-sized flashlights from the 1993 Jurassic Park are no better than flashlights that today could fit in a large pocket.

    https://youtu.be/WSM8GcShChk?t=94

    https://youtu.be/V1bSsr1MYmc

    • keaswaran says:

      Do people still get flashlights?! I thought this was part of the desktop that was replaced by the smartphone!

      • Nick says:

        I have a flashlight for power outages, when I want to save the charge on my phone as much as possible.

        • baconbits9 says:

          Flashlights are also still much easier to use than your phone flashlight if you are doing anything beyond just looking where you are walking.

          Phone flashlights are still great.

      • bullseye says:

        I have a combination keychain/bottle opener/flashlight that I got for free at a bar (with purchase of the beer advertised on the keychain). It’s almost as good as the much larger flashlight I had in the 90s. I also have a very good (and probably very cheap!) flashlight I got for free at college (with purchase of the education advertised on the flashlight).

      • ChangingTime says:

        Your phone cannot throw nearly as much light as even a $10 pocket flashlight.

        • Matt M says:

          Yeah, a phone is a reasonable substitute for a flashlight in certain quick use cases where you need something quickly for a small period of time. But it’s really not competitive with a real flashlight in any situation in which you can reasonably forecast needing a flashlight ahead of time…

      • No One In Particular says:

        Flashlights are, like cameras and TVs, a device that a phone provides a serviceable replacement when one needs only a basic version, but still provides numerous advantages: a flashlight’s battery doesn’t run down while on “standby” (and you can get a crank powered flashlight at the dollar store, and there are low-powered lights that last longer while on than a phone does while on stand-by), it’s designed so that it can easily be held such that the light points forward, they often come with easily replaced batteries (you could get several phone batteries, and there probably are devices that allow you to charge a phone’s battery when it’s not in the phone, but that’s a bit of a hassle and phone batteries aren’t standardized like AA and AAA are), many flashlights have a “choke” (ability to change the spread), etc.

    • craftman says:

      Gwern’s “not worrying about Blockbuster or library late fines for VHS tapes” is replaced in 2020 with “worrying about cancelling your free trial of 80 different online services before the free trial ends”.

      Half of my Outlook calendar events are reminders to cancel something before I get charged, or to change my recurring bill-pays to a different credit card to maximize rewards, etc.

      • gwern says:

        or to change my recurring bill-pays to a different credit card to maximize rewards

        I was going to say, if you treat your online services like the hobby of credit card churning, you have no one else to blame other than yourself; but I see you already made that point for me.

    • tsutsifrutsi says:

      Narrow-beam flashlights are pretty good for “scanning” an area with; but I’ve still yet to see a good battery-powered portable point-source of light that’s a valid replacement for a wall torch/kerosene lamp/etc., in terms of ability to leave one in the center of the room, and so light up enough of the room to find things in it without constantly moving the light around.

      I feel like we’ve actually regressed here, technologically, in that we used to have lamps that also served this purpose, but now we’re reduced to relying on slowly doing a raster search-grid with a tight-beam flashlight. I’ll never find the spider I saw crawling along on my wall that way!

      I’m not sure why we don’t have these, either. It’s pretty simple: take one of those nightlight-globes, and put enough LED chips in it to throw 1K lm. Doesn’t need to last more than 30 minutes, just like a regular flashlight doesn’t, so it’s not like the battery needs to be huge even at that output.

    • matthewravery says:

      Another unambiguous improvement is wide-mouth canned drinks. The wider openings are such an obvious improvement that I don’t think you can even find the narrower ones that existed in the 1980s anymore.

    • LesHapablap says:

      LCD and plasma monitors and TVs! Lugging around a computer or moving a tv used to be an incredible pain. Now setting up a new computer is easy and takes hardly any space.

      Transferring money around the world, banking in general. Instead of writing checks for everything I can send payments directly from my account with no fees. I can set up autopayments, whatever.

      • Creutzer says:

        Transferring money around the world, banking in general. Instead of writing checks for everything I can send payments directly from my account with no fees. I can set up autopayments, whatever.

        Are you American? If not, be aware that Americans have no such luxury and effectively live in Europe’s 1990s in that regard. If you are: wait until you get banking like in Europe.

        It puzzles me to no end how user-unfriendly American banking is. People use checks all the time, some people seem to even have to go to a physical branch to make an international wire transfer, online banking has unthinkably bad UX, bank statements are totally unreadable and don’t even contain hints about who you sent the money to or who sent it to you (beyond the account number)…

        • Whereas I was shocked, many years ago, to discover that if I had a check drawn on a British bank the bank would not pay out the money. Apparently the check could only be used to transfer money from one account to another.

          Direct to bank transfers seem to be becoming fairly common in the U.S., although people still use checks as well.

  45. craftman says:

    More comments on Gwern:

    “Batteries are built-in no scrambling for batteries at Christmas” – increasingly so, yes, but clearly Gwern does not have children. I have a complicated stockpile of batteries that seems to change constantly. Baby toys all used D-batteries, now I have 20 that I can’t use because the toddler toys seem to prefer AA and C batteries. And now I have 5 different types of button-cell batteries (CR2032, etc) for the assorted tiny toys in my life.

    Car technology – not even just power windows, when I rent a car I can be in an absolutely base-level, no frills 2019 Nissan Versa and the thing will beep at me when I drift out of the lane. Incredible. My newest car is from 2017 and doesn’t even have this technology. Now it is standard (same with backup cameras, those aftermarket cameras will have no market in 5 years).

    • dpm96c says:

      This is definitely true—we had our first kid last year and rediscovering all the C and D batteries in the world felt kind of like one of those dreams where you discover a hidden room in your house, only much more expensive.

  46. keaswaran says:

    Moore’s Law is pretty great, but “in many areas, performance gains due to improvements in algorithms have vastly exceeded even the dramatic performance gains due to increased processor speed”.

    Note that the linked article is talking about specific high-end problem-solving. In most consumer applications, algorithms have likely gotten much kludgier, which is why computers from 5-10 years ago now often have trouble with things that should be simple, like restaurant websites.

    • albatross11 says:

      When do you stop worrying about making the program run faster? When it runs fast enough already.

      Fast hardware means that often crappy code that poorly implements a crappy algorithm still does okay.

    • Garrett says:

      > algorithms have likely gotten much kludgier

      For whom? In a lot of cases the results you are seeing are because the application/page designers are using libraries or tools to make their jobs easier and faster. This is because their time is the predominant cost in the use of the website/application/whatever. Faster computers let us produce faster results more cheaply. That these results happen to be bloated is immaterial to the people who care.

    • tsutsifrutsi says:

      I wouldn’t say that the algorithms most people use have gotten kludgier. People write fewer algorithms, instead relying on the algorithms included in ever-more-fully-featured standard runtime libraries; and those algorithms are usually quite well-written indeed.

      It used to be that you could achieve better performance by switching from writing e.g. your sorting routine in Ruby, to writing a naive implementation in C. But these days, you can achieve even more performance by rewriting your naive C implementation into one that relies on e.g. the data types and sorting routines built into Objective-C’s CoreFoundation, or C#’s CLR. And, unlike with switching to C “for performance”, anyone who uses any language based on these runtimes, even if it’s not a “performance-oriented” language, is just going to end up using those same optimized ADTs and calling those same optimized sort routines. For them, the easy thing is also, coincidentally, the optimal thing.

    • No One In Particular says:

      Like a blog comment section where every time someone posts a comment, their computer refreshes the entire page?

  47. keaswaran says:

    Regarding trust in experts, it looks like the polls cited in those data are of the UK. But according to Gallup, in the US the story seems to be different:

    https://news.gallup.com/poll/1597/confidence-institutions.aspx

    It looks like for all sorts of institutions and experts, trust has greatly decreased, except the military, which has gone way up, and police, which is only a little bit up. The medical establishment is the only look at science there, and it’s probably the one that has undergone the biggest drop.

    Pew only has data going back 4 years on trust in scientists in particular, and there’s been a major increase in trust in scientists in the US during the post-2016 era:

    https://www.pewresearch.org/science/2019/08/02/trust-and-mistrust-in-americans-views-of-scientific-experts/

  48. Aapje says:

    The story of the Great Eastern is really quite interesting.

    In programming, it is pretty clear that code reviews only find certain types of errors, which are typically very overt issues, so they are not a substitute for unit tests or functional tests. I think that the same is true for peer reviews in science. They are mainly good for overt problems, but not so much for more subtle issues.

    My standard for science is for something better than a minimum standard of honesty.

    • Lambert says:

      SS Great Britain also did a bunch of things over its 98 year working lifetime.

      Then in the 70s they refloated it and took it back to Bristol, where it remains in drydock as a museum ship.

    • I don’t think you can get something much better than a minimal standard of honesty via review before publication, given the costs in time and expertise of having every article seriously analyzed in advance. The way you get something better than that is by having an environment where, when someone publishes an interesting result, other people who are skeptical of it get to try to tear it apart. It helps if the norms require publication to include making the data freely available.

      That method works poorly where there is strong social pressure for some results and against others.

      • Aapje says:

        There are currently strong pressures to create positive results, which undermines the replication process in several ways.

      • No One In Particular says:

        Well, you could require everyone to post a $1 million bond before submitting a paper, to be confiscated if the paper is found to be erroneous, but I suppose some spoilsport will come along with objections to that plan.

        • Controls Freak says:

          This idea is fascinating and tantalizing, especially if the people who successfully point out the error are given some portion of the bond. Just with the errors I’ve already found in the literature and subsequently published corrections to, I think I’d be a much wealthier person. If it was often funded basically with insurance premiums (that rose as a person had more errors uncovered), it could get around any possible schemes to inject errors and collect on them. Probably would need to make reviewers unable to collect, because otherwise they could just approve a paper with errors in it and then turn around and collect. This would severely reduce the already-low incentive to be a reviewer, though it may incentivize people to review and then tell their buddies of some errors that are about to come out. Perhaps the idea would be that we could just get rid of the pre-publication peer review process altogether, substituting this strict post-publication financial review process.

          I think it would definitely cut down the total amount of absolute crap we have in the literature (yes, much of which makes it through peer review), just out of sheer fear to publish versus the more marginal incentive. But it presumably wouldn’t get at what I think is the far more often cause of crap papers – where they didn’t do anything wrong, so I can’t say, “This is wrong, so it shouldn’t be published,” but I want to scream, “THIS IS UTTERLY POINTLESS; YOU’RE SO JUST NOT EVEN GETTING IT; THERE IS NO VALUE IN THIS WORK WHATSOEVER!” …but I still say, “Fine. Publish it. There’s no skin off my back for having yet another crap paper out there. So long as it’s not actively wrong, I’ll let you be just another one of the folks who is gaming your institution’s metrics.”

        • anonymousskimmer says:

          But is a failure to replicate an error in the paper? What if the failure to replicate is because supplier X no longer makes the product, and their formulation had a particular undisclosed additive that made things work? Or that the 5 ppb contamination in the Milli-Q water happened to be mostly sulfur, versus the mostly copper in the other guy’s Milli-Q water?

      • Anthony says:

        We have evidence that editors really do blind papers they send to reviewers:

        https://twitter.com/KamdenStrunk/status/1264181757299118083

        Though sometimes they goof that up:

        https://twitter.com/NeumarkDN/status/1264717633036771332

    • zzzzort says:

      Peer review definitely has it’s issues, but I think the most likely alternatives are to lower standards for publication, and then have people adjust their opinions of the paper after the fact. A standard where only independently replicated science was published would just mean that the actual new ideas would move to preprints/working papers/some other faster medium.

  49. Aftagley says:

    Content Warning: Will be talking about forced abortions pretty often in this post.

    And “data manipulation” includes things like “sometimes they force abortions on unconsenting mothers because the fetus looks sickly and if it dies after birth it will ruin their infant mortality stats”.

    I went back and read the linked study, as well as some of the supporting studies. I feel like focusing on forced abortions portrays the least interesting version of this argument possible.

    Starting with forced abortions, the paper IMO is playing fast and lose with its claims here. Here’s the quote from the study:

    Physicians often perform abortions without clear consent of the mother, raising serious issues of medical ethics, when ultrasound reveals fetal abnormalities because ‘otherwise it might raise the infant mortality rate’

    Focus in on their use of the word often. Now, let’s go to the source they cite for this claim. (source) It’s the result of a series of interviews and embeds that one sociologist did with Cuban health care workers during the late 00s. It’s not bad at all, but it’s definitely qualitative. She was more interested in figuring out people’s perspectives and opinions then getting any kind of stats.

    The one time forced abortions are mentioned in her paper is in the following quote:

    One of the family doctors I worked with in Havana was quite politically militant and took these health goals very seriously. One day during my clinic observations I observed her scheduling an ultrasound for a pregnant woman. “What happens if an ultrasound shows some fetal abnormalities?” I asked. “The mother would have an abortion,” the doctor replied casually. “Why?” I queried. “Otherwise it might raise the infant mortality rate.”

    This is in a larger discussion about how some doctors are more political than others and how trust in them by the people varies along that axis. (IE, more political means less trust and patients, less political means the opposite). This is the only quote in the piece that discusses forced abortions.

    So, we have one quote from one researcher around a year ago who talks about how one doctor who is described as being especially and explicitly political was willing to theoretically say that this kind of abortion would take place. From this quote, the study decides to lower their estimate of the “actual” cuban life expectancy of males down almost 2 years. I mean, it’s their paper, they can do what they want, but that’s not especially scientific.

    I think the more interesting line of thought in that study is that a country that decides to optimize for something has the ability to optimize for it, even if that leads to metrics-chasing and paradoxical outcomes. An example they bring up is that the country, for political reasons, can’t import cars and mandates the use of bikes. Health outcome – way less obesity, heart disease and associated health risks.

    • Nikitis says:

      Thank you for doing the work to dig through the sources. The original claim was just as unbelievable as the claims of the average antivaxer or “pro-life” activist, and it’s nice to have confirmation that it’s as misleading as I expected, as well as how exactly it came to be.

    • From this quote, the study decides to lower their estimate of the “actual” cuban life expectancy of males down almost 2 years.

      That is not correct. They also had the anomalously high ratio of late fetal mortality to infant mortality — data, not anecdote — and the high abortion rate. The argument doesn’t depend on whether the abortions are forced, only on whether they happened.

      • Aftagley says:

        I think you might have misread the study. That’s from another section of the study and it alleges that doctors are reporting neonatal deaths as fetal deaths which very well could be happening, but that has nothing to do with abortions. Here’s the quote:

        Physicians are given health outcome targets to meet or face penalties. This provides incentives to manipulate data. Take Cuba’s much praised infant mortality rate for example. In most countries, the ratio of the numbers of neonatal deaths and late fetal deaths stay within a certain range of each other as they have many common causes and determinants. One study found that that while the ratio of late fetal deaths to early neonatal deaths in countries with available data stood between 1.04 and 3.03 (Gonzalez, 2015)—a ratio which is representative of Latin American countries as well (Gonzalez and Gilleskie, 2017).2 Cuba, with a ratio of 6, was a clear outlier. This skewed ratio is evidence that physicians likely reclassified early neonatal deaths as late fetal deaths, thus deflating the infant mortality statistics and propping up life expectancy.3 Cuban doctors were re-categorizing neonatal deaths as late fetal deaths in order for doctors to meet government targets for infant mortality.

        IE a child dying after being born looks bad on the stats, but one dying before birth doesn’t so it doesn’t matter when the kid dies, you always say it was prenatal so you don’t look bad to the central committee (or whatever system Cuba uses). I agree, this is fascinating, but doesn’t have anything to do with abortions.

        Also, this correction only accounted for around .5 years added lifespan.

        • It has something to do with abortions, because although one way of doing it is to lie about what died, another is to encourage a late term abortion of a fetus believed likely to die shortly after birth. The anecdote you discuss implies that at least one doctor was doing that — how many more we don’t know.

          But I admit that I haven’t figured out how the two different figures for the effect on life expectancy were separately produced, since both seem to depend on the anomalous late fetal death rate.

  50. bullseye says:

    Armchair speculation on the burnt houses:

    Their experiment was a single house in an open field. Would it burn more thoroughly if it were surrounded by other burning houses?

  51. Haru says:

    > Moore’s Law is pretty great, but “in many areas, performance gains due to improvements in algorithms have vastly exceeded even the dramatic performance gains due to increased processor speed”.

    I was curious about exactly which algorithmic advancements we’re talking about. According to the blog, it’s numerical linear algebra, but it gave no citation.

    I managed to track down the source for this claim to the “Operations Research” journal. Here is a link: https://www.jstor.org/stable/3088443

  52. Forlorn Hopes says:

    Regarding The Scunthorpe Problem. I was hit by that on this very site while trying to mention a DC comic charachter – the Joker’s female sidekick, you know the one – because the site filtered her surname.

    Mostly likely the filter was targeted at mentions of a personage in the sphere of videogames.

  53. alexmennen says:

    The Taiwan Junket reminds me of that time the California legislature unanimously passed a resolution endorsing the Asilomar AI principles, and the Future of Life Institute acted like this was significant.

    • Scott Alexander says:

      Wow, somehow I never heard about that! To save anyone else a Google, https://futureoflife.org/2018/08/31/state-of-california-endorses-asilomar-ai-principles/

      This actually seems nonzero importance to me. I’m remembering (ironically) some article on China’s Taiwan strategy, which argued that getting a bunch of countries to unrecognize Taiwan in small and trivial ways meant that if something important ever came up (eg a Taiwan-China war), China would be in a better situation to “hold those countries to their professed principles” and prevent them from recognizing Taiwan.

      In the same way, I think it could be potentially good that the state where most AI research is likely to happen is officially on record endorsing these principles, in case someone needs to pressure some mid-level bureaucrat to come up with regulations saying one thing and not another.

      I agree it’s probably pretty minimal in the grand scheme of things, though.

    • Deiseach says:

      I hate to throw cold water on any enthusiasm about the California state legislature recognising the importance for AI safety, but this is the same legislature that warns us going to the dentist can give us cancer.

      I mean, I’m very glad to dodge visiting the dentist for my annual check-up, and I know the risks of mercury, but I’d imagine I’d have a very good chance of getting the California legislature to endorse the use of shamrock as ethical feedstuff for leprechauns if I worded a proposal properly and got the lobbyists on the job.

    • tsutsifrutsi says:

      A mildly-different case, insofar as the actions of both tech giants and SV startups could in theory be heavily influenced by the actions of the California legislature (insofar as that’s where all the tech employees doing the AI inventing are.) For example, this support could later lead some California politician to propose a bill banning any “non-Asilomar” AI development from occurring within California. That’d be a non-negligible effect!

  54. sclmlw says:

    In Dan Carlin’s episode about strategic bombing (Logical Insanity) he talks about how destructive conventional bombing had become by the time the nuclear option became available.

    More people died in individual for bombing runs over places like Tokyo than at Hiroshima, and the process itself was horrific. The streets would melt into rivers, and people would get stuck in the goo. (It’s called Hardcore History for a reason.) To the point where one general justified dropping the nukes to stop the fire bombing! As Carlin puts it, how do you get to the point where you commit a war crime to stop yourself from committing more war crimes?

    The individual bomb may have been larger, but it was a natural linear ‘improvement’ in the development of bombing during WWII, at least as measured by casualties inflicted. Maybe fusion bombs were exponentially more powerful, but they were never used in ‘combat’, thankfully.

  55. Freddie deBoer says:

    “they’ve resisted Western pressure to stop having a gifted program in their education system, which probably helps a lot.”

    Please cite.

    • Randy M says:

      Agreed, seems to go against the prevailing (here) opinion of no great gains in IQ by education.

      • Haru says:

        It wouldn’t be surprising in the slightest that gifted programs in education help increase *scores in standardized tests*. Which is what is mentioned in the blog post.

        • Randy M says:

          Is that all that’s meant by “the above study shows Kazakhstan as having among the highest IQs in the world”?

          Also, I don’t think a “gifted program” probably helps as much as tracking the higher scoring into a more rigorous content coursework that goes deeper into the subject matter material. If that’s what’s meant by gifted program, then nevermind, but my impression is that it refers to a pull-out program with enrichment lessons–ie, unrelated to subject matter.

          • keaswaran says:

            I mean, “high IQ” is basically synonym for “does well on a standardized test”, since the only definition of IQ is a particular test score.

          • Randy M says:

            Sure, but I was under the impression from, ie, the nurture assumption discussions, that this particular standardized test was not especially trainable.

      • Viliam says:

        You can’t increase IQ by education, but you can increase education by putting high-IQ kids in the same classroom.

    • Deiseach says:

      Kazakhstan seems to have a robust investment in early years curriculum and policies; looking at this OECD report it reminds me very much of the programmes the Irish government itself is adopting and promulgating in this sector. So if I judge by the stated practice, ignoring how it may shake out in the field, it is on a par with a Western state (your opinions of the Irish education system may vary).

      This 2014 OECD report on secondary education gives a good overview of the state of schooling, the problems, and recommendations for improvement. A cursory reading shows that the Kazakhstani government is interested in getting specialised schools as a larger part of the education system and generally overhauling the entire system (rural, ungraded, schools do a lot worse and they’re trying to phase these out and get a better system in place):

      Table 1.3 also shows “specialisation schools”. These schools aim to offer education focused on certain groups of curriculum subjects in which their pupils have shown special interest or aptitude – currently either maths and natural sciences or social science and humanities. Between 2010 and 2011 the numbers of specialisation schools rose from 25.2% to 26.8% of the general MESRK schools, and their pupil numbers increased from 28% to 31% of all pupils in general MESRK schools.

      So pre-independence they were tied very closely to Russia and The Russian Way Of Doing Everything, and I’d say that there was a good chance the best-performing kids were creamed off, sent to specialised schools, and drilled into being high performers, thus getting the good results on standardised international tests:

      Kazakhstan has a longstanding tradition of participation in international Olympiads in natural sciences and mathematics. In 2010 Kazakhstan held the 51st international mathematical Olympiad at which Kazakh students reached the 5th place among 98 participating countries. In addition, Kazakhstan has participated in international student assessments – the first time in 2007 in the Trends in International Mathematics and Science Study (TIMSS) conducted by the International Association for the Evaluation of Educational Achievement, followed by participation in the 4th cycle of the OECD Programme for International Student Assessment (PISA) in 2009.

      However, they did poorly in later tests, which was widely perceived in the country as a shock result needing immediate remediation:

      Kazakhstan took part in TIMSS 2007 and 2011, as well as PISA 2009 and PISA 2012 (results of the 2012 PISA cycle were not yet available at the time of preparation of this report). In the TIMSS 2007 study Kazakhstan ranked 5th in mathematics and 11th in science among 4th-graders from 36 countries. System performance dropped significantly in the next cycle of TIMSS-2011 to around average performance. The country ranked 27th in the 4th grade mathematics test and 32nd in the 4th grade science test among 50 countries, and 17th in mathematics and 20th in science for 8th graders out of 42 countries. In PISA 2009, Kazakhstan scored 405 points in mathematics (place 56 of 74 participating economies), 400 points in science (place 64) and 390 points in reading (place 64).

      After the good rankings in TIMSS 2007, the PISA results were perceived as disappointing by education authorities and the wider public alike.

      The participation of Kazakhstan in the two international student surveys PISA and TIMSS confronted the local education community with a mixed message. Students in Kazakhstan ranked high in the TIMSS mathematics assessments of 4th and 8th graders (5th place) and TIMSS science assessment (11th place), but very low in PISA. The disappointing PISA results are ascribed “to the unfinished agenda of raising student learning achievement beyond basic literacy and numeracy” (World Bank, 2012). The “PISA shock” of Kazakhstan triggered the Ministry of Education and Science of the Republic of Kazakhstan to look for ways to revise the very traditional approaches related to content, technologies and training methods that currently dominate teaching and learning in schools (MESRK, 2012a) in view of a positional improvement in the rankings of international student assessments.

      Part of the PISA shock response was the establishment of centres of excellence through a network of new schools called Nazarbayev Intellectual Schools (NIS) mentioned above. These are designed as incubators of innovation that focus on providing students with incentives to learn, teachers with a new system of professional development, and schools with the capacity to offer a personalised education environment which is more sensitive to the needs of each student. In parallel, the authorities are working on improving
      the assessment system through the introduction of standardised national evaluations at the end of each education cycle and of assessment standards for classroom assessment of student performance.

      So the authorities have responded by throwing money and resources at upskilling teachers, making teaching a respected profession, and overhauling the pedagogical system, but there has always been an emphasis on “gifted children” even under the old model:

      Figure 2.1 shows the number of schools catering for gifted children in each region of Kazakhstan, but as noted, there are many other schools within the “normal” system which also cater for gifted children. Overall, education policy in Kazakhstan attaches high priority to “gifted children” (a category for which there appears to be no exact definition), which expresses itself through more favourable resource allocations, rewards, and overall attention in government plans and statements that spell out the importance of helping them to develop their gifts. Schools for gifted children also have better buildings and facilities. As the Background Report notes: “One of the objectives of the national education policy is to identify and provide guidance and support to gifted children to facilitate the development of well-educated, competitive, and creative personalities. The network of special educational organisations for gifted children in Kazakhstan has been expanded to create better conditions and opportunities for uncovering and developing children’s abilities and fulfilling their potential.”

      Even in (and between) “normal” schools that are not specialised in supporting young talent there seem to be an extraordinary number of competitions, awards and prizes for high performers. The rewards for emerging victorious from these competitions and contests go not just to the students themselves, but also to their teachers and schools; teachers may receivespecial titles and benefits and schools, if persistently successful in producing outstanding results in competitions and on the Unified National Test (UNT), might ultimately be granted a more privileged status such as “experimental” school, “specialisation” school (see Chapters 1 and 2), or lyceum.

      To that end, it seems that the best teachers are expected to devote their time and attention to making the best students even better. By contrast, there are no special schools or programmes for children who are less gifted, or who are struggling academically, either in regular schools or in centres offering extra-curricular education.

      Another conclusion is that Kazakhstan followed the Russian model, does very well when compared with other Eastern Bloc style countries tested under that model, but not so well on Western-model tests like PISA, and they very much want to do so:

      As the table shows, there were relatively few excellent performers among the Kazakh students who participated in PISA 2009. Kazakhstan’s Level 6 and 5 percentages were less than a tenth of the OECD averages in all subjects, and just one OECD country, Mexico, had fewer top performers (in maths and science: in reading, the percentages were the same).

      Three possible explanations could be given. The first is that Kazakhstan’s very best students (those capable of beating the world’s best in other international competitions) were not tested. This might or might not be an explanation. The school sample countries prepare for participation in PISA must meet strict standards in order to be accepted by the PISA consortium.

      One of them is the requirement for the sample to be representative. Since Kazakhstan is selecting and concentrating all of its excellence students in few dedicated institutions, it is possible that none or only very few of these schools were sampled …The second possible explanation is that PISA 2009 did not test students in those aspects of reading, maths and science on which their class teaching focuses. This explanation seems plausible. Kazakhstan shares a teaching heritage with other countries formerly hidden behind the iron curtain, all of which have disappointing PISA results …The third possible explanation is that Kazakhstan’s emphasis on identifying gifted children as special, giving them special attention and rewards and putting pressure on schoolchildren of all abilities to compete for prizes, is not necessarily the best way to foster high performance as measured by PISA.

      And finally, language (which seems to correlate to a whole host of other background elements) may have something to do with it!

      According to PISA 2009, almost two third of students in Kazakhstan speak Kazakh at home; around 30% speak Russian and the remaining 3% another language. Analysis of PISA results by the language students speak at home reveals large performance gaps between Kazakh- and Russianspeaking students, who generally attend separate schools with a different language of instruction.

      Russian-speaking students outperform their Kazakh-speaking peers in all 3 subjects, but still lag nearly 50 score points behind the OECD average of 500 points. The gap between Russian- and Kazakh-speaking students is smallest in maths and biggest in reading, where it corresponds to more than 2 years of schooling …There are also dramatic differences in proficiency levels …These differences suggest the presence of inequities in the education system in Kazakhstan, but closer inspection reveals that much of the performance difference between Russian and Kazakh speakers can be attributed to differences other than language. Russian-speaking students had higher preprimary attendance rates than Kazakh-speaking students. They also had, on average, higher socio-economic status, higher family income and more educational resources in their homes, though Kazakh-speaking students had more cultural possessions in theirs. The proportion of girls was higher in the Russian-speaking sample (51.1% compared to 48.7%) which is likely to have had a positive influence on the outcomes of the reading test, at which in all countries participating in PISA girls outperform boys. The proportion of Russian-speaking students participating in academic as opposed to vocational programmes was higher, and more of the Russian-speakers were in private schools, whose students generally perform better in PISA than their peers from public schools. Last but not least, significantly fewer Russian than Kazakh speakers were schooled in villages (21.3% to 39.8%) and significantly more in cities (40.9% to 28%) and in large cities (12.2% to 4.8%).

      Also, girls. Girls do better than boys, having more girls in your school/taking the tests bumps up test scores 🙂

    • No One In Particular says:

      I think that the natural reading is that it is Scott’s personal opinion that this helping is a likely possibility, so asking for a cite is a bit odd.

  56. Freddie deBoer says:

    Romney’s homeless “solution” is a solution only if you assume the people unwilling to sleep in a shelter are genuinely unworthy of government-funded temporary shelter.

    • Randy M says:

      Isn’t it more fair on the face of it to give the nicer accommodations to those that have been in the shelter the longest?

      Beyond that, do you have some argument that the shelters in question were particularly bad? Communal living is never going to be great, but that’s due to the community. My college dorm, lacking most amenities including private bathrooms, was a great experience, because of who was there.

      But is there an efficient way for the government to provide private housing to anyone who wishes it?
      Also, there’s sometimes the case that homeless dislike homeless shelters because of the rules, like no drinking or fighting or whatever. It’s certainly their choice to follow their preferences, but if that is the case I don’t see the wider community obligated to oblige them.

    • Matt M says:

      If we start with a prior that homeless people are generally unworthy of shelter in the specific form of private hotel rooms – then yes. Anyone whose position is “I want shelter but only if it’s a private hotel room, I will choose to reject all other forms of shelter” is, in fact, unworthy of shelter.

    • 10240 says:

      Please cite.

      (I just wanted to point out that personal commentary on a website like this is not normally expected to be based on external sources, and insinuating that a particular comment shouldn’t be made without a citation is an isolated demand for rigor.)

      • Dan L says:

        Speaking for myself: it’s not an isolated demand for rigor, it’s a conventional double standard*. I have significantly elevated expectations when I think someone might actually be rational enough to be truth-seeking as a matter of systemic practice rather than happy accident. It’s much easier to meet the standard that replacement-level blogs might be held to, but then there’s no reason to take one seriously.

        *Obviously not a true binary and subject-matter expertise/compartmentalization is a thing that must be accounted for.

    • matkoniecz says:

      Government-funded temporary shelter for homeless is going to be, by definition, a shelter for homeless.

      It is likely that homeless shelters should be better, but “once shelter is full anyone who will arrive will get hotel voucher for free” seems a poor solution for achieving anything desirable.

      And after change, shelters are still not going to be filled beyond capacity.

      • Matt M says:

        It is likely that homeless shelters should be better, but “once shelter is full anyone who will arrive will get hotel voucher for free” seems a poor solution for achieving anything desirable.

        It’s funny – I experienced this myself once in terms of military housing.

        When you traveled for training, you normally had to stay in the BEQ (bachelor enlisted quarters, it’s basically a hotel, but double occupancy and less nice than a normal hotel). BUT, if the BEQ was full, they’d give you a CNA (certificate of non-availability) which meant you could basically go to any reasonably decent hotel in town and expense the cost.

        So the first thing you did when you showed up was beg the guy at the desk to give you a CNA, which he pretty much could at his own discretion, regardless of how full the BEQ actually is. I’m told bribery was common, but never engaged in it myself. But there were multiple times where I went to training classes where I ended up stuck in the BEQ but many of the other participants were “out in town” because they got CNAs…

    • No One In Particular says:

      Can you quote the part of the blog post that talks about Romney’s solution to homeless? I only see a discussion of a solution to a homeless shelter *budget crisis*.

    • Controls Freak says:

      Romney’s homeless “solution” is a solution only if you assume the people unwilling to sleep in a shelter are genuinely unworthy of government-funded temporary shelter.

      Nobody said anything about “unworthy”. If I try to give away something on Craigslist, and some people don’t want it, that doesn’t mean that they’re unworthy of the item. Nor does it mean that they are more broadly unworthy of my charity. It means that they don’t want that thing. Maybe I’ll consider giving them some other form of charity, but that’s simply a different matter.

    • deciusbrutus says:

      I consider it mostly acceptable if the homeless shelter was itself at least minimally adequate and almost all of the people who didn’t sleep at the shelter went somewhere superior.

      It’s likely that a lot of the substitutions were to couch-dwelling or other specific living conditions that I personally consider better than homeless shelters but inferior to hotel rooms. You might not share my rankings, but I found them uncontroversial among other homeless people.

  57. LadyJane says:

    Pretty sure Safe Space and Snowflake are supposed to be joke characters, like the rest of the new New Warriors lineup. This whole New Warriors reboot seems to be going with a “refuge in audacity” tone, kinda like that short-lived “X-Statix” book from the early 2000s with characters like Mr. Sensitive, U-Go Girl, and Mysterious Fan Boy.

    Also, since the late 80s/early 90s, there’s been a decline in the amount of new characters in Marvel/DC comics who generate enough interest to stick around. This is partially due to the rise of indie comics, combined with copyright incentives: Writers know that they won’t own the rights to any character they introduce in a Marvel or DC book, so they tend to save their best ideas for creator-owned indie comics; when they do write for Marvel and DC, they either won’t introduce new characters at all, or they’ll come up with less interesting and developed characters than they do for their own books. It’s also partially just because the Marvel and DC universes are massively overcrowded already. There are just so many Marvel and DC characters already that all but the most unique new heroes and villains will seem like superfluous knockoffs of existing ones. Making absurd, ridiculous, over-the-top joke characters is one of the few reliable ways to make new characters that stand out (although if comics keep trying to put out Deadpool knockoffs, that niche is going to start getting overcrowded too).

    • Le Maistre Chat says:

      This is partially due to the rise of indie comics, combined with copyright incentives: Writers know that they won’t own the rights to any character they introduce in a Marvel or DC book, so they tend to save their best ideas for creator-owned indie comics; when they do write for Marvel and DC, they either won’t introduce new characters at all, or they’ll come up with less interesting and developed characters than they do for their own books. It’s also partially just because the Marvel and DC universes are massively overcrowded already.

      Bingo. Comic books are low-selling source material for TV and Marvel movies (sorry, DC). Superhero movies and TV will very probably go the way of the Western before they’ve brought most of the characters created in the 1940s-80s to screen.

    • JPNunez says:

      This effect is so big that sometimes companies hire writers to revitalize old/obscure super heroes; see Ryan North and Squirrel Girl or Warren Ellis and Nextwave.

      That said, there’s still enough low hanging fruit to create new superheroes that are minorities: see Spiderman but black, aka Miles Morales, and Inhumans -uh…maybe not the best word choice- but muslim aka Kamala Khan.

      • Le Maistre Chat says:

        That said, there’s still enough low hanging fruit to create new superheroes that are minorities: see Spiderman but black, aka Miles Morales,

        If I understand right, Miles Morales is black AND Latino. Not in the sense that someone remembered that Latinos can be black, but “Ms. Morales is a Latina and his father an ‘Anglo’ Afro-American named Jefferson Davis (!)”.

      • LadyJane says:

        I’m only really familiar with Miles Morales from the Spider-Verse movie, but he certainly seemed like a very different character than Peter Parker in ways that went beyond skin color or racial stereotypes, at least in that film. Can’t say whether the same is true for the comics version.

      • Mabuse7 says:

        But one common complaint about the “legacy character” approach is that it’s a compromised half measure that often doesn’t please anyone, fans of the original character will be mad that the spotlight is being taken off of them and those seeking greater representation will have to be content to only get such in the shadow of older characters. Really, the whole “corporate mythos” approach to storytelling that the Big 2 superhero houses have adopted is inherently flawed, over time it becomes almost impossible to really change the status quo and any attempt to create something new will be swamped by all of the established content. The more self-contained approach of creator-owned comics or European and Japanese publishers really works better for this type of content. Stories, settings, and character arcs have a beginning, a middle, and an end, successful works can be revisited in certain ways but never just stretched out beyond their useful life. That way creators can make works taking cues and influences from works they admire but truly make it their own original thing rather than being forced to use the exact same characters, settings, and story beats over and over again.

  58. Lambert says:

    Can’t you just use a chandelier full of 1500lm bulbs, if you can’t find one 4000lm one?

  59. Mercurial says:

    I can’t vouch for drugsand.me at this point. They describe Alcohol+Benzos as:

    Undesirable

    Taking both these drugs together may produce unpleasant effects such as physical discomfort or overstimulation. High doses put you at risk. If you mix, start low and go slow

    While describing Alcohol+Amphetamines as:

    Dangerous

    We strongly recommend you avoid taking these drugs together, reactions are highly unpredictable and can be fatal. You will likely come to some degree of harm even if you take strong precautions.

    Their source is the tripsit.me chart (which every druggy I know treats like a bible). The Chart contains the opposite information, listing Alcohol+Benzos as “danger” and Alcohol+Amphetamines as “caution”. It seems to me like they made a mistake copying from the chart.

    • Lillian says:

      From what I recall, alcohol, benzos, and opiates are the unholy accidental overdose triad. Mix any two of the three, and you are taking a huge risk of hurting or killing yourself no matter how careful you are due to the sheer unpredictability of interactions. It’s my understanding that the issue is they’re all downers, which means they can down your body all the way into shutdown. Moreover they all act through very different mechanisms, which means the effects of taking them together are more multiplicative than additive.

      Amphetamine on the other hand is an upper so mixing it with a downer is not as immediately and horribly dangerous, but that also means there’s little reason why you’d want to as the clashing effects tend to be unpleasant, and of course you can still kill yourself if you’re careless. The thing that is super dangerous to mix amphetamines with is cocaine, unless you fancy a heart attack.

      • theredsheep says:

        It is indeed fairly stupid to mix benzos and opiates, but my pharmacy sees at least a half-dozen patients each day who are on both at the same time, by prescription. Some people have a truly prodigious tolerance; I’ve known some who are on a benzo, an opiate, some ambien, methocarbamol, and like every other drug in the pharmacy that causes sedation. Some of them needed a little Adderall to avoid walking into walls, but I’m damned if they aren’t out and about nonetheless.

      • Mercurial says:

        Some people I know swear by nursing a beer or two while using amphetamine for work. They say it really takes the edge off negative effects, especially the comedown. I wonder how much of it is the alcohol and how much of it is the nutritional content of the drink..

    • caryatis says:

      I wish they’d say more about specifically what the adverse effects are. In this case, alcohol plus benzos increases risk of blackouts. (Edit: and respiratory depression, which is more serious but much less likely.)

  60. Aftagley says:

    Related: a story about how Mitt Romney solved a homeless shelter budget crisis in Massachusetts.

    I swear I’m not just trying to nitpick. The only reason I looked at this one was because that Washington Post article was such a puff piece that I just couldn’t believe it was true. Turns out, it might not be:

    For starters, something like this seems like it would have been big news; it’s folksy and charming so you’d think there’d be reporting at least in Mass about it. But, as far as I can tell, nothing. That being said, this was the early 2000s, so maybe we’ve just lost the reporting from then. I kept looking, found one Buzzfeed article from 2012 that said it might not be true; but other than that nada from the news.

    Then, I found Romney’s book where tells this story, but includes a bit more detail. Quoting from a speech he gave a CPAC in 2011,

    Now before that change in incentive went in place, guess how many rooms a night we were renting in Massachusetts? Five hundred and ninety nine rooms a night on average. In the tens of millions of dollars of cost. After that, we changed that incentive, you know how many rooms we rent a night? Zero. Zero.

    Now, that inclusion of 599 rooms gives me some real information to work with. A bit of research turns up this report from a 2008 summary of homelessness in Massachusetts. It mentions the hotel thing, but the account differs somewhat from Romney’s. Here’s a snippet:

    In August 1999, the Department of Transitional Assistance (DTA) had to begin using hotel and
    motel placements, after a period of rapid rent inflation and cuts in State funding for family
    homelessness prevention assistance led to rising family homelessness. Hotel/motel use rose to
    599 families and State spending on shelters, hotels and motels increased by 82% between
    FY1999 and FY2004, but the State was not able to end hotel/motel use until August 2004,
    when it created a one-time subsidy program for families in shelters and added 148 more rooms to
    the shelter system.

    That experience led the Governor to create a special commission in 2003 to recommend ways to
    improve state programs for the homeless.64 In November 2003, the Commission concluded
    that “the state’s historic focus on emergency shelter is an ineffective way to manage…resources
    and services and recommended that the Sta

    So – the first piece of news is that these were hotel rooms for homeless families, not homeless individuals. Now, Romney never explicitly says that he’s talking about individuals, but it’s definitely the implication I got from the piece. I get that it doesn’t materially change the facts of the situation… but I get a different vibe from this story when I consider that the individuals involved all have kids.

    Second biggest change is that it looks like Romney didn’t re-prioritize who gets hotel rooms, he shut down the hotel program in favor of expanded access to long term shelters and direct payment plans. These aren’t bad ideas… but it looks like the real reason people weren’t getting hotel rooms wasn’t his folksy first in-first out plan, it was because he said no more hotel rooms.

    Third, from later on in that report:

    Overall, however, the State did not implement the recommended shift to prevention programs
    and shelter use has continued to rise. … In November 2007,
    DTA was forced to start using hotel/motel placements again

    Now, to be fair, Romney left office in January 2007, so he wasn’t in charge when the state shifted back to using hotels… but at best his novel change only lasted 3 years and outlasted his tenure by only 11 months.

    TLDR: it is very unlikely that this story is true. While Romney did reduce the number of hotel rooms occupied by the homeless, it looks like he misrepresented the problem (hotels were primarily used by families, not individuals) he either lied about or dramatically oversimplified his solution (he didn’t change who got to use hotels, he straight up ended the use of hotels altogether) and by repeating the idea that the state was no longer purchasing hotel rooms in 2011-2012 (when the WaPo article quoted him) he was either factually incorrect or lying, since the state went back to buying hotel rooms in 2007.

  61. caryatis says:

    Poking around on Drugs And Me, I’m seeing that for alcohol + SSRIs, they say:

    Dangerous: We strongly recommend you avoid taking these drugs together, reactions are highly unpredictable and can be fatal. You will likely come to some degree of harm even if you take strong precautions.

    I never heard of this! Is it true, and how bad is the harm? Seems like a pretty big drawback of SSRIs. Considering how mainstream both they and alcohol are, there must be a significant percentage of people who drink while on them.

    • Thomas Jorgensen says:

      ….. That is not the question I would ask. Instead, this: How much of efficacy of SSRIs as anti-depressants is down to people not drinking while on them? That is, are they actually doing anything a prescription of disulfiram would not also accomplish?

    • Deiseach says:

      If I believe the NHS, while it’s not advisable to drink and take SSRIs it’s not as dangerous as that makes it sound. MAOIs seem to be the danger ones here, I wonder if that website got confused, particularly if it’s cutting’n’pasting from elsewhere:

      Selective serotonin re-uptake inhibitors (SSRIs)
      SSRIs generally do not cause problems when taken with alcohol, and it may be safe to drink alcohol while taking them.

      But the manufacturers advise avoiding alcohol during treatment because it might make you feel drowsy.

      Tricyclic antidepressants (TCAs)
      TCAs can make you feel drowsy and affect your co-ordination, particularly during the first few weeks.

      The manufacturers advise avoiding alcohol while taking TCAs, although it may be safe to drink small amounts after a few weeks once the side effects have settled.

      Monoamine-oxidase inhibitors (MAOIs)
      A substance called tyramine, found in some alcoholic drinks, such as wine, beer and sherry, can cause serious side effects if taken with MAOIs, including a sudden and dangerous rise in blood pressure.

      If you’re taking an MAOI, it’s best not to drink alcohol and avoid consuming food or drinks containing tyramine.

  62. gwern says:

    Turn on faster? No, incandescents are instant. LEDs have a delay. It’s not enough of a delay that it bothers me, but the statement is inaccurate.

    Faster was referring to fluorescents there, although I don’t believe you about incandescents being noticeably faster either (certainly I’ve never noticed) and I’m doubtful about your other claims as well.

    • albatross11 says:

      I’ve noticed a very small delay (maybe 1/10 sec or so?) when I flip the switch on LED bulbs in my house. I suspect this has to do with the electronics needed to turn 120v AC to 5v DC, but maybe there’s also other stuff going on.

    • tsutsifrutsi says:

      You’re thinking of fluorescents. Fluorescents emit light at a small number of distinct wavelengths. LEDs (or at least “white” LEDs) internally emit light at *one* wavelength—blue-ish—which is then passed through a phosphor layer, re-emitting the energy smeared out into a continuous—though not flat—spectrum (E.g. the bottom-middle and bottom-right in https://i.stack.imgur.com/dT3mv.jpg .) Such a curve might not be as smooth as the blackbody curve of incandescents, but it doesn’t look like it was put through a band-filter, the way a “white” image on an LCD screen does; nor is it “spiky” like a fluorescent’s curve is. It’s just low-ish on cyan frequencies. (Which throws off a 6000K LED light when comparing it to daylight, but really doesn’t make much difference when comparing a 2700K LED light to an incandescent.)

      • Lambert says:

        Fluorescents also have a phosphor layer. That’s what fluoresces.
        It’s just a different sort of phosphor because the mercury naturally emits UVC photons.

  63. Quideck says:

    Regarding Japanese efficiency in the auto industry, I have some crackpot answers to Scott’s question:

    exactly what the Japanese advantage was and why it was so hard for Americans to do

    1) Japanese advantage: Basically all of those little techniques they mentioned, added up, are very useful. It seems the competitive environment Japanese automakers found themselves in forced them to innovate efficient (and very different compared to a Ford) manufacturing techniques. Lean Manufacturing / Six Sigma methods are still often taught as originating from the Toyota way ( https://en.wikipedia.org/wiki/Lean_Six_Sigma – though reading the wiki it sounds like the “Lean” part came from Japanese auto and the “Six Sigma” from the semiconductor industry). Basically, manufacturing leadership is very hard, but Toyota and co managed to distill some principles that navigate reality better than the old Ford mass production philosophy.

    2) Why was it hard for Americans to replicate?

    I am wondering if Institutional Momentum or something like the costs associated with retraining managers in new techniques could explain the lag in productivity.

    Today, every manufacturer I have worked for implements policies that at least nod to the Toyota way. I worked at one plant founded in the 90’s that more-or-less successfully implemented many of the improvements detailed in the article. I more recently worked at a factory that exactly matched the article’s description of old-school manufacturing management. This second factory had older leadership who had worked at the plant longer, and operated in an industry with much higher profit margins than the first plant. Both factories employed between 200 and 300 people.

    I attended Lean Six Sigma training from corporate at both companies. However, at the second factory I found it impossible to implement many of the described improvements without management support, and most of the managers were honestly just too far along in their career to make drastic changes to their habits. This second plant also had larger profit margins just due to the industry (shrinking today) than the first plant, and so the disruption of removing a nominally-competent leadership team to chase efficiency improvements was less of a requirement.

    Circling back to the actual issue: From my understanding, American automotive manufacturing is approaching the sort of efficiency you get from Japan, and from my personal anecdata adjacent to the semiconductor business, there is no great feeling of inferiority in most American manufacturing nowadays. I am wondering if it literally took about a generation after anyone even thought of copying Toyota for American managers to effectively adopt the methodologies of their Japanese competitors. If so, this would suggest that Management or Social technologies often spread much slower than engineering/science sort of technologies.

    Also: perhaps a company or industry needs to be in extremely dire straits (like 2008 financial crisis level for American Auto, or the destruction and occupation of your country for Japanese auto) to feel comfortable instituting the massive cullings/restructurings necessary to radically reshape company culture.

    • Lambert says:

      I think the West has learned these lessons.
      Or has at least been taught them. If I had a dollar for every time my technical ops lecturer mentioned Deming or Taguchi…
      Also they taught us about JIT by setting us up in a production line building lego cars first with push manufacturing and then with kanbans.

  64. NostalgiaForInfinity says:

    The UK spends £6,300 per pupil (in high school), which is about $9,100 (PPP), lower than any of the states listed in that tweet. London would be slightly higher, but I can’t find the figures for high school only and the overall spend is only about 15% higher. FWIW, the UK does about as well or slightly better than the US in PISA comparison. Something is particularly weird about the American public school system. Teacher salaries seem to be about the same, as is the ratio of teachers to students. So how are they spending twice as much?

    • aristides says:

      A quick google search, meaning I’m not sure if this is an apples to apples comparison shows that average teacher salary In the UK is 38,400 pounds, or $47,000, and the average salary for a US teacher is $60,477. Assuming it’s not just teachers that have this pay increase, I’d be shocked if our Education Administrators aren’t higher paid higher as well, that could close the cost per student gap. Other possibilities is that the US might have a higher concentration of English as a Second Language students, and we might be spending more to educate them. Also, average high school class size in UK is supposedly 21 and it is supposedly 17 in the US. That means the UK needs to pay less teachers for the same amount of students.

  65. SEE says:

    but although it has lots of great history I still don’t have a good feeling for exactly what the Japanese advantage was and why it was so hard for Americans to do even when the Japanese basically handed their US competitors all their procedures on a silver platter.

    Hmm? Simple enough. You can’t adopt Japanese procedures when the labor rules in your labor contract specifically prohibit you from doing so. The UAW is why US automakers could not properly and fully adopt Japanese procedures.

    Seriously, this was proven by Saturn. GM-owned, GM-operated, but with the Spring Hill factory not under the standard UAW contract, Saturn produced the S-series, an excellent small car beloved of places like Consumer Reports that earned astonishing customer loyalty.

    Then Saturn expanded to a second model at a second plant, this second plant under the standard UAW contract, and the resulting L-Series sucked just as much as the rest of GM’s products, and destroyed customer loyalty as a bunch of S-to-L upgraders were burned.

    The single common thread at GM, Ford, Chrysler, and American Motors (acquired by Chrysler in 1987) was the UAW pattern contract. Which didn’t apply to the Japanese-owned plants in the US, the German-owned plants in the US, and the original Spring Hill Saturn (until 2004).

    • 10240 says:

      How does a union contract reduce the quality of the car? I’d understand it if it made it more expensive.

      • John Schilling says:

        In order to make high-quality cars, you have to not have people on the assembly line who do low-quality work. Typical union contracts care more about job security than they do about wages, making it hard to fire (or otherwise penalize) workers for any reason, including poor workmanship. Not impossible, but more trouble than it’s worth at the margin, meaning a lot of marginally crappy work is going to get through the system. You may not even be able to promote the workers who do an extraordinarily good job, if there’s another worker with more seniority doing a mediocre job.

        • JPNunez says:

          Keep in mind that Toyota and other japanese automakers also had unions _in_japan_ of differing strength; Toyota had one that cooperated with management the most, due to the trust they had on management, while other japanese companies had more strike prone unions, but still managed to outperform the americans.

          So it’s specifically the UAW contract that’s the problem, not unions per se, so “typical unon contract” may not be all that accurate.

          Not the most unbiased reference but

          https://www.toyota-global.com/company/history_of_toyota/75years/data/company_information/personnel/labor-management_relations/outline.html

        • tsutsifrutsi says:

          I mean, in theory, you could just build your assembly line in a Fault Tolerant manner, with redundant/idempotent process applications at every step (i.e. person A tightens a bolt until it cams out; then person B tightens the same bolt until it cams out, which should normally happen right away if person A did their job), overproduction+filtering (i.e. you make five parts for every three you need, with the expectation of throwing two away), etc.

          Then you don’t need to fire anybody, or even move them out of nominally “important” roles—you just make them into non-load-bearing components of the system.

          This would, of course, increase costs. Which I think is what 10240 meant by “I’d understand if it made it more expensive.” You can totally set a quality target and reach it even in the face of some (but not all) employees being incompetent; you just have to relax the budget constraint.

        • SEE says:

          Eh. Fremont Assembly under NUMMI (the Toyota-GM joint venture) inherited 85% of the old GM workforce and most of the old plant union hierarchy, and got down to Toyota-level defects in its first year. If the method had been firing and disciplining workers, that would have provoked disgruntlement and sabotage and wildcat strikes, not efficiency.

          Rather, the key was they (just like Spring Hill) simply didn’t have the dozens-and-dozens of job classifications and detailed work rules contract that was standard in UAW plants. They were able to use Toyota processes and procedures without it being a contract violation sparking grievance procedures, and the Toyota approach was just that much better.

      • CatCube says:

        You still have to hit price points commensurate with your competitors or nobody will buy from you anyway. If you can’t increase efficiency and reduce labor costs, reducing build quality with cheaper and easier to fabricate components is the only dial available to you. Reducing quality will affect your sales later, while keeping your cars costing twice as much will affect your sales now. Maybe the horse will sing, and all that.

        While this isn’t the same because I don’t believe there was a union involved, you can see some examples with other industries. The Youtuber 8bitguy talked about his time with the computer company AST in the late ’90s. AST was the last company to fabricate most of its computers in the US, and desperately tried to keep their plants here. If they kept build quality the same, they would cost twice as much as their competitors, which market research (and I think some actual experience) showed would result in plummeting sales. So they reduced build quality to become price-competitive, and eventually went bankrupt because they quickly got a reputation as shoddy. This demonstrates why we don’t produce stuff here in the US like masks: no matter how much you bang on about how people should purchase American made to maintain local industry, saving money will always win out, and you can’t make a profit in many industries while paying American salaries.

        That’s probably the main reason, but onerous union restrictions can prevent firing people who are causing problems, or reduce your ability to improve efficiency with the threat of firing, as well. Sometimes it sucks when a new process will mean you have to work harder and a third of your coworkers will be fired. The new process might also be necessary to prevent the company from collapsing in the face of competition. If there’s a veto point that prevents its adoption, the people who will have fewer, more stressful jobs will use it.

      • David W says:

        I think it’s fair to allocate some blame to management – they signed the contracts, and could have worked harder to get out of them. At least they could have focused more on the parts of the contract where they gave up flexibility in lieu of money. On top, you don’t get to a point with such pervasive distrust from the union without earning it.

        Still, quality effects happened through two channels as I understand it. First: work roles were much more tightly defined, causing a lot of ‘not my fault, not my problem’ attitudes (also a factor influenced by management!)

        The Toyota system works by building an esprit de corps, where problems are supposed to be raised by any and every worker who notices them. And, in turn, where there is a support structure where that leads to changes, rather than blame. You can’t get that effect when people aren’t allowed to go outside their role and are hammered when they do. And I’m not sure you can build that kind of trust when the union men rightly suspect that management is looking for any excuse to fire people. But you just can’t get the same quality out of a system where you rely on designated inspectors as you can from a system where everyone cares.

        Take a tangible example. I’m bolting on doors, at the hinges. Sometimes the holes are misaligned and I have to force the bolts in. That slows me down, and it slightly misaligns the door itself, leads to normal stress on the door that it wasn’t designed for. Early failure of the latches or leaks through the seals, something like that. But I can still bodge it to fit, the doors will open and close and latch when the car is new, enough to pass inspection.

        Under the UAW/Detroit system, if I raise this concern, I might get yelled at to stop whining and get my work done. Alternately, I might get my friend fired for drilling holes badly. Even if they focus on the problem and get it fixed – now I can do my job faster, and six months from now management may notice and cut someone from my department – they need to get labor costs down, after all. So why should I say anything? Is the inspector really going to taking calipers to every bolt hole, or is he going to just check that the doors open and close?

        Under the Toyota system, my friend and some engineers will work to figure out a new jig to make it easier to drill the holes consistently, and I will get at least recognition, maybe a small bonus, as will my friend. If this leads to my department being cut because our job is easier, management will do their best to find me some other role. That’s easier for them to do than in Detroit because they’re already ahead and growing, and they haven’t signed a contract locking people into their roles. They can shuffle people into the new factory they’re building and find homes for everyone, even if it’s fewer people per car off the line. If I trust that raising a concern will improve the company *and* that improving the company will help me and my friends…well now I’m willing to raise the concern. And all the doors will close properly without stressing the hinges and the seals.

        But how do you get from Company A to Company B? The whole reason the union insisted on the tight contract was that sometimes management would reassign workers to make them slow to provide an excuse to fire them. You have to solve the chicken and egg problem where you don’t have discretion because they don’t trust you, and you can’t prove you’re now trustworthy because you don’t have any discretion. If you’re trustworthy; maybe the way to get promoted is to have a department that’s cut costs, and who cares what that does to the union’s attitude next time the contract expires?

        Second: especially in the short run, it was easier to try to compete with Japanese prices than with Japanese quality. If your labor costs more but you’re aiming for the same final price, then you have to cut on non-labor costs. Thinner metal, looser tolerances, fewer parts, less engineering, less testing, less rework.

        • David W says:

          As an aside, Scott’s initial comment: “why it was so hard for Americans to do even when the Japanese basically handed their US competitors all their procedures on a silver platter.”

          How hard is it for a couple to improve their marriage even when their in-laws keep giving them well-meaning advice? You can describe a happy marriage all you like, but that doesn’t fix the problem. My understanding of the American car companies is that the fundamental problem was complete and total earned lack of trust, going both ways, between labor and management. It wasn’t something that could be fixed by just following the recipe.

          • CatCube says:

            I like this metaphor.

          • Mabuse7 says:

            Probably something like the German model combined with Joseph Studwell’s export discipline. State-backed industrial financial institutions providing easy capital to both old and new industrial firms and public-private industrial R&D institutes developing innovations shared by the whole domestic industry coupled with ruthless culling of any business or industrial segment that proves uncompetitive in the export market.

      • SEE says:

        How does a union contract reduce the quality of the car? I’d understand it if it made it more expensive.

        You literally can’t follow Toyota’s production and quality control procedures if the contract-set work rules and dozens of job classifications are incompatible with them.

      • bean says:

        How does a union contract reduce the quality of the car?

        Have you ever heard of British Leyland?

        Basically, if the workers don’t care, and the management doesn’t have any means to make them care, they’ll build terrible cars.

  66. AlphaGamma says:

    Another interesting piece of information about the McLibel case- Keir Starmer, who acted for the defendants pro bono, is now leader of the Labour Party.

  67. kokotajlod@gmail.com says:

    I think it is a huge disservice to the Aztec and Inca to call them Stone Age empires. It is true that they did not have iron or steel, but they had a sort of bronze. In my opinion their general level of technological sophistication seems to be similar to that of the bronze age empires.

    • John Schilling says:

      I don’t think the Aztecs had bronze, except perhaps tiny quantities acquired by indirect trade with the Incas. And the Incas never had enough bronze for broad utilitarian purposes.

      Pragmatically, I wouldn’t count a civilization as “bronze age” until e.g. every professional soldier has at least a bronze spear-tip.

    • bullseye says:

      From a quick look at Wikipedia and Google, it looks like we shouldn’t be lumping the Aztecs and Incas together. Both empires used stone, copper, and bronze, but I get the impression that the Aztecs used a lot more stone and the Incas used more metal. The Inca made maces from metal (which seems like a job where stone would do very nearly as well), and the Aztecs made “swords” from stone and wood.

    • Douglas Knight says:

      Bronze Age isn’t about bronze. It’s about a bundle of technologies that occurred together in Eurasia, probably because it they were copied. The New World progress doesn’t match Old World progress, so it doesn’t fit well into the system. The Copper Age is probably the best fit. The Aztec and Inca technologies aren’t good matches for each other, either. The Inca were ahead of the Aztec on everything except probably writing.

  68. zzzzort says:

    CRI is a weird metric. Candle light has a CRI of 100, because it’s a blackbody spectrum, but colors won’t look ‘right’ because the temperature is so low. For most applications getting the temperature right is more important, imo.

  69. hnau says:

    538 on which states have produced the most presidential nominees. Did you know eight nominees have come from the western US, and all of them were Republicans? Why should that be?

    My guess: The 538 statistics suggest presidential nominees disproportionately come from large and politically influential states (New York, Illinois, California) as one might expect given party politics and the Electoral College. Given that nominees tend to be seasoned politicians we might also expect nominee numbers to be a lagging indicator of a state’s size and political leanings. California leaned Republican in national politics until quite recently (say 1970) and produced 6 of the 8 nominees in question, so given the small sample size this doesn’t seem like a huge surprise.

    • grendelkhan says:

      This is a misleading headline; the school intends to produce its own entrance exam, which will just raise money for the university and costs for applicants. It’s marketing, not institution-destroying, and this sort of both-sidesism is strongly redolent of XKCD 774.

      • Douglas Knight says:

        Which headline?
        “full elimination or a new UC test by 2025”?

        The system is only considering creating its own test. For the next two years, the SAT/ACT will be optional, with no replacement. For the two years after that it will be banned in state and optional out of state. Do you have a source claiming a new test before 2025? Why are you so sure that there will be a new test even by then?

        What do you put at the odds that this decision will be reversed?

        Conditional on it continuing, what odds do you put that it does produce a new test? Lots of people have promised tests and not delivered. People who believed in tests would not drop the test before rolling out the new test. Whereas, the new period is an experiment to see what removing tests is like.

        Conditional on it creating a new test, what are the odds that after seeing it you would endorse the phrase “institution-destroying stupidity”?
        the odds that I would endorse the phrase?

      • albatross11 says:

        There is a pretty long history of liberal opposition to using standardized tests for admissions, because the demographics don’t come out right. As best I can tell, for all their faults, using SAT or ACT scores to make admissions decisions is one thing that led to a massive democratization of elite schools–the smartest, hardest-working people tended to be a lot more likely to end up in a top school.

        There’s also a long history of people trying to make new standadized tests that don’t yield the wrong demographics for admissions/hiring/etc. This never actually works.

        My prediction is that no longer including standardized tests in the application will make the UC system student body whiter, since it will disadvantage Asian students, and wealthier, since many of the other things you can do to enhance your chances of getting into a top school involve having your parents provide opportunities for you.

        My second prediction is that, if the UC system eliminates testing for admissions entirely[1], they will slowly lose their elite reputation, as the average student (and then the average graduate) becomes much less impressive over time. But that will take a decade or two, so as long as everyone agrees not to talk about or notice what’s going in, the current set of decisionmakers will be out of power long before the consequences really start to hit.

        [1] If they try to make a new standardized test that gives the right demographic answers, they’ll either fail or they’ll succeed by making it a very poor test of academic knowledge or intelligence, and there’s a lot of hostility to standardized tests anyway, so it’s quite plausible to me that the outcome will be “drop testing altogether and use class rank, AP classes, essays, interviews, and extracurriculars” to choose students.

        • One other prediction is that the UC schools will become less accessible to home schooled students. At present, the SAT (or ACT) is the best substitute for high school grades and recommendations available to them.

        • mtl1882 says:

          As someone who has tutored these exams for ten years, I think the original SAT was valuable and helped democratize things, even with the ability to “game” the system that tutoring provides. Better than any other metric, though far from perfect.

          A few years ago, they redid the SAT and made it more like the ACT. It is harder to differentiate yourself in the new format, and is in some sense more coachable, I think. To give one example, the vocabulary section is gone, replaced with a few vocab in context questions in the reading and grammar. While more well-off students are more likely to be drilled in vocab by parents and tutors, the fact is most kids don’t want to learn obscure vocab, and most privileged parents don’t make them do it (a small number of elite families take it very seriously—these kids are the best-positioned). So it only goes so far. What makes a real difference is having read widely—voracious readers from all backgrounds could stand out in the verbal section due to the vocab component alone.

          It has been my opinion, from working in the industry, that these tests have become less and less useful to colleges, practically none of which require the essay now, since the new format is so difficult for most students and colleges don’t care about writing anymore. These tests were made for a higher education system more rooted in a classical education, and are just not super helpful. Attempts to make the test more career-oriented has made them cumbersome. The math part is okay by comparison, but I think the ship sailed on the tests a few years ago. I would not have supported modifying or doing away with the SAT’s old format.

          The higher education industry was going to be forced into a reorganization in this decade, even without COVID-19. My view was that fewer people would go to college, and there would be a relatively small number of small liberal arts colleges, alongside some big elite institutions. I believed that colleges that cared to do so would go back to making their own tests, or adopting a new classical education-based standardized exam getting traction with homeschoolers. With COVID-19, I feel like the debate over the SAT and ACT will seem like a luxury from another era. This will speed up and intensify the reorganization, and with many fewer applicants, and the abandonment of college as a universal goal, the battle over testing won’t be as intense.

          There’s something freaky about COVID-19 in how it almost mocks our most fraught debates by choosing for us, rendering them moot (an exaggeration, but still). I mean, after all the interminable agonizing over the existence of borders, with Brexit finally going through, everyone closes their borders, Johnson is derailed again, and Macron says the EU has to give up on the globalization stuff. After all the attempts to create an existential crisis for the Trump presidency, a real one sneaks up on us. After all the attempts to root out the radicals like Sanders and Yang and get back to the status quo, at the Dem. establishment and Biden’s moment of victory, we can’t help but play with ideas of massive welfare payments and universal healthcare, and the status quo is gone.

      • anonymousskimmer says:

        [1] If they try to make a new standardized test that gives the right demographic answers

        I would like a test that yields relative scores between the sub-components. You don’t need the absolute scores between test-takers, just the scores within a test-taker.

        Relative balance has shown to be a real-world indicator of predilection to complete or not complete. So if you can identify these imbalances, you can take preemptive measures to ensure you have the situation necessary for these students to succeed.

        https://cepa.stanford.edu/sites/default/files/ACT%20Paper%20%28final%29.pdf (note well that the authors’ policy recommendation isn’t the one I’m pitching here; I’m citing them for their data.)

        so it’s quite plausible to me that the outcome will be “drop testing altogether and use class rank, AP classes, essays, interviews, and extracurriculars” to choose students.

        Which means people like me don’t stand a chance. Oh well, I never would have applied to a school like UC in the first place given how ignorant I was during the application process. I undermatched.

        • SamChevre says:

          I’m another who got into a selective college (as a transfer from an open-admission college) just on the strength of SAT scores. (I didn’t attend high school–my SAT score and one semester of Lambuth grades was all I had.)

        • mcpalenik says:

          Wouldn’t the easiest way to do “well” become guessing randomly at every question?

          • anonymousskimmer says:

            There’s no “well”. There’s “what’s your profile”. Lying on the test through random guessing makes it less likely that the ‘accommodations track’ (first year classes, groups, tutorials, dorms) your school places you into will work for you, and thus increase your odds of dropping out.

          • mcpalenik says:

            Then, you’re not suggesting using balance as a metric for admissions but only to determine a track for your education?

          • anonymousskimmer says:

            Yes. For large comprehensives, at least. For smaller schools that can’t cater to every need some sorting would have to happen. Ideally said sorting would be done for the benefit of all applicants (through a system of schools approach, perhaps, where a student would apply to a system and state their ideal school[s], with the expectation that they may not be admitted to their ideal school).

            Re “balance”: Students who do better on the English/Math portion of the ACTs versus the Reading/Science portion tend to complete versus those who do better on Reading/Science versus English/Math. It is thus not “balance” that is preferred in the current systems, but not having one’s Reading/Science composite higher than one’s English/Math. This is oddly true even for the Sciences.

  70. tcheasdfjkl says:

    [pedantic nit]

    Reporters Without Borders has built The Uncensored Library on Minecraft, containing all the information banned by the repressive governments of the world.

    I have not explored this library but I am very sure it can’t possibly contain all such information, as a lot of such information is stuff they wouldn’t want to host and/or would be banned from hosting even by U.S. law. Countries that do lots of censorship tend to censor not just politics/journalism they dislike but also lots of more mundane illegal stuff (e.g. people selling drugs or sex online, copyright infringement) and also a fair amount of genuinely unsavory and/or horrifying Internet content.

  71. bullseye says:

    Regarding the library in Minecraft, why can’t repressive governments deny access to that Minecraft server? How is it different from them denying access to a regular website they don’t like?

  72. No One In Particular says:

    for example, in New York you could buy a $100K/year tutor to teach five kids full-time

    You could hire a tutor for $100k/year, but buying one would pose problems.

  73. No One In Particular says:

    discouraged talking, for fear of breaking the Eigth Commandment [against bearing false witness], and on some days communication was only conducted in sign language.

    It’s an odd interpretation of the Commandment that it applies only to spoken communication. But I suppose that might have helped them avoid COVID.

  74. grendelkhan says:

    Gwern’s list is wonderful. My partner and I had a chat, and came up with some more items that might go well there.

    Bullying and school violence are way down. The DOJ’s NCVS surveys students, and found a 74% decline in violent victimization (along with an 82% decline in theft victimization) from 1992 to 2010.

    In the same vein, the experience of queer or gender-nonconforming kids in high school is completely different now than it was in, say, the late 1990s. Younger gay people come out earlier than older ones did; this means less closetedness, which means less misery.

    TV shows have arcs now! It’s not just high-budget prestige shows like Game of Thrones or The Wire; it’s sitcoms like Parks and Rec. In that vein, there’s a lot more nichey, strange things available out there, like Dirty Money or The Good Place.

    Animation is much better. Compare, say, Captain Planet to Gravity Falls. Kids’ shows are well-made enough to be palatable to adults, e.g., Avatar: The Last Airbender.

    Sex toys are way better, and come in more varieties. (For example, women can buy a reliable orgasm machine off the internet where the primary complaint is that (NSFW) it works too efficiently.) You can skip the scary shop in the wrong end of town, and anyway, the shops are now clean and friendly, a la “Good Vibrations”, if you still want to shop in person.

    The “road trip music” experience went from a giant wallet full of CDs, to a CD full of MP3s, to a dedicated MP3 player, to Spotify or Pandora or whatever streaming service people use.

    Portable electronics are frequently waterproof, like ‘drop in the bathtub and it’s no problem’ waterproof, in ways that they weren’t five or ten years ago.

    Our food is more delicious; “Per capita spice consumption in 1966 was 1.2 pounds annually, while that figure more than tripled by 2015 to 3.7 pounds.”

    Consumer paint now doesn’t emit harmful volatiles when it’s drying. Low- and No-VOC paints were introduced in the 1990s, and have since become standard for pretty much any paint you find in Home Depot.

    The chickenpox vaccine was introduced (in the United States) in 1995; before that, getting chickenpox was a normal part of childhood, and now it’s just not.

    The widespread availability of instructionals on YouTube means that if I want to, say, have someone walk me through replacing the air filter on my particular model of car, that’s trivial to find. Same goes for explainers and educational content–being able to learn anything I want to, whenever I want to, is an everyday miracle.

    • From my standpoint, the fact that I can self-publish my books is another big plus. My most recent nonfiction book, Legal Systems Very Different from Ours, I shopped around to a few high status academic publishers. When none of them were interested I published it myself. Twenty years ago I would have had to spend a lot of time and effort approaching lots of publishers and might or might not have ever gotten published. And I can convert a book to an audiobook and self-publish that too, to get a substantial increase in readership. For my most recent novel I didn’t even bother looking for a publisher. Getting a respectable one would have been difficult or impossible.

    • grendelkhan says:

      Other items I didn’t think of in time to add in the edits:

      Low-fat ice cream has better texture and mouthfeel due to ice-structuring proteins, known as “antifreeze” proteins in fish, grown in yeast and renamed for obvious marketing reasons.

      Non-caloric sweeteners are better; neotame (2002) and advantame (2014) lack the off-flavors of older sweeteners like saccharin; sucralose (1998) similarly improved on taste, shelf-stability and safety of earlier sweeteners.

    • grendelkhan says:

      We had some more chats, and came up with some more, though the timeline goes a little back beyond 1990 here.

      Long-acting reversible contraception is widely available: implants in the US in 1990/2006, hormonal IUDs in 2000; these are much better than previously available methods like DMPA (lasts three months, heavy side effects) and or copper IUDs (less effective, less safe). Also, since around 2011, they’re available in the United States with no cost-sharing. You can also buy emergency contraception for as little as ten bucks off Amazon; it’s more expensive if you buy it at, say, Walgreens.

      Cosmetic dentistry in general is much improved; dental fillings are no longer obviously visible. Resin composite fillings looked better than metal amalgam restorations, but were leaky and prone to breakage; due to advancements through the 1990s and 2000s, they’re strong enough to be used on molars and last as long as metal ones do.

      Listening to music at home is way better! Sound quality since the introduction of the CD (first album released 1985) has gotten good enough that modern “audiophile” products are just snake oil. Headphones and earbuds are now cheap and good enough that people regularly lose and replace them.

      Sedation drugs are better; propofol (approved 1989) means you don’t need to give people benzodiazepines, it has fast onset and recovery, and the side effects are minimal. (It’s the one that lets you be conscious for a procedure but remember nothing.)

    • Gerry Quinn says:

      I quite liked licking stamps and winding car windows, though.

      • Lambert says:

        The problem with cars now is that all the fancy stuff only works when the electrics are turned on.

        I’d much rather plunge into a lake in a car with with manual windows than electric.

      • Edward Scizorhands says:

        Now I have to lick windows and wind stamps. 🙁

    • Liam Breathnach says:

      Road trip music didn’t start with a giant wallet of CDs, but with an untidy bundle of cassettes in the glovebox. CDs were a step-up.

      Hospital beds. Old ones were manually adjusted by a nurse for the patient to sit up. Now patients can do it themselves with a button.

      Deregulation of public utilities. Used to be a three year* (!) wait for a telephone to be installed in Ireland by the Department of Posts and Telegraphs.

      (*I can verify this – took that long for my parents in 1978-1981j.

      • albatross11 says:

        I suspect what we’ve seen is:

        a. Technology has gotten overall, not just in electronics but in other stuff.

        b. Deregulation/competition has made some stuff a lot better/more functional.

        c. Institutional dysfunction has gotten worse, and made some stuff a lot worse.

        d. The society is overall somewhat less flexible and able to change quickly than it used to be, particularly in stuff like regulation and land-use/NIMBY, and that’s also made a lot of stuff worse.

    • Lord Nelson says:

      I would like to add vegan food substitutes to this list. I’ve been lactose intolerant for the last 20 years, and it’s gradually increased in severity, eventually leading to me avoiding all dairy (not even buttering my pans).

      Ten years ago it was difficult to find vegan substitutes that tasted good. These days, it’s fairly easy. The worst ones (cheese slices, pudding) taste fine but have a texture that’s slightly off, or leave a bad aftertaste. The best ones (various ice cream brands) taste just as good as the dairy versions. Or better than dairy version, in the case of my favorite vanilla-mango swirl.

  75. skylabfield says:

    As per Scott Alexander’s note that Alcoholics Anonymous is not “necessarily better than any other equally-structured programs”, that is where the current scientific evidence points. See https://www.ncbi.nlm.nih.gov/pmc/articles/PMC5884451/ for the study or https://www.vox.com/science-and-health/2018/3/5/17071690/alcoholics-anonymous-aa-smart-lifering-study for a summary of the study.

    The big thing about the 2020 Cochrane report is that it’s the final nail in the coffin with the “Alcoholics Anonymous only has a 5% success rate” nonsense out there.

  76. benf says:

    In this hypothetical where you’re paying a private tutor 100k a year to tutor 5 students, who is paying the cost of the real estate required to have a place to teach them? The cost of materials and curricula? Is your tutor going to be equally good at teaching physics as phonics? Where are these kids getting lunch? Breakfast? Computers to learn keyboarding? Supervision for when the tutor goes on their lunchbreak? Supervision for if the teacher is a pedophile?

    Point being, the all-in cost of educating children is much more than the salary you pay any given teacher..

    • albatross11 says:

      How do test prep companies, private tutors, private music teachers, and homeschoolers handle this problem?

      • Nikitis says:

        They get payed more to handle those expenses + profit?

      • mtl1882 says:

        It’s not a directly comparable situation, but many test prep tutors either meet at the student’s home or at a public place like a cafe or library. Lessons are usually 1-2 hours, so long-term supervision and meals aren’t really an issue. Families usually buy the materials directly.

    • sharper13 says:

      The current cost is much more than the cost of the teacher.

      However, that’s not an argument that all of that additional cost is actually useful for the objective of helping the students learn. Most of what you describe is easily handled much less expensively without the modern “school” structure.

      • benf says:

        First, it’s not obvious that this is true at all. Secondly, assuming it IS true, how MUCH are the potential savings? Two percent? Five percent? Ten percent? Are those percentages worth the tradeoffs?

        And nobody has answered the point about specialization. The major value-added of a school is that you have many different types of teachers who can each teach a particular subject. You couldn’t just hire one private tutor, you’d need a team, and someone to supervise and select that team, and eventually you wind up just reinventing private schools, which make public schools look like a steal, cost-wise. And of course the whole point of public schools is that everyone gets to go, regardless of ability to pay, which makes society as a whole wealthier regardless of whether you actually have children in the school system or not.

        The public school system is a Chesterton’s Fence. Before you get out the sledgehammers, take a second to think through the reasons it might exist in its current form in the first place.

        • albatross11 says:

          My suspicion is that one thing that’s happening here is that there are a bunch of different things bundled together as public schools, most of which aren’t about education. Public schools educate, but also provide free babysitting, after school activities to keep kids out of trouble, free breakfasts (some places) and lunches, various propaganda/PSA kinds of activities, some social services, community activities (football and basketball games, for example), etc. And also there’s likely a fair bit of waste and featherbedding and dead weight carried around because it’s easier than firing them.

          If we just wanted education, we could get it a lot cheaper. But then we’d have to do without or find substitutes for those other functions.

        • , and eventually you wind up just reinventing private schools, which make public schools look like a steal, cost-wise.

          You are thinking of elite prep schools, which are a very small minority of private schools. The average private school costs less than the public school, not more, even without counting the implicit cost of the public school’s land and buildings.

        • Edward Scizorhands says:

          As soon as it’s known “we can’t get rid of this institution” the institution will attract all sorts of leeches and parasites and hangers-on.

          Some people who can’t recognize this about the educational system will instantly see it if I say it’s true of the the military system. And vice versa.

        • albatross11 says:

          People do hire private tutors for kids in some circumstances (kids training for the olympics or acting in a movie/TV show), so it’s not like this is something we can’t possible work out how to do. Also, it sure seems to me that we could do pretty well with maybe three serious tutors (one for language arts/humanities, one for social sciences/history, one for math/science) at a high school level. Add in specialist teachers for music and languages.

          Again, I think the main issue here is that we expect public schools to do a bunch of non-education functions. I’m sure there’s significant waste and corruption and featherbedding, too, but even if you eliminated all that (which never happens this side of heaven), you’d still have all those non-education functions of public schools that would need to either be done without or handled separately.

        • John Schilling says:

          And nobody has answered the point about specialization. The major value-added of a school is that you have many different types of teachers who can each teach a particular subject.

          I am skeptical of this claim. There is certainly some value to this, but at the level of primary and secondary education it is I think fairly minor. My mother, for example, went to a literal one-room schoolhouse with one teacher for every subject at every grade level K-12 simultaneously, and was adequately prepared to continue her education at an Ivy-league university. The idea that you need a math degree to effectively teach high school math, a chemistry degree to teach high school chemistry, etc, is mostly just credentialism. And even more so in the 21st century when specialized resources are a mouse-click away when needed, a substantial improvement over the state of the art in 1950s rural Minnesota.

          The major value-added of a school is the collateral day-care, or possibly the opportunity for indoctrination if you are so inclined.

          • Randy M says:

            I am skeptical of this claim.

            +1
            Ideally the teacher for any subject would have a deep understanding of the material and be able to guide the pupil as deep as their curiosity led them.

            But for all practical purposes, you can probably get away with someone who has taken the course previously.

            Saying that the average high school graduate is unqualified to teach most subjects at the middle school or high school level is implicitly admitting that the current system fails to produce graduates who retain much knowledge of the principles it purportedly exists to impart.

          • AlphaGamma says:

            My mother, for example, went to a literal one-room schoolhouse with one teacher for every subject at every grade level K-12 simultaneously

            Genuine question- how common were one-room schoolhouses that went all the way through to the end of high school? From what I heard from family friends who went to them (further east than Minnesota), they tended to finish at eighth grade. Students who wanted to study further would go to a high school in the local town and board with a family in town if necessary.

          • anonymousskimmer says:

            @Randy M

            Most PhD recipients are unqualified to teach high school classes in their discipline. Teaching takes practice.

            If you absolutely had to hire someone without a teaching credential to teach, you’d want to hire the people who have tutoring experience.

          • Randy M says:

            @Skinner,
            It varies by child, I think. You can have an expert unable to communicate to a lay person and makes it worse. For most teacher/student pairs, they can manage to communicate the basic concepts if they have patience.
            Then you have the students who are slower and may require an expert trained or experienced or gifted in teaching to help them learn or learn how to learn.
            Then you have the behavior problem children who don’t care and need motivation.

            But in most of these cases, it isn’t strictly necessary to have a high degree of competence in the subject matter to teach to the high school level, exceptions like foreign language and advanced mathematics aside. (Obviously assuming a competently written text book.)

          • Edward Scizorhands says:

            Didn’t a lot of us experience, as students, teachers who didn’t really know anything beyond what was in the textbook? I’m not sure that’s a good idea.

            For the highest level AP classes, you want someone with a serious understanding of the topic. I’m not sure a credential is the answer here, because there are lots of ways someone could have the needed knowledge without a degree in that field. Someone who took 3 chemistry classes in college probably has enough, even if their degree isn’t in chemistry (or even if they didn’t graduate college).

  77. From one of the links:

    Until the 20th century, American and European men—including physicians—believed that women did not experience sexual desire or pleasure,” wrote sex expert Michael Castleman, in Psychology Today.

    He had obviously never read any Victorian porn. Or the memoirs of Casanova. Or …

    Casanova even has a discussion of whether men or women enjoy sex more, and concludes that women do. Part of his evidence is revealed preference. He has observed childbirth and concluded that if he had to go through that as the price of sex he would remain celibate.

    • mtl1882 says:

      Yeah. I’ve read a ton of 19th century stuff that indicates a belief that most women experience sexual desire. Generally, it was indicated that they did not experience it as much as or as often as men, but not always. Also, Byron’s works feature sexually aggressive women. And while I’m sure ignorant physicians abounded, archive.org has tons of 19th century medical manuals written for other doctors and for regular people that acknowledge female sexuality.

      • This in a way links to LadyJane’s thread about people distrusting authorities. From the standpoint of a reader unfamiliar with the relevant facts, “sex expert Michael Castleman, in Psychology Today” sounds like an authority. In fact, his claim is obvious nonsense. That makes us less willing to trust other things we read that claim to be sourced to authorities.

        I came across another case yesterday, a book that mentioned something about my history in the context of discussing my father. What it said was not true. The author has been the foreign editor and National Economic Correspondent for Newsweek, is currently Deputy News Editor for Foreign Policy Magazine, clearly an authority. I’m easy to find online, so he could have checked the facts with me, but didn’t bother.

        • mtl1882 says:

          Yeah. My amateur historical studies are what marked a firm break with my inclination to trust experts of this kind, which had eroded somewhat already. Most just consistently promote “obvious nonsense,” even if they get a lot right. Usually because they are repeating earlier incorrect work, lack imagination, and can’t draw realistic inferences about human behavior, especially people who lived in a different value system. I’ve since had similar experiences in other fields. There are always notable exceptions, of course, but they aren’t always the most qualified—they just think more clearly than most, and may be experts in related fields or exceptional amateurs. With all the information we now have access to, it’s easier to know where the gaps are. As you point out, these mistakes are made by the most meticulously qualified people.

          I’m not saying I don’t trust any experts, or that I prefer non-experts. I just believe that most people don’t know what they’re talking about when they talk about things that don’t affect them directly, and that they tend to be biased by values held by their group. So where I really care about something being right, I have to look into it myself if possible, or find someone who has the ability to evaluate these things. And even when I’m being very careful and writing about something I understand well, I will often find that an offhand reference to something tangential turns out to be embarrassingly wrong. Usually I got it from some other expert or “common knowledge.” I’ve realized I have to obsessively check the most basic things if I’m going for total accuracy, and realistically it doesn’t always happen.

          • Does your distrust of expertise extend to hard sciences or math-adjacent subject? Personally I’m very interested in math-adjacent fields — math, physics, and theoretical computer science — and my impression of the experts there is extremely good. I believe I used to have too much trust in experts on other fields from my good experiences there. I’m curious on your impression because I want to know how much my good impression has been from a genuine difference in the expertise in math-adjacent areas and how much has been bias from being an insider, which includes better knowledge of who the experts are and what they actually believe (news reporting on these subjects can be terrible).

          • mtl1882 says:

            Does your distrust of expertise extend to hard sciences or math-adjacent subject?

            Good question. I added the vague qualifier “of that kind” to experts because I couldn’t figure out how to convey exactly who I meant. In general, I would say I do trust experts in those fields, but that is partly because I lack the expertise to see where they are wrong or research it myself. However, I also don’t pay as much attention to those fields in a technical sense, so I’m not really put into the position to evaluate them. Certainly the hard sciences have some contact with reality that makes them unable to stray too far from it on purely technical questions. That is a real difference. I just feel like those things are not usually what’s in dispute.

            When I do have to evaluate that sort of thing, it is usually when experts are being used to support certain policies. I think things tend to break down at this level, because experts in hard sciences aren’t usually experts on how those things should be applied. And even when they are, policymakers tend to present them out of context or selectively, which we’ve recently seen a lot of. There’s also the problem of experts in one field assuming they are experts in others, and a lot of regular people deferring to that assumption.

            Related to this, I have had some unsettling experiences related to medicine/doctors–they knew what they knew, and were experts with regard to those things. There are things you have to actually understand to be a practicing physician, or your career will be over quickly. But they lacked the ability to see the implications of a lot of things, or take the perspective of someone not like themselves. This rendered their judgment kind of useless despite their expertise. I’ve also seen a lot of people use statistical models in a completely misleading way, even if the data is correct, which makes me wary. Basically, expertise is usually just a lot narrower than people realize. There is often a pattern to the blindspots, so you can learn what questions they are able to answer accurately.

            I’m curious on your impression because I want to know how much my good impression has been from a genuine difference in the expertise in math-adjacent areas and how much has been bias from being an insider, which includes better knowledge of who the experts are and what they actually believe (news reporting on these subjects can be terrible).

            Yeah, news reports will always make things look more absurd than they are. I find that the biggest problem is that so many experts essentially behave like reporters, paraphrasing, misinterpreting, and garbling the work of other experts in absurd ways. I imagine there are a lot of people in math-adjacent areas who act as middlemen and who make this mistake as well. But if the calculations are actually being used to build important things, then I would expect the experts typically know what they are doing. As Taleb says, skin in the game is crucial. If you are an acknowledged expert in a field in which your errors have consequences and are easy to discern, you probably know what you are doing.

          • albatross11 says:

            I think it also matters whether your field is one in which being wrong has visible consequences. Engineers whose buildings fall down and whose bridges collapse, doctors whose patients die, laboratory scientists whose experiments fail to replicate, those all have (in descending order) some opportunity for reality to tell us whether they understand what they’re doing or not.

            As you move into harder-to-verify territory, you get more and more of people being judged by being convincing to the other experts in their field–mathematicians at least have to be convincing in a very demanding logical realm of discussion, whereas social scientists usually have to be convincing in a mix of narrative sense and observational sense. The more complex the models and the less direct opportunity for contradiction by reality, the less tethered any expert field is likely to be to reality. Consider macroeconomics, or sociology, or philosophy. Lots of smart, thoughtful people work in all three fields, but you’re overwhelmingly being judged by whether what you say is convincing to your peers in the field, with only very rare resort to an opportunity for reality to just demonstrate that your theory is wrong.

    • Watchman says:

      I’m inclined to wonder how one qualifies as a “sex expert”.in the eyes of a popular science journal. Besides the non-academic practical route (which presumably is non-credentialed), I guess it’s in the psychology/medicine field. It would not exactly be unheard of for someone working in that sphere to assume a factoid such as the establishment not believing in female sexual desire, which strikes me as likely to originate with first-wave feminism seeking a target to attach in this area.

      In general the historical record shows the opposite view: men believed female sexuality was dangerous and had to be tamed (as the better feminist writers recognised).

      • anonymousskimmer says:

        After reading your first sentence, in retrospect it’s notable that he was identified as a “sex expert”, and not a “sex historian”.

  78. grendelkhan says:

    I still don’t have a good feeling for exactly what the Japanese advantage was and why it was so hard for Americans to do even when the Japanese basically handed their US competitors all their procedures on a silver platter.

    The non-English-speaking methods of producing infrastructure are hardly secret, but American institutions just… don’t look. See here, under section nine, “global incuriosity”; I don’t know if this is a consequence of rampant exceptionalism or what, but it’s real:

    American business culture does not care much for imitation, not does American society give high prestige to people who perfect something that someone else invented. The industry that teaches how to adopt best practices, consulting, has poor reputation in American culture. Instead, Americans venerate founders and innovators, an approach that works in industries where the US is in the global frontier, like tech or retail, but not in ones where it lags, like cars and the entire public sector. To avoid learning from others, Americans end up believing in myths about what is and isn’t possible: they insist they are so much richer than Europe that they have nothing to learn from across the Pond, and hang all their hopes on any flim-flam artist who comes from within American business culture who insists there is no real need for public transit or any of the other things Europe and high-income Asia do better.

    It makes sense that American business culture was so insular that the Japanese could come in and easily eat our lunch, and so they did.

    • SEE says:

      No. GM didn’t form a joint venture with Toyota (Fremont NUMMI) because they were sure they were right, nor did GM start Saturn to keep doing things the same old way, nor did Ford buy a quarter of Mazda and hand over Flat Rock to them because they thought they had nothing to learn.

    • Skeptical Wolf says:

      Setting aside, for a moment, the irony of citing an article about subways in a publication called Pedestrian Observations on the topic of car manufacturing practices…

      The cited section misscharacterizes US business culture badly in service of scoring a culture war point that is more explicitly stated after the excerpt. In reality, US businesses study their international competitors extensively. The 90s Toyota studies, in particular, have become foundational to modern management techniques (they were generalized into Lean Manufacturing, spawning Six Sigma in the process, then further generalized into Lean Management, which is still one of the stronger management trends).

      Also, in the areas where Japanese cars tend to outperform US cars, they also tend to outperform European cars. But European businesses are the good example that article suggests Americans should meekly follow (again, its sweeping and inaccurate statements are being made in service of a point about subway building, not automotive manufacturing).

      I don’t understand the seemingly endless demand for just-so stories attributing various intellectual and moral failures to the United States. Having such works be popular in nations that see themselves as US rivals would be understandable, but I am puzzled by how many get written for a US audience.

  79. John Lynch says:

    Regarding the Scunthorpe problem-

    My last name gets censored all the time online. Even when I don’t, the workarounds are unsatisfactory.

    Facebook wouldn’t let my author page be “johnllynch,” but “lynchjohnl” was OK. ????

    https://www.facebook.com/lynchjohnl

    It’s incredibly annoying, and wtf am I supposed to do? Change my name? It’s not like it’s a “white” name, either. Plenty of black folks are named “Lynch” and presumably have the same problem.

  80. 1 says:

    Kazakhstan’s population is about 24% ethnic Russian according to the 2009 census. The IQ vs ethnicity data will be interesting to see if available.

  81. 1 says:

    Hi Scott, do you intentionally include some typo like repetition of some words or phrases? E.g. there’s one here

    …American babies born in low-income families to have the same to have the same opportunities for attentive health care as [Cuban babies] will have…

    I have seen such repetitions multiple times in your blog.

    I remember you ran some experiment/survey which showed that people don’t notice such repetitions. If you’re still doing this this to confirm that hypothesis then can you please stop? Such mistakes are jarring and screws with the smooth flow of the text.

  82. Watchman says:

    On the conquistadors, I think there’s an argument that whilst they were startlingly successful they in fact repeat a common European pattern, whereby effectively-private military forces regularly overturn existing political units. A few examples:

    The late Roman Republic, where magistrates effectively waged private wars to conquer established states in the eastern Mediterranean. Note once the Principate was established the Empire basically stopped adding conquests (Britain and Dacia in the first and second centuries, and Armenia on a regular basis) despite having a (sometimes) united military command.

    It is likely the fall of the western part of the Roman Empire also was caused by small armies (the Visigoths were apparently kept out if the Auvergne by about 20 horsemen although this was possibly more a show of willing than a war), with individual leaders forming groups around them.

    The Vikings are the example par excellence, knocking out the two most-established Anglo-Saxon kingdoms and being able to take a large chunk of the heartland of the Frankish Empire (Normandy) with what were arguably armies of less than a thousand warriors, and with leaders with no obvious connections to states. The same pattern of forming alliances and exploiting internal divisions as the conquistadors is particularly clear here.

    Normans then somehow conquered Muslim Sicily and Byzantine Apulia, again with a number of leaders using small military forces which were diplomatic as well as military tools.

    We might consider the First Crusade as a classic example of this, albeit with unusual organisation. The crusade was effectively a collection of independent groups with their own leaders, who cooperated and somehow defeated the established local states and also out-manouvered their Byzantine allies.

    It is worth noting that the same pattern can also be seen in the later European imperial history, or at least the British part of it (I don’t know the other empires’ histories that well). Names like Clive in India and Rhodes in South Africa come to mind.

    There is a repeated pattern in all of this. The conquistadors of every age had their successes against developed states with strong centralised government. Where they faced fractured power structures (the Vikings in Ireland for example, or the very slow conquest of the big-city cultures of the New World) then we don’t see the same success, with conquests generally driven by state agendas. Likewise, where another active disruptor group were present (the Zulus for example) conquistador activities do not seem to have taken place.

    I’m not sure if this pattern of behaviour is a European peculiarity, but can at least suggest a reason why it happened. All the examples here were of military units effectively constituted through loyalty to the leader, often including choosing to follow that specific leader, and even appointing them. This differs from most forces they faced who were fighting for a ruler with whom they were not necessarily personally linked, even if they too had sworn oaths. It might be that centralised power structures, at least in non-mosen societies, were vulnerable to well-organised and united small groups of outsiders because they were able to operate outside the normal rules whilst the defenders of the centralised power structure were less likely to be committed to their cause.

  83. TJ2001 says:

    On the subject of massive military success of the conquistdors and many European nations of the time… One factor is overlooked….

    By the 1500’s – quite a few European powers were making attempts to organize their military powers based on skill and success rather than based on family ties and political alliances. This is VERY different from the usual “Tribal” or “Royal” systems used most everywhere in the world which vested all the command and leadership positions in the hands of “The Family” independent of their skill and training with such matters.

    And when a highly determined force organized around considerable skill, expertise, and training fights against forces lead by people who got their jobs because of nepotism and bribery – it usually goes very badly for the latter.

    So then you would ask “Why would anybody do it any other way?” Of course the answer is obvious…. The main risk isn’t invasion – it’s revolution.

    A very highly trained and expert military lead by an extremely competent and highly trained General Officer Corps could probably conquer their own nation more or less at will – which often makes the “Political” and hereditary leadership very uncomfortable…. And the “obvious” solution is to install family members who will make sure things don’t get out of hand quickly..

    • Aapje says:

      The Romans did this as well, resulting in a very long lived empire, even though their political system was mediocre.

  84. Wolfobert says:

    What actually ARE the great algorithmical advances in optimization and numerical linear algebra in the last 20 years?

    I have a PhD in applied math, and all I ever came across were (sometimes really fancy) methods for specific problems like TV-regularization, but never anything that generalizes to a reasonably broad class of problems.

    Now, in industry, everyone just seems to use whatever scipy.optimize function does the job. Those are usually based on 70ies FORTRAN code, as far as I know.

    Any neat new hammers I should know about when I find a fitting nail?

    • albatross11 says:

      My understanding is that SAT solvers and similar stuff have gotten a *lot* more optimized over the years. A fair bit of work in symmetric cryptanalysis involves reducing a cryptanalytic problem to a SAT problem and then feeding it to a SAT solver.

      Similarly, I think the techniques for doing automated proofs of correctness of code have gotten much better, so that some nontrivial things have been proven correct without some kind of heroic moonshot-level effort.

      • Loriot says:

        A fair bit of work in symmetric cryptanalysis involves reducing a cryptanalytic problem to a SAT problem and then feeding it to a SAT solver.

        Have there actually been any successes doing that? Crypto is one of the areas where SAT solvers are *least* suitable.

        SAT solvers are a bit like swiss army knives, or maybe a hobbyist 3d printer. They aren’t the best at any particular task, but they are decently good at a large number of things, so if you run into a problem that is both novel and easy, SAT solvers can handle it without requiring you to go to the trouble of writing a more efficient bespoke solver.

        Needless to say, this is pretty much the opposite of what cryptanalysis looks like. They can tell you if a new algorithm is obviously broken, but they won’t help you push the state of the art at all. If a SAT solver can make any headway against your algorithm, it was doomed from the begnning.

        • albatross11 says:

          No, you have to set up the cryptanalytic problem to be something in the form a SAT solver can help you with.

          Suppose you’re trying to find a collision for a hash function like SHA1. You have a difference pattern in the message (which bits may change) that’s determined by how the 16-word message is expanded into 80 words for the hash computation. If you just took random pairs of messages with those bits flipped, though, the probability that you’d get a collision (the probability that the pair would follow a specific differential characteristic you wanted all the way through) would be really low. So you start setting conditions–this bit needs to be a one, that bit needs to change from a 1 in M to a 0 in M*, etc. Getting the characteristic to hold for the whole hash requires satisfying a ton of these conditions. You can set that problem up as something that’s fed into a SAT solver, and get a solution that lets you find the collision. This is a link to an early paper using this idea.

          You’re not just feeding a description of the hash function to the SAT solver and trying to get it to solve your problem–you’re doing a bunch of cryptanalysis to get down to a problem that can be solved efficiently by a SAT solver.

  85. notpeerreviewed says:

    Re: Hard-to-notice improvements, old-timey recipes for eggplant call for a somewhat lengthy salting-and-soaking process to draw out bitterness. That’s no longer necessary, as modern varieties have the bitterness bred out.

  86. notpeerreviewed says:

    This may be just faulty memory, but I think whatever technology they use to deodorize portable toilets has improved a lot since I was a kid.

  87. OriginalSeeing says:

    burned house horizon

    Rationalists have already been doing this.
    Event Horizon was built on the burnt ruins of the building where CFAR’s office used to be.

  88. Crake says:

    Wow, kind of stunned but a lot racist takes towards Kazakh people in this thread and by Scott himself. First of all, the steadfast belief in IQ shouldn’t be surprising given the crowd that gathers on SSC, that’s fine, I’m not here to argue against the usefulness of that metric, nor am I here to stir controversy.

    First of all, the notion that somehow Koreans/Russian should explain the IQ scores seen in the study. If you look up similar “studies” (hesitate to call them that) the scores of Kazakhs rise alongside the industrialization rates within the country during Soviet rule. Besides, most of the migration that happened during the Soviet rule was low skilled laborers (sent to create farmlands) and not the scientists, which for your reference inhabit Novosibirsk and population of Baikonur is so low that it couldn’t explain these differences.
    Second, most of these posters know next to nothing about Kazakhstan and posit things with high degree confidence. I wonder if I could apply Dunning-Kruger to most of you folks.
    Third, some of the posters here who allegedly spent time in Kazakhstan and use their very anecdotal impressions of Kazakhs to find the results surprising.
    Fourth, as a person from Kazakhstan I can assure you that the government is in the least bit interested in gaming metrics you seem to put so much emphasis on. The PISA scores is what they look at the assess the quality of our education, which is admittedly lacking. (Cue in takes that tell me how PISA scores and IQ scores correlate.)
    Fifth, I honestly don’t think it’s even worth mentioning this because I’m pretty sure I will see a million responses which claim that income and well-being don’t explain IQ scores (and it’s other way around) yada-yada, but you do realize that Kazakhstan is a nomadic country that was forcefully turned sedentary which resulted in hunger rivaling Holodomor?

    If you really want to argue with me more on “Kazakh intelligence” please look up the ethnic makeup of IMO/IPhO/IChO participants on count the number of Russians/Koreans among them. Aside from Igor Ganichev I know of no one.

  89. Percival412 says:

    So does anyone have an explanation for why Kazakhstan’s GDP per capita is so high using PPP per capita ($30,000 2020, IMF) and so low using Nominal per capita ($9,000 2020, IMF)?

    As a quick sanity check it looks like a fairly developed country rather than a stable developing country by GDP growth at 4%.

    https://en.wikipedia.org/wiki/List_of_Asian_states_by_GDP_growth

    As someone without an economics background this leaves me uncertain how to interpret the IQ findings.